Clerkship Qstream Flashcards

1
Q

An outbreak of bloody diarrhea accompanied by fever occurred at a day care center. Gram-negative bacilli that do not ferment lactose were isolated from 7 of 10 children with the illness. No common food had been shared among the children. However, an iguana was brought to the infected children’s classroom three days previously as part of a show-and-tell assignment and was confirmed by culture as the source of the infection.

Which ONE of the following is the MOST LIKELY cause of the outbreak?

Campylobacter jejuni
Salmonella enterica serovar Marina
Escherichia coli O157:H7
Shigella flexneri

A

The correct answer is Salmonella enterica serovar Marina

Explanation: Salmonella is a zoonotic pathogen that is a frequent cause of blood diarrhea in humans. Contact with reptiles such as turtles and iguanas has been associated with infection.

Other answers: All of the other agents listed can cause bloody diarrhea. Diarrhea associated with E. coli O157:H7 is not as likely as the others to be accompanied by fever, and is usually associated with undercooked beef or leafy vegetables. Campylobacter is generally contracted from food that has been contaminated with raw poultry. Shigella is a strictly human pathogen.

How well did you know this?
1
Not at all
2
3
4
5
Perfectly
2
Q

A 53-year-old Army colonel reported passing bright red blood per rectum. On colonoscopy, a sessile 3.1 cm diameter mass was observed protruding into the distal sigmoid colon.

This lesion is most likely which of the following?

A

Answer: Adenomatous polyp.

Explanation: The patient’s clinical history is most consistent with a left-sided intestinal polyp. An adenomatous polyp arises from the epithelial surface of the gastrointestinal tract. Adenomatous polyps are not a frequent cause of lower GI bleed, but they can present in this fashion.

Other answers: Meckel’s diverticulum is a congenital diverticulum in the ileum resulting from incomplete closure of the omphalomesenteric duct. Approximately half of Meckel’s diverticula have gastric mucosa. GI bleeding can occur in association with Meckel’s diverticula because the acid produced from the gastric mucosa in the diverticulum can produce an ulcer that bleeds. Meckel’s is not the correct answer because 1) it is not located in the colon, 2) it does not typically cause bright red blood per rectum (because the bleeding source is more proximal, the blood is either black or maroon in color in the stool), and 3) it typically presents in childhood.

A teratoma is a neoplasm originating in the testis, ovary or, rarely, the mediastinum. The neoplasm contains recognizable mature or immature cells or tissues representative of one or more germ cell layers. Usually all three germ cell layers are represented in the neoplasm.

A lipoma is a benign neoplasm of adipocytes. It can occur almost anywhere, and can occur in the colon. However, it is very unusual for intestinal lipomas to bleed. When intestinal lipomas become very bulky, they can cause intususseption and obstruction.

Robbins PBOD 8ed. pp 260-262

How well did you know this?
1
Not at all
2
3
4
5
Perfectly
3
Q

A 70 kg man who has been exhibiting symptoms of a stroke for 30 minutes is brought to the emergency room. You determine that administration of the thrombolytic agent alteplase is the appropriate course of treatment. The volume of distribution of alteplase is 0.1 L/kg, and its elimination half-life is 5 minutes. The product instructions state that an intravenous bolus dose of 70 μg/kg should be administered, followed by an intravenous infusion of 700 μg/kg over the next 60 minutes. What is the serum concentration immediately after the bolus dose?

4.9 mg/L
70 ug/L
700 ug/L
460 ug/L

A

The correct answer is 700 µg/L.

Explanation: The question asks for the serum concentration after administering a loading dose of 70 µg/kg.

To determine the serum concentration immediately after the bolus infusion of the drug, use the equation: Loading dose = Vd x TC, where Vd is the volume of distribution (the volume in which the drug will be diluted) and TC is the target concentration or the diluted concentration of the drug.

A simple way to think of this equation is by using the following units:

Loading dose in µg = Vd in L x TC in µg/L.

However, people of different sizes are going to have different volumes of distribution. So the units of Vd are often given as L/kg. Similarly, as the loading dose of a drug will also depend on the size of a person, loading dose is usually referred to as µg/kg.

Thus, we end up with the equation Loading dose in µg/kg = Vd in L/kg x Tc in µg/L.

In the question above, the Vd is given as 0.1 L/kg and the loading dose (the initial IV bolus) is given as 70 µg/kg. Thus, Tc = (70 µg/kg) / (0.1L/kg) = 700 µg/L.

How well did you know this?
1
Not at all
2
3
4
5
Perfectly
4
Q

In a study of trivalent inactivated influenza vaccine in adults aged 18–64 years, 24 of 2,000 vaccinated participants developed influenza, compared with 72 of 2,000 who received placebo. What is the relative risk of influenza in the vaccinated group compared to controls?

  1. 012
  2. 67
  3. 33
  4. 0
A

0.33

How well did you know this?
1
Not at all
2
3
4
5
Perfectly
5
Q

A young woman is brought to the emergency department by her friends, who say she participated in a witchcraft ritual in which she ingested belladonna potion (which contains atropine). She has a beet red face and dilated pupils. She is psychotic and difficult to handle and repeatedly pulls out her intravenous line containing diazepam. To reduce the belladonna-induced psychosis, you administer:

physostigmine
pralidoxime (2-PAM)
clozapine
pyridostigmine

A

The correct answer is physostigmine.

Explanation: To counter the blockade of muscarinic receptors by atropine, you should administer a cholinesterase inhibitor that can enter the brain. The cholinesterase inhibitor will bring about an elevation of endogenous acetylcholine levels that can competitively overcome the blockade of cholinergic muscarinic receptors by atropine. Since the patient has psychotic symptoms, an antidote that can reach the brain is needed. Although both physostigmine and pyridostigmine are anticholinesterases, only physostigmine can rapidly cross the blood-brain barrier.

Incorrect answers:

Clozapine is an atypical antipsychotic drug that binds to serotonergic and dopamine receptors. Additionally, clozapine can antagonize some cholinergic receptors, and thus could potentially worsen the effects of the belladonna potion.

Pralidoxime reverses acetylcholinesterase inhibition caused by organophosphate poisoning. By rescuing acetylcholinesterase, pralidoxime increases acetylcholine breakdown, and thus works as an anticholinergic agent. Since atropine is a competitive inhibitor of muscarinic acetylcholine receptors, pralidoxime would augment the toxic effects of belladonna (which contains atropine).

Pyridostigmine does not cross the BBB and therefore would not elevate acetylcholine levels in the brain. Thus, pyridostigmine would reverse the peripheral effects of atropine but would not reverse the central effects of atropine.

How well did you know this?
1
Not at all
2
3
4
5
Perfectly
6
Q

A high incidence of skin infections occurred among members of a high school wrestling team. Several of the teenagers were treated with intravenous nafcillin or oral dicloxacillin (semisynthetic penicillins related to methicillin), but the infections did not resolve. Gram-positive bacteria were isolated from pus within the inflamed skin lesions and the anterior nares of two team members, and bacteria of similar morphology were isolated from the wrestling mats and a tube of taping gel in the locker room.

All of the isolates were resistant to methicillin. Which ONE of the following is responsible for methicillin resistance in this organism?

An rRNA gene that underwent spontaneous mutation
Acquistion of a plasmid that encodes β-lactamase
Integration of an altered penicillin-binding protein PBP2a gene
An altered RNA polymerase

A

The correct answer is Integration of an altered penicillin-binding protein PBP2a gene.

Explanation: Most Staphylococcus aureus strains are resistant to penicillin by virtue of beta-lactamase production. Methicillin was developed as an antibiotic to overcome beta-lactamase. Like other antibiotics in the class, methicillin blocks the transpeptidation that cross-links the bacterial cell wall because it is an analogue to D-ala D-ala. Over time, resistance to methicillin has emerged because S. aureus acquired the mecA gene via a transposon (passed among S. aureus strains via conjugation). The mecA gene encodes for an alternative transpeptidase, penicillin-binding protein PBP2a, that has decreased affinity for methicillin. S. aureus strains with the mecA gene (PBP2a) are resistant to methicillin.

How well did you know this?
1
Not at all
2
3
4
5
Perfectly
7
Q

A 55-year-old female is admitted to an outpatient clinic with a 39°C fever and flank pain. She was recently discharged from a 3-day hospital visit during which she was catheterized. Many leukocytes and several white blood cell casts are seen in a microscopic examination of her urine. A Gram-negative organism that demonstrates a spreading morphology on blood agar was isolated in pure culture from her urine.

Which ONE of the following is the MOST LIKELY diagnosis?

Uncomplicated cystitis
Pelvic inflammatory disease
Pyelonephritis
Urethritis

A

The correct answer is Pyelonephritis.

Explanation: This patient was at risk of acquiring a urinary tract infection when hospitalized because she was catheterized. The symptoms of fever and flank pain are consistent with pyelonephritis and the appearance of renal casts, along with bacteria and white blood cells in the urine provides evidence that the urinary tract infection has ascended to the kidneys. Furthermore, the presence of a Gram-negative bacillus that exhibits spreading motility on blood agar suggests that Proteus mirabilis is the causative agent. P. mirabilis is notorious for its ability to persist in the urinary tract through pili and to ascend the ureters from the bladder due to its hyper-motility provided by this swarming mechanism.

How well did you know this?
1
Not at all
2
3
4
5
Perfectly
8
Q

A 45 year old man with long standing gastric acid reflux presents to his physician with a complaint of dysphagia. A biopsy of the lower third of the esophagus is performed. Examination of the biopsy by a pathologist shows the presence of columnar epithelium replacing the stratified squamous epithelium which typically lines the distal esophagus.

Which of the following choices describes the process present?

metaplasia
atrophy
dysplasia
hyperplasia

A

Correct answer: Metaplasia.

The question describes the replacement of the normal squamous epithelium of the distal esophagus with columnar epithelium. This phenomenon is known as metaplasia, defined as a change in a tissue from one cell type to another. Metaplasia of the distal esophagus from normal squamous epithelium to columnar epithelium occurs as a result of chronic gastroesophageal reflux.

Other answers: Atrophy refers to a decrease in size due to decrease in cell number or cell size. Dysplasia refers to changes in the epithelium where cells are pleomorphic and lose their normal orientation. Hyperplasia refers to an increase in cell number. Hypertrophy refers to an increase in cell size. Robbins PBOD 8ed. pp 10-11

How well did you know this?
1
Not at all
2
3
4
5
Perfectly
9
Q

During cold weather, many families use space heaters to stay warm. Space heaters, improperly used, lead to an increased risk of carbon monoxide poisoning.

You obtain a CBC, a pulse oximetry reading at the bedside, and an arterial blood gas (ABG) run on a co-oximeter. Which of the following best characterizes the findings one would expect in the blood of a patient suffering from acute carbon monoxide poisoning?

Normal hemoglobin levels, normal SaO2 by pulse oximetry, normal SaO2 and low PaO2 by ABG
Low hemoglobin levels, low SaO2 by pulse oximetry, low SaO2 and normal PaO2 by ABG
Low hemoglobin levels, normal SaO2 by pulse oximetry, normal SaO2 and low PaO2 by ABG
Normal hemoglobin levels, normal SaO2 by pulse oximetry, low SaO2 and normal PaO2 by ABG

A

Answer: Normal hemoglobin levels, normal SaO2 by pulse oximetry, low SaO2 and normal PaO2 by ABG

Hemoglobin has a higher affinity for CO than for oxygen. Therefore, CO binds to hemoglobin in erythrocytes and blocks oxygen from binding. Because CO blocks oxygen binding to hemoglobin, oxygen saturation of hemoglobin is low. CO poisoning does not affect hemoglobin levels. The amount of dissolved oxygen in the blood, measured as the partial pressure of oxygen on an arterial blood gas, is also unaffected by CO.

A pulse oximeter measures the percentage of hemoglobin which is saturated with oxygen (SaO2). However, a standard bedside pulse oximeter is unable to differentiate between oxyhemoglobin and carboxyhemoglobin. Therefore, standard bedside pulse oximeters typically do not read low in cases of carbon monoxide poisoning even though the true oxygen saturation of hemoglobin is low in this setting. In contrast to standard pulse oximetry, certain blood gas analyzers have co-oximeters that can directly measure concentrations of oxygenated hemoglobin, deoxygenated hemoglobin, carboxyhemoglobin, and methemoglobin as percentages of total hemoglobin. These units will report low oxygen saturation of hemoglobin in patients with CO poisoning. Please note, though, that not all ABG machines have co-oximeter functionality. Standard blood gas analyzers that lack co-oximetry calculate oxygen saturation of hemoglobin by inputting measured values for PaO2 and PH into standard oxygen dissociation curves. Like pulse oximeters, standard ABG analyzers may thus report erroneously normal saturation values in patients with CO poisoning.

For these reasons, when considering CO poisoning in a patient it is important to ask the respiratory technicians who run the ABG machines whether ABG values for oxygenated hemoglobin are being calculated or directly measured. At WRNMMC, the ABG machines in the ICU have co-oximetry capability to directly measure oxygen saturation. Thus, the oxygen saturation values reported from ABGs at WRNMMC are often directly measured values. However, for a week in February of 2013 the co-oximetry function on the ABG machines in the ICU at WRNMMC wasn’t working, and so the oxygen saturation values reported from the machines were calculated. So, in cases of suspected CO poisoning, it’s prudent to ask how the O2 saturation is being determined (calculated or measured) on the ABG machine that day.

How well did you know this?
1
Not at all
2
3
4
5
Perfectly
10
Q

The beginning of implantation occurs when the conceptus is at which of the following stages?

2-cell zygote
Morula
Blastocyst
4-cell zygote

A

Explanation: The blastocyst implants into the uterine lining at ~6 days. The human blastocyst contains from 70-100 cells. A blastocyst has an inner cell mass (the embryoblast), which will develop into the embryo, and an outer cell mass (the trophoblast), which will develop into a large part of the placenta.

The other options are earlier stages in embryonic development. During these stages of development the conceptus is located in the Fallopian tube. Implantation in the Fallopian tube is abnormal and may lead to an ectopic pregnancy. The morula has 32 cells, and is the point at which a zygote resembles a mulberry (from the Latin “morus” for mulberry). The morula is the stage immediately before the blastocyst.

How well did you know this?
1
Not at all
2
3
4
5
Perfectly
11
Q

An infant, born at 26 weeks gestation, is rushed into the intensive care unit at the hospital with significant respiratory difficulties. The consequences of the premature delivery on the respiratory system include all of the following except

severe arterial hypoxemia.
areas of atelectasis.
difficulty with inflation of the lungs.
atrophy of the respiratory muscles.

A

Ans: atrophy of the respiratory muscles

The premature delivery occurs before the full development of the lung alveolar structure, including importantly the Type II alveolar cells and the secretion of surfactant. This occurs at about 26 weeks (the terminal sac stage). Because surfactant is absent, surface tension is high and the normal inflation of the lungs is difficult. This increases areas of atelectasis in the lungs, reduces the exchange of gases, and reduce the levels of oxygen in the blood. Muscle development would be expected to be normal for this developmental stage and not be atrophied.

How well did you know this?
1
Not at all
2
3
4
5
Perfectly
12
Q

A 22 year old soldier presents with an acute abdomen with rebound tenderness . At exploratory laparotomy infarcted bowel is found secondary to a volvulus. The type of infarct that would result from this type of obstruction is due to which of the following mechanisms?

Proliferation of the endothelium at the site of the volvulus
Venous compression
Activation of the clotting cascade distal to the obstruction
Liquefactive necrosis in the infarcted area

A

Correct answer: Venous compression.

Explanation: The volvulus will preferentially compress the thinner-walled veins, occluding venous return, while arterial perfusion continues (even if to a lesser degree).

Other answers: In epithelial injury, the clotting cascade is activated at the site of injury, not distally. Liquefactive necrosis is seen in abscess formation and in ischemic injury to the central nervous system. A volvulus constricts vascular flow due to pressure of the twisting of the intestines, not a hyperplastic process. Robbins PBOD 8ed. pp 127-129.

How well did you know this?
1
Not at all
2
3
4
5
Perfectly
13
Q

A 45-year-old female received a nicotinic acetylcholine receptor antagonist to produce neuromuscular blockade during a major surgical procedure. At the end of the surgery administration of neostigmine and atropine to this patient will MOST LIKELY

enhance neuromuscular blockade and reduce the bradycardia caused by neostigmine.
enhance neuromuscular blockade and reduce the tachycardia caused by neostigmine.
reverse neuromuscular blockade and reduce the bradycardia caused by neostigmine.
reverse neuromuscular blockade and reduce the tachycardia caused by neostigmine.

A

The correct answer is , reverse neuromuscular blockade and reduce the bradycardia caused by neostigmine.

Explanation: To overcome the nicotinic blockade following the surgery, neostigmine is given to inhibit acetylcholinesterase, elevate acetylcholine at the neuromuscular junction, and overcome the nicotinic blockade. However, acetylcholine will also be elevated at muscarinic receptors where there is no block. Therefore, a muscarinic antagonist, like atropine is given to prevent the elevated level of acetylcholine from acting at M2 muscarinic receptors on the heart.

How well did you know this?
1
Not at all
2
3
4
5
Perfectly
14
Q

A 30 year old G1P0 woman presents to her physician at 30 weeks gestation with chief complaint of swollen legs and increased weight gain. On physical examination, her blood pressure is elevated. Urinalysis reveals the presence of protein. Labor is induced and a small for gestational age infant is delivered. On examination of the placenta, there is extensive infarction. Which of the following types of necrosis would be seen in the spiral arterioles that supply the placenta?

coagulative
fibrinoid
caseous
fat

A

The correct answer is Fibrinoid necrosis.

Explanation: Development of hypertension and proteinuria after 20 weeks of gestation is most consistent with a diagnosis of pre-eclampsia. While swollen legs can occur for a number of reasons during pregnancy, peripheral edema is also often a component of pre-eclampsia (though not necessary for the diagnosis). The exact mechanisms underlying pre-eclampsia remain unknown, but it is believed that both placental hypoxia and inadequate immune tolerance towards the placenta (resulting in increased maternal immune response to paternal antigens in the placenta) are involved. Pre-eclampsia can progress to eclampsia, a life-threatening condition which includes the development of generalized tonic-clonic seizures in the mother. The only cure for pre-eclampsia is delivery by either Caeserean section or induction of labor. Examination of the spiral arteries in pre-eclampsia typically reveals fibrinoid necrosis. Fibrinoid necrosis is characteristic of immune–mediated injuries.

Other answers: Caseous necrosis is associated with granulomatous inflammation associated with tuberculosis and fungal infections. Coagulative necrosis is seen in organs, except for the central nervous system, when there is ischemic injury. Fat necrosis is seen in acute pancreatitis. Robbins PBOD 8ed page 16

How well did you know this?
1
Not at all
2
3
4
5
Perfectly
15
Q

A 50 yo male presents for a routine clinic visit. He is a nonsmoker, drinks 2-3 glasses of wine a week, and is mildly overweight with a BMI of 28. His systolic blood pressure has been running in the high 130’s to low 140’s with diastolic pressures in the mid 80’s for the past few years. He has been trying to improve his diet and has only been exercising sporadically. Which of the following would include only evidence-based US Preventive Services Task Force recommended (A or B) clinical preventive services for this patient?

Screen for Type 2 Diabetes
Screen for abdominal aortic aneurysm
Screen for prostate cancer with a PSA
Screen for heart disease with an ECG

A

Correct answer: A, screen for diabetes.

Explanation: The USPSTF recommends screening for type 2 diabetes in asymptomatic adults with sustained blood pressure (either treated or untreated) greater than 135/80 mm Hg, a “B” recommendation.

The USPSTF recommends one-time screening for abdominal aortic aneurysm (AAA) with ultrasonography in men ages 65 to 75 years who have ever smoked, a “B” recommendation, but no recommendation exists for this patient’s age.

The U.S. Preventive Services Task Force (USPSTF) recommends against prostate-specific antigen (PSA)-based screening for prostate cancer, a “D” recommendation, which means that the harms outweigh the benefits.

The USPSTF recommends against screening with resting or exercise electrocardiography (ECG) for the prediction of coronary heart disease (CHD) events in asymptomatic adults at low risk for CHD events, a “D” recommendation. Even if you think he is at high risk for CHD, The USPSTF concludes that the current evidence is insufficient to assess the balance of benefits and harms of screening with resting or exercise ECG for the prediction of CHD events in asymptomatic adults at intermediate or high risk for CHD events, an “I” recommendation.

How well did you know this?
1
Not at all
2
3
4
5
Perfectly
16
Q

A dehydrated 25 year old serviceman comes into the clinic with acute diarrhea and leg cramps having returned three days prior from deployment to Haiti. You suspect Cholera may be the cause and immediately start intravenous fluids. The toxin released by Vibrio cholerae causes the symptoms of Cholera by covalently modifying which signaling molecule?

The monomeric G-protein Ras
The second messenger cAMP
The guanine nucleotide exchange factor SOS
A subunit of a heterotrimeric G protein

A

Answer: a subunit of a heterotrimeric G protein

Cholera toxin acts by covalently modifying the Gαs subunit of a heterotrimeric G-protein. This modification prevents the Gαs subunit from hydrolyzing GTP to GDP, and thus the Gαs subunit is always “ON” and the signal cannot be terminated. This causes a rise in cAMP level, activation of Protein Kinase A, and subsequent dephosphorylation of the cystic fibrosis transmembrane conductance regulator (CFTR) chloride channel, causing efflux of chloride ions out of the cells followed by secretion of water, sodium, potassium, and bicarbonate into the intestinal lumen. This results in massive diarrhea, with a loss of up to two liters of fluid per hour.

How well did you know this?
1
Not at all
2
3
4
5
Perfectly
17
Q

A 30-year-old male presents in the emergency room complaining of chest pain and shortness of breath. The triage nurse orders a chest radiograph prior to your examination of the patient. You look at his portable chest radiograph on the way to the exam room and see significant leftward shift of the mediastinum and no lung markings on the right side. He is hypoxic with distended vessels along the right side of his neck and there are diminished breath sounds over his right hemithorax. Your next best step would be:

Place a left anterior axillary line thoracostomy tube
Perform a right anterior chest wall needle decompression
Obtain a chest CT to evaluate for vascular compression
Intubate the patient for ventilator support

A

CORRECT ANSWER: Perform a right anterior chest wall needle decompression of a pneumothorax is the correct answer for this question.

EXPLANATION: The complaint of chest pain and shortness of breath combined with hypoxia, jugular venous distention and diminished breath sounds over the right hemithorax suggest a pneumothorax. The chest radiograph demonstrating a right hemithorax lucency (ie. no lung markings) with leftward shift of the mediastinal structures further implies a tension pneumothorax. Although a thoracostomy tube will likely eventually become necessary, the best next step would be an emergent right-sided needle decompression along the anterior chest wall at the midclavicular line over the 3rd rib using a large bore needle.

OTHER ANSWERS: The clinical presentation, physical exam, and radiographic findings all suggest a right-sided pneumothorax. Intubating the patient for ventilator support will not decompress the pneumothorax that exists within the right hemithorax. This option would also be premature in a patient that is still conscious not requiring ventilator support.

Obtaining a chest CT will end up wasting valuable time further delaying decompression of the patient’s pneumothorax. A chest CT might be considered if there is diagnostic confusion in a clinically stable patient.

All of the findings listed in the question stem indicate a right-sided pneumothorax. Placing a thoracostomy tube on the left would be incorrect and will only result in bilateral pneumothoraces. Although this patient will eventually need a right-sided thoracostomy tube.

How well did you know this?
1
Not at all
2
3
4
5
Perfectly
18
Q

A 76-year old male with a history of hypertension was seen in an emergency room with a complaint of sudden-onset weakness of the right arm. Upon arrival he had difficulty speaking and was only able to utter single syllables with great effort. Examination revealed weakness, hypertonia, hyperreflexia and loss of sensation on the right upper limb. The corneal (blink) reflex was present bilaterally, but there was weakness of the musculature on the lower half of the face on the right side.

The observed neurological deficits are MOST LIKELY due to a stroke involving the

right anterior cerebral artery
left middle cerebral artery
left anterior cerebral artery
left posterior cerebral artery

A

The clinical signs are consistent with a lesion involving the lateral aspect of the frontal lobe on the left side, which is in the territory of the MCA. Damage to Broca’s area causes motor aphasia and damage to the primary motor cortex on the lateral aspect of the hemisphere causes weakness of the right upper limb and lower half of the face.

Other answers: Stroke of the anterior cerebral artery often results in opposite leg weakness. Among the many other possible presentations of anterior cerebral artery stroke are anosmia, apraxia (inability to execute purposeful movements), and sensory deficits of the opposite lower extremity. While the clinical symptoms associated with stroke of a posterior cerebral artery can also be varied and depend on the exact location of the occlusion, they often include visual field defects opposite the side of the lesion.

How well did you know this?
1
Not at all
2
3
4
5
Perfectly
19
Q

You are caring for a patient who requires mechanical ventilation. The ventilator is currently delivering 10 breaths per minute and a Tidal Volume (VT) of 500 mL. On morning work rounds the attending physician says “the PCO2 is too high, increase the Minute Alveolar Ventilation (VA) by 25%.” Assuming the patient’s Anatomic Dead Space (VD) to be 100 mL, which of the following ventilator settings will achieve a 25% increase in Minute Alveolar Ventilation (VA)?

Ventilator Rate - 10 breaths per minute, Tidal Volume – 625 mL
Ventilator Rate - 15 breaths per minute, Tidal Volume – 600 mL
Ventilator Rate - 10 breaths per minute, Tidal Volume – 600 mL
Ventilator Rate - 15 breaths per minute, Tidal Volume – 500 mL

A

Answer: Ventilator Rate - 10 breaths per minute, Tidal Volume – 600 mL

Minute alveolar ventilation is the product of respiratory rate x air exchange volume. To calculate this volume, we must substract out the dead space volume which does not come in contact with the lung alveoli. In this example, the initial MAV would be (500-100 mL) x 10 breaths per minute = 4000 mL. A 25% increase would be (600-100 mL) x 10 bpm = 5000 mL.

How well did you know this?
1
Not at all
2
3
4
5
Perfectly
20
Q

A term male infant of birth weight 2450 grams is born to a 27-year old woman who is G 3, P 2. The infant is noted to have muscular hypotonia. He receives his first feeding at 2 hours of age and again at 4 hours. Shortly afterwards he develops an enlarging abdomen and vomits bile-stained material. On further examination, he is noted to have a significant cardiac murmur. Which of the following chromosomal abnormalities is most likely to be present?

45,X
47,XY,+21
47,XX,+18
47,XXY

A

Correct answer: 47, XY, +21.

Explanation: The infant has Down Syndrome (DS), Trisomy 21 with a heart defect (most frequently an endocardial cushion defect), muscular hypotonia, and intestinal atresia. Infants with Trisomy 21 have an increased frequency of a number of gastrointestinal malformations including duodenal atresia, Hirschsprung’s disease and congenital megacolon, esophageal atresia, and anorectal malformations. The areas of intestinal atresia, seen in almost 10% of infants with Down syndrome, include the duodenum, the rest of the small bowel, and rarely the large bowel. Trisomy 21 has an incidence of 1 in 700 newborns. Trisomy 21 is due to nondisjunction in 95% of cases, with an additional 4% of cases due to translocation with 50% having a balanced translocation carrier parent, most frequently 21:14 (rarely 21q;21q translocation with risk of 100%). 1% of infants with DS are mosaics (mixture of 46 and 47 chromosomes). Risk of having an infant with Downsyndrome increases with increasing maternal age from 1:1000 infants at age 20 to 1:30 at age 45 years. Still, 75% of Down syndrome babies are born to women under 35 since that is the age at which the vast majority of mothers have children. Risk of recurrence in subsequent pregnancies is 1% but goes to 16% if the mother is a carrier of translocation between 21 and other chromosomes (5% if father is carrier). DS males are infertile, but DS females may reproduce with a 50% chance of DS occurring in the infant.

Other answers:

45,X is Turner syndrome. Characteristics of patients with Turner syndrome include short stature, webbed neck, and low-set ears.

47,XX,+18 is trisomy 18. This is due to presence of an extra whole or partial chromosome 18 and typically results in major congenital abnormalities in a number of organs, including the heart (especially ventrical and septal defects), kidneys, intestines (including omphalocele and esophageal atresia), central nervous sytem, and liver. While infants with trisomy 18 can present with the findings present in the infant described in the question stem, trisomy 18 is a less likely diagnosis because it is far less common at birth than Down syndrome. Whereas Down syndrome occurs in 1 per 700 newborns, trisomy 18 occurs in only about 1 per 6000 newborns. The majority (about 80%) of fetuses with trisomy 18 die in utero. Once born, the lifespan of a an infant with trisomy 18 is typically only 1-2 weeks, though a small percentage survive longer than one year

47,XXY is Klinefelter syndrome, in which males have an extra X chromosome. While patients with this karyotype are often asymptomatic, they can have a variety of clinical manifestations including hypogonadism and decreased fertility.

Robbins PBOD 8ed. pp 161-162

How well did you know this?
1
Not at all
2
3
4
5
Perfectly
21
Q

Sgt Rivera presents to you in clinic with persistent post-concussive symptoms including fatigue, irritability, memory problems, and sensitivity to noise following exposure to an IED blast approximately 6 months ago in theater. At the time of the injury, Sgt Rivera was reported to have experienced 15 minutes of loss of consciousness (LOC) and did not remember the hour following the blast until reaching the hospital for evaluation. 8 hours after the injury Glasgow Coma Scale score was 14 and a head CT was read as within normal limits (WNL). What would be your preliminary characterization of this injury event and recommendations?

Severe TBI. Begin physical therapy and suggest Medical Board Evaluation
Moderate TBI. Refer patient for neuropsychological assessment to characterize deficits and evaluate additional possible comorbid factors (e.g., PTSD, major depression).
Mild TBI. Refer patient for neuropsychological assessment to characterize deficits and evaluate additional possible comorbid factors (e.g., PTSD, major depression).
Mild TBI. Assess for psychological disorder and malingering

A

Correct answer: Mild TBI. Refer for further testing.

Rationale for answer: Mild TBI is defined as LOC less than or equal to 30 minutes and/or posttraumatic amnesia (PTA) less than or equal to 24 hours. Brain imaging is typically normal in mild TBI. GCS score within 48 hours of injury is generally 13 or greater with mild TBI, 9-12 with moderate TBI, and less than 9 with severe TBI.

While in most cases of mild TBI symptoms typically resolve within a few months of injury, the trajectory of healing can vary. It is important to also evaluate for other contributing factors, such as PTSD and depression, which can have significant impacts on cognition, sleep, and other aspects of functioning.

How well did you know this?
1
Not at all
2
3
4
5
Perfectly
22
Q

A rapid expansion of the intra-vascular volume by an infusion of 2 liters of isotonic saline will normally be compensated for by:

an increase in thirst
a decrease in angiotesin II production
a decrease in glomerular filtration rate
an increase in ADH secretion

A

Answer: a decrease in angiotensin II production.

Explanation: increased intravascular volume = increased GFR = decreased renin secretion by JG apparatus = decreased angiotensin I production = decreased substrate for the formation of angiotensin II = decreased angiotensin II production.

Incorrect answers:

The volume receptor mechanism would inhibit ADH secretion

A volume sensing mechanism would inhibit thirst

Increased vascular volume would increase GFR

How well did you know this?
1
Not at all
2
3
4
5
Perfectly
23
Q

Chromosome studies were conducted on a couple that has lost a baby soon after birth because of multiple abnormalities. The father was found to have a balanced reciprocal translocation involving chromosomes 4 and 11.
Given this, which one of the following statements is true?

All of their future babies will be abnormal.
The abnormalities in their baby were probably caused by polyploidy.
Balanced reciprocal translocations are usually harmless so it is very unlikely that this finding is relevant to their baby’s abnormalities.
It is very unlikely that this translocation will cause ill health in the father but there is a risk that he and his partner will have another abnormal baby.

A

The father has a balanced reciprocal translocation. Translocations occur through exchange of DNA between non-homologous chromosomes. The karytoype figure demonstrates the possible outcomes for future infants. Possibilities include normal karyotype, balanced karyotype, unbalanced karyotype with possibility for survival, and unbalanced karyotype not compatible with survival.

Chromosomes with translocations will pair through their homologous parts with each other and with the pair of normal homologous chromosomes leading to the formation of a quadrivalent (instead of two bivalents as in normal meiosis). During the first meiotic division they will segregate randomly to one of two cells. Note that chromosome combinations in the sperm that are shown on the figure only indicate spermatozoa that receive two chromosomes, whereas it is possible that some of spermatozoa may receive three chromosomes from the quadrivalent and some may receive only one.

Incorrect answers:

The answer that all of their future babies will be abnormal is incorrect. As shown on the diagram below at least half of the couple’s newborns will be normal (will have either a normal or balanced karyotype). Keep in mind that spermatozoa with some unbalanced karyotypes either do not survive or are not proficient in fertilization, and most embryos with trisomies and monosomies (e.g., the last two on the figure) are spontaneously aborted, often before the pregnancy is even recognized.

The abnormalities in their baby were probably not caused by polyploidy. Polyploidy is the increase in the number of complete chromosomal sets (i.e., chromosomes #1 through #22 and a sex chromosome). Balanced rearrangements may result in aneuploidies (e.g., one extra and one missing chromosome as shown on the figure) in the offspring, but they do not induce polyploidy.

Although balanced translocations are usually harmless to their carriers the random nature of segregation of the rearranged chromosomes and their corresponding homologous chromosomes in meiosis may lead to unbalanced karyotypes in the progeny.

How well did you know this?
1
Not at all
2
3
4
5
Perfectly
24
Q

A 5 year old girl has a history of recurrent bacterial infections. She is found to have an inherited condition in which a gene important in promoting the production of reactive oxygen radicals (O2.) is defective leading to poor bacterial killing. The diagnosis of her condition is:

Chediak-Higashi syndrome
Chronic granulomatous disease
Leukocyte adhesion deficiency type 1
Leukocyte adhesion deficiency type 2

A

Answer: Chronic granulomatous disease.

Chronic granulomatous disease involves a defect in genes encoding a component of phagocyte oxidase, important for generation of O2 radical (O2.). Since initial neutrophil defense is inadequate, a macrophage-rich chronic inflammatory reaction is recruited that tries to control the infection (granuloma). Chediak-Higashi syndrome involves decreased leukocyte function because of mutations affecting a protein involved in lysosomal membrane traffic. Leukocyte adhesion deficiency type 1 involves defective leukocyte adhesion because of mutations in the beta chain of CD11/CD18 integrins. Leukocyte adhesion deficiency type 2 involves defective leukocyte adhesion because of mutations in fucosyl transferase required for synthesis of sialylated oligosaccharide (ligand for selectins).

Robbins PBOD 8ed. pp 55-56.

How well did you know this?
1
Not at all
2
3
4
5
Perfectly
25
Q

A 17-year-old Airman who is scheduled to be deployed to Iraq presents to your office because of shortness of breath and wheezing on exertion, particularly during basketball season. He does not notice any shortness of breath with routine activity. There is no family history of asthma. On physical examination, he is in no respiratory distress. His lungs are clear, with no wheezing during either tidal breathing or forced expiration. His cardiac examination is normal. Baseline spirometry is normal. What is the BEST next diagnostic step?

Allergy testing
Exercise testing with postexercise spirometry
Overnight oximetry
Reassurance

A

The most likely diagnosis is exercise-induced asthma, and exercise testing with postexercise spirometry is the most appropriate diagnostic test. Exercise testing for exercise-induced asthma should attempt to mimic the activity that induces the shortness of breath. In addition to establishing the diagnosis of exercise-induced asthma, exercise testing can reveal exercise-induced laryngeal dysfunction that sometimes mimics exercise-induced asthma. All patients with significant disease should also be instructed to use prophylactic treatment 5 to 10 minutes before exercise, usually two puffs of a medium-acting inhaled beta2-agonist (albuterol). A better term for this condition is “exercise-induced bronchospasm,” because not all persons with the condition have asthma. The estimated prevalence of exercise-induced bronchospasm ranges from 7% to more than 20% in the general population.

           Exercise-induced bronchospasm probably results from changes in airway physiology triggered by the large volume of relatively cool, dry air inhaled during vigorous activity. Bronchodilation is the more common first event during exercise and lasts for 1 to 3 minutes after exercise. In patients with exercise-induced bronchospasm, the initial bronchodilation is followed by bronchoconstriction, which begins within 3 minutes, generally peaks within 10 to 15 minutes, and resolves by 60 minutes. Acute bronchoconstriction was previously followed by late-phase bronchoconstriction in some patients; however, the risk and severity of late-phase bronchoconstriction due to exercise-induced bronchospasm is decreased when compared with allergen-induced asthma.

Other answers: Although some patients with exercise-induced asthma may also have an allergic component, there is no clear correlation between the development of exercise-induced asthma and allergies. Patients with exercise-induced asthma may have equivocal results on methacholine challenge testing, making it a less helpful test than postexercise spirometry. Overnight oximetry would provide no information regarding the onset of bronchospasm during exercise.

How well did you know this?
1
Not at all
2
3
4
5
Perfectly
26
Q

A 5 year old boy develops an acute bacterial pneumonia. His CBC shows a marked elevation of neutrophils with a “shift to the left” (i.e. increase in immature band forms). Gram stain shows Gram positive cocci in pairs and short chains. The mechanism for this change involves:

Direct IL-1 and TNF-alpha stimulation of the bone marrow post-mitotic reserve pool.
Induction of G-CSF from macrophages through bacterial stimulation of Toll-like receptors
Acute release of C-reactive protein from the liver
LPS-induced release of IL-1 and TNF-alpha resulting in stimulation of PGE2 production

A

Answer: Direct IL-1 and TNF-alpha stimulation of the bone marrow post-mitotic reserve pool.

IL-1 and TNF-a are cytokines with largely overlapping biologic properties that play a major role in mediating acute inflammation. One effect is to act directly on the bone marrow post-mitotic reserve pool resulting in a rise in the number of immature neutrophils (bands) in addition to rapid release of mature neutrophils.

Other answers: LPS (lipopolysaccharide) is found in the membranes of Gram negative bacteria. PGE2 plays a role in induction of fever and stimulation of vasodilation and increase vascular permeability. It has no role in causing a left-shift.

C-reactive protein (CRP) is an acute phase reactant released by the liver in response to IL-6 production. CRP binds to phosphocholine on certain bacterial polysaccharide capsules and on dead and dying cells. Once bound to a cell or bacterial surface, CRP activates the complement system and functions as an opsonin to enhance clearance of dead cells and bacteria by macrophages.

G-CSF stimulates bone marrow synthesis of mature neutrophils.

Robbins PBOD 8ed. p75.

How well did you know this?
1
Not at all
2
3
4
5
Perfectly
27
Q

A 46 year old man complains of gradually developing numbness on his left hand over a two-week period. You have him remove his watch, which is relatively tight. Your examination shows that the motor examination is normal and that the numbness involves the skin on the dorsum of his hand between the thumb and the index metacarpal bones. This area is innervated by the:

lateral antebrachial cutaneous nerve
superficial branch of the radial nerve
dorsal branch of the ulnar nerve
median nerve

A

The radial nerve has a specific sensory distribution that is consistent and that covers the dorsoradial aspect of the hand. These sensory fibers are transmitted via the superficial branch of the radial nerve (the deep branch transmits primarily motor impulses), with little if any “cross-over” from the other nerves distal to the wrist.

How well did you know this?
1
Not at all
2
3
4
5
Perfectly
28
Q

Because of a high prevalence of hepatitis C virus (HCV) infection (10–20%) among veterans seeking care in Department of Veterans Affairs hospitals, current US military forces were evaluated for HCV infection. Banked serum samples were randomly selected from 10,000 active-duty military personnel and were tested for antibody to HCV (anti-HCV). 42 samples were initially positive. Over 5 years of follow-up, 6 subjects who were initially negative for anti-HCV seroconverted. What is the 5-year incidence of a positive test result for HCV infection in active-duty military personnel?

Choices
 48/9,958
 6/9,958
 6/10,000
 42/10,000
A

Incidence is number of new cases divided by population at risk of disease. There were 6 new cases during the follow-up period. Although the entire population consists of 10,000 personnel, 42 were initially positive and thus could not become a new case, so the population at risk is 10,000 – 42 = 9,958. Incidence is 6/9,958 over the 5 year period.

How well did you know this?
1
Not at all
2
3
4
5
Perfectly
29
Q

An 18-year-old college freshman is brought to a local emergency room with a fever of 103.4°, a few petechiae on her trunk, and, on examination, positive Kernig and Brudzinski’s signs. A sample of her cerebrospinal fluid is obtained and Gram stain reveals a few Gram-negative cocci. The major component that allows this organism to resist complement-mediated phagocytosis is MOST LIKELY

lipooligosaccharide
a serotype of M protein
a polysaccharide capsule
an IgA protease

A

The correct answer is a polysaccharide capsule.

The disease described is most likely meningococcal meningitis. A primary virulence factor of Neisseria meningitidis is its anti-phagocytic polysaccharide capsule. Protection against common types of N. meningitidis is provided through vaccination of young people with a polyvalent vaccine that consists of several of these capsular antigens.

Other answers: The M protein is a helical protein component of the cell wall of Group A streptococci that has an antiphagocytic property as well. Several organisms produce IgA proteases that have activity against that class of secretory immunoglobulins, however IgA is a poor activator of complement and opsonizes very weakly. While N. meningitidis expresses lipooligosaccharide (LOS), this is not a mechanism by which the bacteria resists complement-mediated phagocytosis. Rather, LOS contributes to clinical disease by binding to toll-like receptor 4 on innate immune cells, resulting in the release of massive amounts of pro-inflammatory cytokines.

How well did you know this?
1
Not at all
2
3
4
5
Perfectly
30
Q

A Finnish woman is the mother of a child with Meckel syndrome, a rare autosomal recessive disorder occurring in 1/10000 births in her part of Finland. She is widowed and is now going to remarry. What is the increased risk of the disease in a fetus if she marries the brother of her deceased husband rather than an unrelated Finnish man from her geographic area?

¼ versus 1/200
1/8 versus 1/10000
1/8 versus 1/200
100% versus 50%

A

Answer: 1/8 versus 1/200

If the child is affected by autosomal recessive disease he must be a homozygote. Therefore, his parents are obligate heterozygotes. Either the deceased husband’s mother or father is also a heterozygote, because their son must have inherited the mutant gene from one of them. Then the chance that the deceased husband’s brother is a carrier is 1/2 (he can inherit either a normal or a mutant allele from the heterozygous parent). If the mother and the deceased husband’s brother are heterozygotes the chance of the fetus inheriting both mutant alleles is 1/4 so the total risk is 1/2 X 1/4 = 1/8.

The mother is an obligate heterozygote. The disease allele frequency in this population is √q = √ 1/10000 = 1/100. Normal allele frequency is 1–1/100 ~ 1. Carrier (heterozygote) frequency is 2pq = 2 x 1 x 1/100 = 1/50. This is the risk of a random person being a carrier. If the mother and a random Finnish male are both carriers for autosomal recessive disease the chance of their child being affected is 1/4. Total risk for the fetus is 1/4 x 1/50 = 1/200.

How well did you know this?
1
Not at all
2
3
4
5
Perfectly
31
Q

Your patient is a 46-year-old male who tells you that he is having difficulty with his vision. He also tells you that his breasts have recently enlarged and started to lactate. What would be the most appropriate drug to treat this patient?

pegvisomant
testosterone
bromocriptine
sermorelin

A

The correct answer is bromocriptine.

The patient’s symptoms, breast enlargement with lactation and loss of vision, are compatible with hyperprolactinemia resulting from an anterior pituitary prolactin-secreting adenoma. A blood test for serum prolactin should be performed to confirm your diagnosis. A long-acting dopamine agonist, like bromocriptine, will inhibit prolactin secretion and cause the adenoma to shrink. None of the other drugs [testosterone, pegvisomant (a GH antagonist), or sermorelin (GHRH1-29)] would affect prolactin levels or the adenoma. The patient may eventually want to have the adenoma surgically removed, but it would be appropriate to shrink the adenoma before proceeding with the surgery.

How well did you know this?
1
Not at all
2
3
4
5
Perfectly
32
Q

A cheese-linked outbreak of bacteremia and meningitis in the elderly and stillbirths in pregnant women was revealed through the diligent work of a nurse epidemiologist. The cheese was made from raw milk and stored for weeks in the cold. The MOST LIKELY cause of this outbreak was

Brucella abortus
Listeria monocytogenes
Mycobacterium bovis
Escherichia coli O157:H7

A

The correct answer is Listeria monocytogenes.

Although all of these agents may be found in unpasteurized milk, Listeria monocytogenes is capable of actually replicating at refrigerator temperatures and hence is often linked to refrigerated meats and dairy products. Generally speaking, healthy people develop only mild self-limiting flu-like or gastrointestinal symptoms from foods contaminated with Listeria, but individuals that lack good cell mediated immunity are at risk from this facultatively intracellular pathogen. Unlike the other agents listed, Listeria has the capacity to cross the placenta in pregnant patients and cause stillbirth or meningitis in the newborn. Meningitis is also seen in AIDS patients or the elderly infected with listeria. Brucella causes a recurrent febrile syndrome, E. coli causes hemorrhagic colitis and hemolytic uremic syndrome, and M. bovis is associated with miliary TB, especially where unpasteurized milk is a major food source.

How well did you know this?
1
Not at all
2
3
4
5
Perfectly
33
Q

Willamena Whiner is a 40-year-old female who sprained her wrist playing golf. She tells her doctor that she has been taking aspirin for the pain, but her wrist still hurts. Her doctor prescribes a mild opiate that she is to use in combination with the aspirin. Now, she returns to her doctor claiming that the prescribed opiate is completely ineffective. In a follow-up test, it is discovered that she is deficient in active CYP2D6. Which opiate had her doctor MOST LIKELY prescribed that was ineffective in this patient?

fentanyl
codeine
loperamide
morphine

A

The correct answer is codeine.

Explanation: In order to be an effective analgesic, codeine must be converted to morphine by CYP2D6. In patients deficient in CYP2D6, codeine is ineffective as an analgesic.

Incorrect answers: Morphine and fentanyl are full mu opiate agonists that need no modification for activity. Loperamide lacks any analgesic property because it is rapidly pumped out of the CNS by the MDR transporter. Loperamide is an opiate that is used as an anti-diarrheal agent.

How well did you know this?
1
Not at all
2
3
4
5
Perfectly
34
Q

A 50 year old male with chronic diverticulitis eventually developed a fibrotic stricture in his colon resulting in gastrointestinal obstruction. Which of the following was the most likely sequence of events leading to this outcome?

Activation of macrophages and lymphocytes, production of TGF-b and PDGF, and decrease in metalloproteinase activity.
Activation of mast cells, secretion of histamine, increased vascular permeability
Secretion of IL-5 by Th2 cells, recruitment of eosinophils, release of major basic protein
Secretion of IFN-γ by Th1 cells, macrophage activation, focal collections of epithelioid cells

A

Answer: Activation of macrophages and lymphocytes, production of TGF-b and PDGF, and decrease in metalloproteinase activity.

Explanation: Chronic inflammation as observed in chronic diverticulitis results in tissue fibrosis. Activated macrophages and lymphocytes release a variety of mediators, including PDGF and TGF-β, which play a role in promoting fibroblast proliferation and collage synthesis as well as inhibiting metalloproteinase synthesis (with resultant decreased collagen degradation).

Other answers: Activation of mast cells, secretion of histamine, and increased vascular permeability are observed in type 1 hypersensitivity reactions. Secretion of IFN-γ by Th1 cells, macrophage activation, and focal collections of epithelioid cells are involved in granulomatous inflammation. Secretion of IL-5 by Th2 cells, recruitment of eosinophils, and release of major basic protein are observed in allergic reactions and in response to helminth infections.

Robbins PBOD 8ed. pp 107-108

How well did you know this?
1
Not at all
2
3
4
5
Perfectly
35
Q

A patient with severe Chronic Obstructive Pulmonary Disease (emphysema) presents to the Emergency Department because of increasing shortness of breath due to an acute respiratory infection. You know the patient and are aware that her arterial PaCO2 is usually about 60 mmHg. Arterial blood gases obtained in the Emergency Department show a PaO2 of 30mm Hg, a PaCO2 of 70 mm Hg, and an arterial pH of 7.15. You administer a high concentration of inspired oxygen by face mask. Repeat arterial blood gases, obtained with the patient on the inspired oxygen, reveal a PaO2 of 40mm Hg, a PaCO2 of 90 mm Hg, and an arterial pH of 6.90. Which of the following choices would be the most appropriate at this time?

Have the patient inhale CO2 to stimulate breathing.
Have the patient breathe 100% oxygen.
Place the patient on a ventilator and adjust the settings to achieve adequate arterial PO2 and PCO2.
Return the patient to breathing room air.

A

Ans: Place the patient on a ventilator and adjust the settings to achieve adequate arterial PO2 and PCO2.

The patient is suffering from an inability to provide sufficient exchange of CO2 and oxygen. Administering increased oxygen signals to the respiratory centers to slow down the ventilatory rate. However, because of her COPD, she needs to breathe more rapidly to exchange CO2. Increasing CO2 does not stimulate increased breathing because the patient has been at elevated CO2 levels for so long that her receptors have accommodated to the increased CO2. Therefore, mechanical intervention is required to get her balance of ventilation rate and volume to the point that her PaCO2 will be reduce to acceptable levels.

How well did you know this?
1
Not at all
2
3
4
5
Perfectly
36
Q

The development of protective antibodies specific for which ONE of the following bacterial surface components is often protective and is also the basis of several successful vaccines against bacterial pathogens?

Capsule
Multidrug resistance efflux pumps
Lipopolysaccharide
Colonization pili

A

The correct answer Capsule.

Explanation: Capsular polysaccharides are the basis for a number of vaccines including Stretococcus pneumoniae, Haemophilus influenzae and Neisseria meningitidis. To develop an anamnestic immune response, these polysaccharides are often conjugated to proteins to stimulate a T-cell response. Protection against colonization with pili is a potential strategy but thus far there are no such vaccines available. The toxic nature of lipopolysaccharide rules it out as a potential immunogen, although it is easy to envision that anti-LPS antibody would be useful in blocking shock associated with Gram-negative infections. Antibodies against drug resistance pumps have not been used as a vaccine strategy.

How well did you know this?
1
Not at all
2
3
4
5
Perfectly
37
Q

A Marine normally stationed at Camp Pendleton, CA is deployed to Afghanistan and helicoptered to a forward operating base situated at 2500m. He is uncomfortable with headache and nausea but the next day is feeling well enough to perform his duties. Which of the following changes accounts for his accommodation to the altitude of his new base?

Increased red cell production
Central chemoreceptor adaptation
Hemoglobin isoform alteration
Angiogenesis

A

Ans: Central chemoreceptor adaptation.

Hypoxemia is the cause of the observed symptoms. Lowered levels of oxygen will cause the peripheral chemoreceptors to increase the rate of breathing in an attempt get more oxygen into the system. This will cause a decrease in PaCO2 which will cause the central chemoreceptors to slow the breathing rate, again decreasing PaO2 . Breathing rate must increase to maintain adequate PaO2. This back and forth feedback continues until the central chemoreceptor adapt to lower PaCO2 levels at the higher respiratory rates. One would expect that red cell production will be stimulated but the differentiation of erythrocytes will take about a week before the increased numbers will show up in the bloodstream.

How well did you know this?
1
Not at all
2
3
4
5
Perfectly
38
Q

An 11-month old male infant is brought to his pediatrician because he seems to be much “slower” in doing things than his siblings. He has just learned to roll over but is unable to sit up. He is noted to have a large head and mildly low set ears that are posteriorly rotated. His sternum is somewhat concave and his fingers show hyperextensibility. He has an older brother with a prominent mandible who is doing poorly in school. Which of the following types of gene mutations is most likely to produce these findings?

Frameshift
Nonsense (stop codon)
Trinucleotide repeat
Point

A

Correct answer: Trinucleotide repeat.

Explanation: These findings are typical of fragile X syndrome, a condition in which there are 250-4000 tandem repeats of the trinucleotide sequence CGG. In normal individuals there are between 10-50 CGG repeats but in families with this disorder the number of repeats tends to increase with succeeding generations (“genetic anticipation”). The gene is on chromosome X at q27.3. Other diseases with unstable trinucleotide repeats of C_G include Myotonic Dystrophy – CTG, Huntington Disease – CAG, and Kennedy’s Disease – CAG. In Fragile X Syndrome, the number of CGG repeats affects the phenotype with 6-60 repeats (normal allele) producing a normal phenotype, 60-200 repeats (premutation allele) also producing a normal phenotype but one who is an obligate carrier, and >200 repeats (full mutation allele) leading to various expression of the Fragile X phenotype.

Robbins PBOD 8ed. pp 169-171

How well did you know this?
1
Not at all
2
3
4
5
Perfectly
39
Q

A 55 year old woman complains of blurred vision and burning and itching of her eyes. She also has difficulty swallowing solid foods and reports a decrease in the ability to taste. On physical exam she has conjunctival erythema and cracks and fissures in the mouth. Serum anti-SS-A antibody is elevated. She has no other associated autoimmune diseases. Which of the following histological changes is most likely to be seen in a salivary gland biopsy?

granulomatous inflammation
periductal lymphocytes
eosinophilic infiltrates and interstitial edema
metaplastic changes in acinar epithelium

A

Correct answer: Periductal and perivascular lymphocytes and germinal centers.

Explanation: This is a case of Sjogren’s syndrome, an autoimmune exocrinopathy. In this condition CD4+ T cells reacting to exocrine gland self-antigens are believed to stimulate ductal epithelial cell hyperplasia, with resulting duct obstruction and acinar cell atrophy and fibrosis. This leads to reduced production of exocrine gland secretions, accounting for many of the signs and symptoms of this disease, including dry eyes and dry mouth. Most of the lymphocytes infiltrating the salivary gland are CD4+ T-cells, and 10% are B cells.

Other answers: Eosinophils are typically seen in allergic conditions and infections with nematodes. Some autoimmune diseases exhibit granulomatous inflammation (e.g. Wegener granulomatosis, temporal arteritis, etc.), but not Sjogren’s syndrome. Although acinar epithelial cells are injured in Sjogren’s syndrome, they do not undergo metaplasia.

Robbins PBOD 8ed. pp 221-223.

How well did you know this?
1
Not at all
2
3
4
5
Perfectly
40
Q

A 44-year-old patient develops fevers while recovering from surgery to repair a bowel perforation that developed secondary to a complicated appendicitis. The microbiology lab calls and informs you that the blood cultures are growing a germ-tube positive yeast. The most appropriate antibiotic to treat this organism initially is:

ceftazidime
fluconazole
ampicillin/sulbactam
amphotericin B

A

The correct answer Fluconazole.

Explanation: The patient has candidemia (Candida infection of the blood). The only two antifungal agents listed are amphotericin B and fluconazole. Clinical studies in immunocompetent patients have demonstrated that fluconazole and amphotericin B are equally effective at treating bloodstream infections with Candida albicans. Of the two agents, amphotericin B has a substantially worse side effect profile, with significant risk of renal toxicity. Having ascertained by the germ-tube test that the yeast is most likely Candida albicans (which, in addition to the rare Candida dublinensis is the only Candida species which is germ tube positive), fluconazole would be a more appropriate choice of drug than amphotericin B in this situation. An echinocandin would also be a reasonable choice. Ampicillin/sulbactam and ceftazidime target bacterial cell wall synthesis and therefore would not be useful against this yeast infection.

If the yeast was germ tube negative, one would be concerned that the patient could possibly be infected with a Candida species that is resistant to fluconazole, such as Candida glabrata or Candida krusei. Thus, in the setting of a germ-tube negative yeast, one would not start with fluconazole. Rather, one would use an echinocandin or amphotericin B. Additionally, because of the widespread use of azoles in the prophylaxis of cancer patients that become neutropenic, patients with neutropenia who develop a Candidemia are frequently started on an echinocandin or amphotericin for initial coverage.

How well did you know this?
1
Not at all
2
3
4
5
Perfectly
41
Q

An 82 year old retired Army Colonel with a history of diabetes, hypertension and obesity presents to the Emergency Department at 4 am with severe pain in his right ankle which woke him from sleep. He states that even having the bedsheet rest on his ankle elicits pain. He has an oral temperature of 101F. The affected ankle appears erythematous. Even slight movement produces excruciating pain. An arthrocentesis yields a small amount of amber fluid which microscopically reveals numerous PMN’s and positively birefringent crystals. Which of the following diagnoses is BEST supported by the findings?

Gout
Intraarticular fracture of the MTP joint
Septic arthritis
Pseudogout

A

Answer: pseudogout

The presentation is suggestive of either a septic joint or crystal-induced synovitis, with severe pain of rapid onset and low-grade fever. This presentation is not really compatible with a fracture (no history of trauma). The appropriate diagnostic test was the joint fluid aspiration, which demonstrated crystals. Uric acid crystals are needle-shaped on polarized microscopy and are negatively birefringent. Calcium pyrophosphate crystals, in contrast, are blue-yellow birefringent. Because the aspiration revealed positively birefringent crystals, the correct answer is pseudogout.

How well did you know this?
1
Not at all
2
3
4
5
Perfectly
42
Q

Which one of the following can infect the joints via hematogenous spread from the reproductive tract?

Chlamydia trachomatis, serovars L1, L2, L2a and L3
Neisseria gonorrhoeae
Haemophilus ducreyi
Chlamydia trachomatis, serovars D-K

A

The correct answer is Neisseria gonorrhoeae

Although all these agents are pathogenic in the reproductive tract, the one associated with hematogenous spread, joint infection, and disseminated gonorrheal infection (DGI) is Neisseria gonorrhoeae. In fact, the leading cause of septic arthritis in people of reproductive age is Neisseria gonorrhoeae. It may be difficult to confirm the diagnosis by culturing synovial fluid or the skin lesions that accompany such an infection. Culture of the urethra, cervix, rectum or pharynx is often positive and would confirm the diagnosis.

Chlamydia trachomatis, serovars D-K, cause urethritis, cervicitis, pelvic inflammatory disease, and neonatal pneumonia and conjunctivitis. Chlamydia trachomatis, serovars L1, L2, L2a and L3 cause lymphogranuloma venereum (LGV), which can cause a painless genital ulceration, inguinal lymphadentis/lymphangitis, and proctitis. Haemophilus ducreyi causes painful genital ulcerations.

How well did you know this?
1
Not at all
2
3
4
5
Perfectly
43
Q

Which of the following structures disappears just before the start of implantation?

Zona pellucida
Cilia on the endometrial epithelium
Allantois
Sperm acrosome

A

Explanation: The zona pellucida is a glycoprotein shell that surrounds the ovum, acts as receptors for sperm and induces the acrosome reaction. At the end of the fourth day the zona pellucida disappears and the conceptus implants into the uterine lining at ~day 6. The conceptus would not be able to implant if it was still surrounded by the zona pellucida.

Incorrect answers:

The allantois forms after implantation.

Cilia on the endometrial epithelium do not disappear.

The sperm acrosome disappears during the process of fertilization, an event that occurs ~6 days before implantation.

How well did you know this?
1
Not at all
2
3
4
5
Perfectly
44
Q

A three year old girl presents with a lesion at the lower palpebral conjunctiva that has been present for two weeks . Periauricular nodes are enlarged. A large mass in the left axilla shows reddening of the skin over the mass. Her six year old brother is noted to have enlarged right inguinal lymph nodes and a lesion on his ankle. The regional lymphadenopathy occurred along with fever, malaise, headache and bone and joint pain. The axillary and inguinal lymph nodes are enlarged, matted, nodular and adherent to surrounding tissue. Histology of an axillary lymph node displays confluent necrotizing granulomas. The children live with their family on a farm and are responsible for care of the chickens, three dogs and two kittens. To identify the organism causing this disease, a biopsy of the tissue must be prepared using which procedure:

Culture the tissue on blood agar in 100% oxygen
Stain the sections with a connective tissue stain (e.g. trichrome stain)
A touch preparation of the tissue stained with Wright’s stain
Culture the tissue in heart-brain infusion broth at 32⁰C

A

ANSWER: Culture the tissue in heart-brain infusion broth at 32⁰C .

Explanation: The lymphadenopathy with the necrotizing granulomas is typical of cat scratch disease (CSD), typically seen in children, but also in adults, who have been in contact with cats, usually kittens. The diagnosis is usually made by positive serology in combination with certain clinical criteria*. To identify the organism, however, the organism typically needs to be grown in culture.

The organism causing CSD (Bartonella henselae) is a fastidious bacillus that grows best in heart-brain infusion broth at 32⁰C - 35⁰C. The typical culture medium (blood agar) used to culture most bacterial and fungal organisms has a low yield for identifying Bartonella henselae. Stains such as H& E, trichrome, gram and Wright will not stain the organism. Silver stains such as Warthin-Starry will show clusters of the bacilli in area of necrosis within the necrotizing granulomas. PCR identification of B. henselae is also useful, but not that sensitive (thus negative PCR does not rule out the diagnosis). It is important to recognize that special stains and culture approaches are needed when considering the diagnosis of CSD, so that the appropriate tests can be ordered on biopsy samples.

  • Clinical criteria supportive of a diagnosis of cat scratch disease include:
    - -> cat or flea contact of any kind
    - -> negative serology for other causes of adenopathy OR + Bartonella PCR OR + liver and spleen lesions on CT OR sterile pus from lymph node aspiration
    - -> biopsy with granulomatous inflammation OR a positive Warthin-Starry silver stain
How well did you know this?
1
Not at all
2
3
4
5
Perfectly
45
Q

A 60-year-old female has recently been diagnosed with Parkinson’s disease. Levodopa has been effective in relieving her rigidity and akinesia, but she is now experiencing severe emesis and orthostatic hypotension. What drug, acting only in peripheral tissues, would be the MOST appropriate addition to her therapy to decrease these side effects of levodopa?

rasagiline
carbidopa
benztropine
pramipexole

A

The correct answer is carbidopa.

Emesis and orthostatic hypotension result from the peripheral actions of dopamine. Carbidopa inhibits the decarboxylation of l-dopa to dopamine, but since carbidopa does not enter the brain, it prevents the conversion of l-dopa to dopamine only in peripheral tissue.

None of the other drugs listed would prevent the peripheral side effects associated with l-dopa administration.

Benztropine is an anticholinergic agent that can be used as a 2nd line agent for Parkinson’s disease to reduce tremor. However, it would not reverse the peripheral effects of l-dopa administration.

Pramiprexole is a dopamine agonist that is sometimes used in early treatment of Parkinson’s disease. Like levodopa, its side effects include nausea, vomiting, and orthostatic hypotension.

Rasagiline is an irreversible monoamine oxidase inhibitor that can be used in early Parkinson’s or as an additional agent in late Parkinson’s disease. It would not reverse the peripheral effects of l-dopa administration.

How well did you know this?
1
Not at all
2
3
4
5
Perfectly
46
Q

A 4 year old child presents to her physician for eye discomfort. Physical examination reveals dry eyes with corneal softening. Which of the following vitamin deficiencies is the most likely cause for this presentation?

Folic acid
Vitamin A
Vitamin B12
Vitamin C

A

The correct answer is Vitamin A.

Explanation: Vitamin A is needed for epithelial maturation. Absence of vitamin A leads to squamous metaplasia of the lacrimal ducts leading to dry eyes and corneal softening (keratomalacia). Other symptoms of vitamin A deficiency include night blindness, corneal ulceration, pruritis, and growth retardation.

Other answers: B12 and folic acid deficiencies are associated with megaloblastic anemia. Additionally, B12 deficiency can cause decreased vibratory sensation. Vitamin C deficiency leads to scurvy, which is characterized by vascular fragility and bone changes. Some other notable vitamin deficiencies include Vitamin D deficiency (which causes rickets), Vitamin K deficiency (which causes hemorrhagic diathesis), and Niacin deficiency (which causes pellagra – diarrhea, dementia and dermatitis).

Robbins PBOD 8ed pp 430-431

How well did you know this?
1
Not at all
2
3
4
5
Perfectly
47
Q

A 70-year-old male was hospitalized for surgical repair of a femur fracture. On hospital day six he developed fever to 40°C, productive cough and hypoxia. Chest X-ray confirmed a right upper lobe pneumonia, and sputum Gram stain showed numerous neutrophils and gram-negative rods. Intravenous ceftazadime (a third generation cephalosporin) therapy was initiated, but the patient failed to improve and developed respiratory failure. Sputum culture, taken at the onset of his pneumonia, yielded Enterobacter cloacae resistant to all of the extended spectrum penicillins and cephalosporins. What is the MOST LIKELY mechanism of resistance manifested by this organism to these antibiotics?

An altered penicillin binding protein
A DNA-gyrase (topoisomorase) mutation
An extended spectrum beta-lactamase
Altered permeability of the bacterial membrane

A

The correct answer An extended spectrum beta-lactamase.

Explanation: It is not unusual, especially in the hospital setting, to encounter members of the family Enterobacteriaceae that have acquired the genes to encode an extended spectrum beta-lactamase (ESBL). This enzyme is capable of inactivating numerous beta-lactam antibiotics of the penicillin and Cephalosporin families. In addition Enterobacter cloacae, other Enterobacteriaceae that often express ESBLs include Escherichia coli, Klebsiella pneumonia, Klebsiella oxytoca, Salmonella species, Proteus mirabilis.

Pseudomonas aeruginosa, which is not a member of the Enterobacteriaceae family, is another common nosocomial gram negative organism that can also express an ESBL.

How well did you know this?
1
Not at all
2
3
4
5
Perfectly
48
Q

A 7 year old boy is brought to his physician for difficulty concentrating at school. A complete blood count performed as part of his workup demonstrates a microcytic anemia, and a peripheral smear of his blood shows basophilic stippling of the red blood cells. An assay for zinc protoporphyrin shows marked elevation. Exposure to which of the following substances is the most likely cause for his presentation?

Mercury
Cadmium
Arsenic
Lead

A

The correct answer is Lead toxicity.

Lead toxicity is associated with CNS injury leading to intellectual and psychologic impairment in children. Lead interferes with enzymes important in heme synthesis, including aminolevulinic dehydratase, aminolevulinic acid synthetase, and ferrochelatasae. Consequently, lead intoxication can result in decreased iron incorporation into heme, causing a microcytic anemia and increased levels of zinc protoporphyrin in the blood. Basophilic stippling is often seen in anemia due to lead poisoning. This phenotype is believed to be due to lead inhibition of a pyrimidine 5’ nucleotidase, which results in increased degradation of ribosomal RNA. Other features of lead toxicity include abdominal colic, growth retardation, neuropathy and nephrotoxicity. Lead lines may be observed in the gums on physical examination. These typically appear as a blue line along the gums, with a blue/black coloring of the gum tissue adjacent to teeth. Deposition of lead in the bones of children can often be observed as a transverse hyperdense line in the metaphysis of long bones.

Other answers: Cadmium toxicity is associated with obstructive lung disease and renal tubular injury.

Arsenic toxicity is associated with gastrointestinal, cardiovascular, and CNS dysfunction. Transverse white lines in the nails (Mees lines) are seen with arsenic exposure. There is also an increase in skin and lung cancer from chronic exposure.

Mercury toxicity is associated with diarrhea, visual field defects and nephrotoxicity.

Robbins PBOD 8ed pp406-407

How well did you know this?
1
Not at all
2
3
4
5
Perfectly
49
Q

The mother of a 2 month old female infant noticed a palpable mass in the upper portion of the abdomen along with numerous blue/black slightly raised spots on the skin of her trunk and face. The infant displayed mild respiratory distress. A CT scan revealed a markedly enlarged liver and a mass in the left upper portion of the abdomen. Laboratory studies showed that the 24-hour urine levels of homovanillic acid (HVA) and vanillylmandelic acid (VMA) were increased. A bone marrow aspirate displayed numerous clusters of “small round blue cells.” Bone scans were unremarkable for any bony destruction. Which of the following should be evaluated to assist in therapeutic decision making?

Alpha Fetoprotein (AFP) staining of the cells in the liver
Ultrasound to check for malformations of the kidney
Assessing for blastemal cells in the biopsy of the mass in the left upper abdomen
Presence of N-MYC amplification

A

ANSWER: Presence of N-MYC amplification.

Explanation: The patient has a neuroblastoma, a tumor that arises from primitive sympathetic ganglion cells. These tumors can arise anywhere within the sympathetic nervous system, including the adrenal glands, abdomen, and thoracic cavity. About 2/3 of neuroblastomas originate in the abdomen (and 2/3 of those are from an adrenal gland). Neuroblastoma cells typically have defective catecholamine synthesis, and thus release increased amounts of catecholamine intermediates such as HVA, VMA, and dopamine. There are many types of neuroblastomas, and they can cause a wide variety of clinical diseases (from benign, mature tumors, to aggressive and metastatic ones). Neuroblastomas can metastasize by both lymphatic and hematogenous routes. Tumors can spread to the skin, where they typically appear as firm, non-tender, bluish nodules.

The infant in the clinical stem most likely has a neuroblastoma with involvement of the left adrenal gland, the liver, the skin and the bone marrow (but not cortical bone). N-MYC (or MYCN) is an oncogene that is an important prognostic feature of neuroblastomas. Presence of the N-MYC oncogene portends a poor prognosis, while absence of N-MYC amplification is associated with a more favorable prognosis. Of note, the child in this question stem most likely had a 4S tumor. While metastatic neuroblastomas typically have a very poor prognosis, children under 1 year of age who have metastatic neuroblastoma without spread to cortical bone typically do quite well with frequent spontaneous regression of the tumors. In these special 4S cases, when the skin lesions regress a biopsy will show only mature ganglion cells. The liver and bone marrow involvement will also disappear.

Other answers: Malformations of the kidney are not associated with neuroblastoma. Blastemal cells are a component of nephroblastoma (Wilms tumor). AFP staining of hepatocytes is associated with hepatoblastoma.

Robbins PBOD 8ed. pp 475-479

How well did you know this?
1
Not at all
2
3
4
5
Perfectly
50
Q

20-year-old U.S. solider is brought to a clinic in El Paso with severe, non-bloody diarrhea and dehydration. No leukocytes are seen in stained fecal smears. Incubation of stool from the patient results in a pure culture of Gram-negative bacilli that ferment lactose. Bacteria are sent to a health department for further identification. The bacteria responsible for his illness are MOST LIKELY to produce which one of the following virulence factors?

Shiga toxin (ST)
Cholera toxin (CT)
Heat labile toxin (LT)
Cytolethal distending toxin

A

The correct answer is Heat labile toxin.

By the description of the agent we know that it is a Gram-negative organism that ferments lactose. That distinguishes E. coli (which is lactose fermenting) from non-lactose fermenters such as Vibrio (which makes cholera toxin) and Campylobacter (which produces cytolethal distending toxin). Heat labile toxin and Shiga toxin are both associated with E. coli, which is lactose fermenting. However Shiga toxin is associated with bloody diarrhea and the Hemolytic Uremic Syndrome. The description in this case is of a secretory diarrhea which is the classic ETEC, or traveler’s diarrhea picture associated with heat labile toxin, an analogue of Cholera toxin.

How well did you know this?
1
Not at all
2
3
4
5
Perfectly
51
Q

A 23-year old woman gave birth to an 1100 gram infant at 31 weeks gestation. The newborn male had APGAR scores of 4 and 6 at one and five minutes and shortly afterwards developed increasing respiratory distress for which therapy with oxygen and high frequency ventilation was initiated. He was weaned from the oxygen and mechanical ventilation over 8 days and began gaining weight. He was sent home at one month of age. Which of the following factors was most likely responsible for this infant’s respiratory disease?

Congenital lobar emphysema
Congenital pulmonary airway malformation
Surfactant deficiency
Tetralogy of Fallot

A

ANSWER: Surfactant deficiency.

Explanation: Prematurity (birth weight under 2500 gms) is frequently complicated by respiratory distress (also called hyaline membrane disease) if the infant is less than 35-36 weeks gestation. Prior to that gestational age the lung has not developed a sufficient number of Type 2 alveolar cells to produce an adequate amount of surfactant to prevent the infant’s lungs from collapsing when exhaling. In fact, any event that may lead to the destruction of Type 2 alveolar lining cells (e.g. anoxia during delivery, congenital infection) can lead to surfactant deficiency even in term infants.

Surfactant is a mixture of lipids (about 90%, mostly phosphatidylcholine), proteins, and glycoproteins. Surfactant decreases the surface tension within alveoli, which decreases the pressure required to expand them. Decreased surfactant production increases alveolar collapse. In alveolar cells, surfactant is stored within specialized granules called lamellar bodies. As fetal lungs mature, the ratio of lecithin to sphingomyelin (both glyoproteins in surfactant) increases. Tests on amniotic fluid can be helpful in predicting degree of pulmonary immaturity before birth. These include measuring the lecithin-sphingomyelin ratio, fluorescence polarization, and lamellar body counts. The recent development of synthetic surfactant has allowed treatment of pulmonary immaturity immediately after birth.

Other answers:

Congenital pulmonary airway malformation, also known as congenital cystic adenomatoid malformation, is the most common primary congenital abnormality of the lung. CPAMs occur as focal growths in the developing lung that can have cystic and adenomatous components. Large lesions can inhibit development and growth of normal alveolar tissue in the lung by compression. In contrast to surfactant deficiency, these do not typically improve as a baby gets older. Thus, symptomatic CPAMS at birth are frequently treated by surgical resection.

Congenital lobar emphysema is a rare congenital abnormality of the lungs that presents with hyperinflation and air trapping. Chest x-ray shows distention of one lung and compression of the contralateral one. As with CPAMs, congenital lobar emphysema does not typically resolve on its own. Asymptomatic cases can be followed. Treatment for symptomatic congenital lobar emphysema usually involves surgical resection of the affected lobe.

Tetralogy of Fallot (TOF) includes RV outflow tract obstruction, ventricular septal defect, overriding aorta to the right (such that the origin of the aorta overrides the VSD), and RV hypertrophy. Infants presenting with respiratory distress due to TOF would not be expected to improve substantially without surgical correction.

Robbins PBOD 8ed. pp Robbins 456-458

How well did you know this?
1
Not at all
2
3
4
5
Perfectly
52
Q

A 57 year old male presents with crushing chest pain, dyspnea, and diaphoresis of one hour duration. He is brought to the emergency department by his family where EKG reveals marked ST elevations in the lateral leads. If the patient dies 48 hours after the onset of the above event, which of the following would be the most classic histologic finding in the involved area of the heart?

Early granulation tissue formation and edema
Coagulative necrosis and neutrophilic infiltrate
Edema and hemorrhage
Wavy fibers throughout the infarcted area
Late granulation tissue and collagen formation
Phagocytosis and macrophages at the border

A

The correct answer is Coagulative necrosis and neutrophilic infiltrate.

Explanation: The patient is presenting with classic symptoms and EKG findings of a myocardial infarction. Histopathological changes seen during a myocardial infarction are as follows.

Wavy fibers are seen at the border of infarcts in the first 4 hours.

Edema and hemorrhage is maximal in the first 12 hours.

Neutrophilic infiltrate and coagulative necrosis are maximal in days 1-3.

Phagocytosis and macrophages are seen most in days 3-7.

Later findings include development of granulation tissue (days 7-10), and collagen formation(weeks 2-8).

Robbins PBOD 8ed. pp page 550, table 12-5.

How well did you know this?
1
Not at all
2
3
4
5
Perfectly
53
Q

A 63 year old female is found dead in her home by her husband. No signs of external trauma are visible. Her past medical history was significant for angina, severe pulmonary hypertension, and a recent viral illness. In addition to aortic valve stenosis, dilated cardiomyopathy, and pulmonary hypertension, which of the following conditions is most likely to cause sudden cardiac death?

Myocarditis
Cardiac myxoma
Atrial septal defect
Papillary Fibroelastoma

A

The correct answer is Myocarditis.

Explanation: Myocarditis can be seen after viral illnesses and is a cause of sudden cardiac death. Myocarditis (inflammation of the muscular tissue of the heart) has both infectious and non-infectious causes. The most common cause is viral infection, especially enteroviruses including Coxsackie B. Other viral infectious causes include adenovirus, HIV, parvovirus B19, herpes virus 6, and hepatitis C. While myocarditis often presents with heart failure, it can also be a cause of sudden death, most likely due to ventricular tachycardia or ventricular fibrillation).

Other answers: Cardiac myxoma is a primary cardiac tumor. These often cause symptoms such as dyspnea and orthopnea by obstructing flow of blood, but are not a common cause of sudden cardiac death. Atrial septal defect is a congenital cardiac defect which is generally well tolerated and often not recognized until the patient is in their 30s. Papillary fibroeleastoma is a benign neoplasm usually found as an incidental finding at autopsy.

Robbins PBOD 8ed. p 558.

How well did you know this?
1
Not at all
2
3
4
5
Perfectly
54
Q

Two and a half weeks after a 9-year-old boy misses school because of a sore throat and fever he develops hematuria (blood in his urine). On physical exam, his physician discovers that he also has elevated blood pressure and edematous fingers. Throat culture does not reveal any pathogens and the urine culture is negative. The physician’s presumptive diagnosis is acute glomerulonephritis. Which ONE of the following tests would be most helpful in establishing the diagnosis for the cause of this boy’s disease?

Assaying the boy’s serum for an elevated titer of streptolysin O antibodies
Analyzing the boy’s serum for antibodies to Shiga toxin
Examining the child’s urine for the presence of a specific capsular antigen by ELISA
Culturing the child’s stool for Escherichia coli O157:H7

A

The correct answer Assaying the boy’s serum for an elevated titer of streptolysin O antibodies.

Explanation: Glomerulonephritis is a post-suppurative complication of streptococcal pharyngitis. As such, the bacteria responsible for the infection are likely to have been cleared by the time of clinical presentation with glomerulonephritis. However, antibody to the bacterial enzyme streptolysin O will remain elevated, and thus provides a useful tool for diagnosis of this syndrome.

Other answers: Capsular antigens are not useful for diagnosis of Group A streptococci, which produce a hyaluronic acid capsule that is not highly immunogenic. The other options listed focus on Shiga toxin-producing E. coli, which would have been more likely had the patient presented with prior or ongoing blood diarrhea rather than a pharyngitis. Infections with Shiga-toxin producing E. coli typically occur after consumption of contaminated food or water can result in bloody diarrhea. A dangerous sequela that may occur following infection with Shiga toxin-producing E. coli is the hemolytic uremic syndrome (HUS). Symptoms of HUS include kidney failure, thrombocytopenia, and hemolytic anemia.

How well did you know this?
1
Not at all
2
3
4
5
Perfectly
55
Q

A 60 y/o retired captain reports to clinic with a chronic productive cough. Physical exam reveals non-tender cervical and axillary lymph node enlargement. Examination of a deep cough sputum shows gram positive diplococci. The patient is placed on a 10 day treatment with oral amoxicillin. A subsequent hemogram shows hemoglobin 9.0 g/dL; hematocrit 24%; MCV 90 µm3; platelet count 140,000/mm3; and WBC count 68,000/mm3. The peripheral blood smear shows a predominance of mature-looking lymphocytes. On flow cytometry the predominant cells are TdT-, CD19+, and CD5+. Additional laboratory tests are ordered: which of the following findings could be expected.

Beta globulin spike consistent with free L-chains
t (8,14) associated with c-Myc overexpression
X-linked mutation
Partial deletion of Chromosome 13 (or chromosome 13q deletion)

A

The correct answer is Partial chromosomal deletion.

Explanation: The patient most likely has chronic lymphocytic leukemia, a chronic lymphoproliferative disorder (neoplasm) of mature B cells. CLL is the most common leukemia in the U.S. It is more common in men than women, but is not X-linked. It typically occurs in older individuals, with the median age at diagnosis being around 70. Patients can present with fevers, sweats, weight loss, and fatigue. Physical signs include lymphadenopathy and splenomegaly. Invovlement of skin (and other organs) can also occur. Blood counts demonstrate an absolute lymphocytosis, and frequently the disease is diagnosed in asymptomatic patients after detection of lymphocytosis on a routine blood count. In some cases it can present with an unexpected infection such as streptococcal pneumonia. Flow cytometry of CLL cells demonstrates expression of B-cell antigens such as CD19 (or CD20), CD5 (a T-cell antigen that is also present on B1 B cells), and low levels of surface immunoglobulin. Cells in CLL are also typically TdT negative (TdT is terminal deoxynucleotidyl transferase, a DNA polymerase typically expressed in immature pre-T and pre-B cells).

In this case, the lymph node enlargement is consistent with the high WBC count and the finding of CD5+ lymphocytes. In ~20% of cases there is a mild hemolytic anemia which would cause a reticulocytosis in early stages of the disease. While the lymphocytes are mature and may produce immunoglobulin they do not produce free light chains as in multiple myeloma. A partial deletion of chromosome 13 (13q14 = deletion of region 14 on the long “q” arm of chromosome 13) occurs in over half of CLL cases and is a positive prognostic indicator (mechanism unknown). Unless immunosuppressed, the patient is unusually old for Burktt’s lymphoma and associated t(8;14).

Robbins PBOD 8ed. pp 603-605

How well did you know this?
1
Not at all
2
3
4
5
Perfectly
56
Q

During a routine physical examination, you determine that Colonel Needapee, a 55-year-old male, has an enlarged prostate. The colonel mentions that he has difficulty initiating urination. Also, you determine that his blood pressure is 100/70. Which drug would be most appropriate for treating the colonel’s benign prostatic hyperplasia while having only a minimal effect on his blood pressure?

albuterol.
tamsulosin.
prazosin.
phenylephrine.

A

The correct answer is D, tamsulosin.

Explanation: One way to increase urine flow in a male with benign prostatic hyperplasia is to administer a drug that will relax smooth muscle lining his urethra. Blockade of a1 adrenergic receptors on urethral smooth muscle will prevent norepinephrine from causing constriction of the urethra. Both prazosin and tamsulosin block a1 receptors and both would relieve the symptoms of benign prostatic hyperplasia. However, tamsulosin is selective for a 1A receptors while prazosin blocks both a 1A and a1B receptors. Since urethral smooth muscle expresses only a 1A receptors and vascular smooth muscle expresses both a1A and a 1B receptors, tamsulosin can effectively relax urethral smooth muscle without preventing vasoconstriction since a1B receptors remain unblocked. On the other hand, prazosin would block both a1A and a 1B receptors on the vasculature and may result in an unsafe drop in blood pressure.

How well did you know this?
1
Not at all
2
3
4
5
Perfectly
57
Q

A 67-year-old man smoked 2 packs of cigarettes daily since the age of 17. He had a prior history of many “bad colds” that he could not shake off. He would cough every morning until he would have his first cigarette puff. He presented with recent significant weight loss, weakness, constipation, and a serum calcium of 12 mg/dL. A central lung shadow was found on chest X-ray that had not been present one year earlier. A diagnostic transbronchial biopsy was obtained. The patient’s most likely diagnosis was:

Bronchiectasis
Squamous cell carcinoma
Tuberculous granuloma
Silicotic nodule

A

Answer: Squamous cell carcinoma.

The patient’s age and heavy smoking history are important clues. His clinical history is also significant for chronic bronchitis, which also is consistent with his smoking history. The features that favor a diagnosis of squamous cell carcinoma are his prior smoking history, recent significant weight loss and the presence of a radiographic lung shadow. His history of constipation, weakness and an elevated serum calcium level are all consistent with a hypercalcemia paraneoplastic syndrome, also a feature of squamous cell carcinoma. None of the other possibilities have a relationship to cigarette smoking although they may coincidentally occur in smokers. Silicotic nodules tend to be multiple and bilateral. Bronchiectasis typically is associated with production of copious, foul-smelling sputum. Tuberculosis can be associated with significant weight loss. However, it frequently also presents with fevers and/or night sweats, and hypercalcemia is not a typical feature of tuberculosis.

Robbins PBOD 8ed. pp 721-729

How well did you know this?
1
Not at all
2
3
4
5
Perfectly
58
Q

A 66 year old woman presents with abdominal pain, increased thirst, constipation and fatigue. She has a history of depression for which she takes multiple over the counter medications, including vitamins. Her examination is notable for a blood pressure of 160/72. Laboratory studies included:

Total calcium 12.5 mg/dL (8.2-10.4)
Phosphorus 2.5 mg/dL (3.0-4.5)
Creatinine 0.8 mg/dL (0.7-1.2)
Intact PTH 80 pg/mL (10-60)

24 hour urine:
Calcium 350 mg/dL
Creatinine 1.2 gm/dL

What is the MOST likely cause of this patient’s hypercalcemia?

Hypercalcemia of malignancy
Sarcoidosis
Multiple myeloma
Primary hyperparathyroidism

A

Answer: primary hyperparathyroidism

Hypercalcemia should normally cause suppression of PTH; therefore this patient has hyperparathyroidism. While frequently asymptomatic, hyperparathyroidism can cause a range of symptoms in patients.

The high PTH levels lead to development of kidney stones, bone disease, proximal renal tubular acidosis, and hyperuricemia with associated gout. High PTH levels also cause electrolyte abnormalities such as low phosphate and magnesium levels.

High calcium levels from any cause can cause disease in multiple organ systems, including the kidneys (kidney stones, nephrogenic diabetes insipidus causing increased thirst and polyuria and polydipsia, distal RTA, and renal insufficiency), the gastrointestinal system (nausea, vomiting, anorexia, pancreatitis, constipation due to decreased bowel motility), the musculoskeletal system (osteopenia, bone pain, muscle aches and weakness), the neurologic system (confusion, decreased concentration), and the cardiovascular system (decreased QT interval, hypertension, and bradycardia).

Other answers: Hypercalcemia of malignancy is mediated through a PTH-related protein that mimics PTH effects on receptors, but is not measured by the PTH immunoassay. Sarcoidosis and other granulomatous diseases cause hypercalcemia through increased conversion of 25 hydroxyvitamin D to 1,25 dihydroxyvitamin D. All of these conditions should present with low PTH levels.

How well did you know this?
1
Not at all
2
3
4
5
Perfectly
59
Q

A 73-year-old retired insulator gave a history of progressive dyspnea over many years and subsequently presented with a non-productive cough and weakness. During WW II he did extensive repairs to the insulation in the boiler rooms aboard ships. Physical examination disclosed bibasilar inspiratory crackles. Chest radiographs demonstrated bilateral irregular lung opacities in the lower lung zones as well as calcification of both hemidiaphragms. An open lung biopsy of this patient is most likely to demonstrate:

Birefringent silica particles associated with concentric, whorled fibrosis
Interstitial fibrosis associated with ferruginous bodies
Caseous necrosis of the pleura
Hyaline membranes lining the alveoli

A

Answer: Interstitial fibrosis associated with ferruginous bodies.

The patient’s occupational history is consistent with prior heavy asbestos exposure that has been well described in insulators and Navy personnel working aboard ships during WW II. The clinical picture is highly suggestive of asbestosis, as reflected by weakness and a non-productive cough in association with bibasilar inspiratory rales. Asbestosis classically is associated with bilateral irregular opacities in the lower lung zones, findings that are characterized by interstitial pulmonary fibrosis associated with dumbbell-shaped, beaded, brown asbestos (ferruginous) bodies. Persons occupationally exposed to asbestos also may have bilateral parietal or diaphragmatic pleural plaques that frequently calcify. These asbestos-related pleural plaques/calcifications occur independently from asbestosis but both types of asbestos-related lesions may coexist in the same patient.

Other answers: None of the other diagnostic possibilities have any relationship to asbestos exposure. Exposure to silica typically occurs in miners, sandblasters (individuals working in quarries), and artists (especially those working with ceramics). Silicosis is characterized by birefringent silica particles within concentric fibrous nodules, mainly in the upper lobes. This is distinct from the lower lobe location of asbestosis. Caseous necrosis is the hallmark of tuberculous granulomas and alveolar hyaline membranes are pathognomonic of adult respiratory distress syndrome.

Robbins PBOD 8ed. pp 696-701

How well did you know this?
1
Not at all
2
3
4
5
Perfectly
60
Q

A 65-year-old retiree presented with a lesion on his tongue that could not be scraped off. He stated that he first noticed it several months previously but he did not feel the need to have it examined because it did not hurt. He had been a pipe smoker for approximately 20 years. He was referred to a surgeon for evaluation. This was a photograph of the lesion. The probable diagnosis was:

Pleomorphic adenoma
Squamous cell carcinoma
Oral thrush
Leukoplakia

A

Answer: Leukoplakia.

Leukoplakia refers to firm, white, warty plaques in the oral cavity, generally of long duration, which cannot be scraped off. They are typically seen in cigarette, pipe or cigar smokers. They must be regarded as premalignant lesions although only about 5-25% actually are premalignant. They occur most commonly in males between 40 and 70 years of age.

Other answers: Oral thrush, which can be seen in young infants, individuals with AIDS or other severe immunodeficiencies, and individuals on systemic or inhaled corticosteroids, is caused by the yeast Candida albicans and is characterized by painful, creamy white, slightly raised lesions on the tongue, inner cheeks, roof of the mouth, gums, tonsils, or back of the throat. Lesions of oral thrush typically bleed slightly when scraped. Oral squamous cell carcinoma manifests either as firm, exophytic or ulcerative growths on the tongue, inner surface of the cheeks or soft palate.

Pleomorphic adenoma is a generally benign, slow-growing tumor of the major salivary glands and usually is characterized by painless, progressive enlargement of the parotid gland. It is not seen on the dorsal surface of the tongue.

Robbins PBOD 8ed. pp 743-745

How well did you know this?
1
Not at all
2
3
4
5
Perfectly
61
Q

A USUHS student went on a summer experience to rural Brazil. During his stay there, he forgot what he’d learned in Medical Parasitology and bathed daily in a small stream. After several days he discontinued these noontime swims because of marked pruritis and the onset of a rash immediately following his exposure to the water. He later learned that the stream was the main swimming and washing area for a small village 200 yards upstream. Given his bathing habits, which of the following is MOST likely to be responsible for his skin symptoms:

Sarcoptic itch
Cercarial dermatitis
Pustular dermatitis secondary to Staphylococcus aureus
Contact dermatitis

A

Answer: Cercarial dermatitis

Cercariae are the infectious stage of parasites of the genus Schistosoma, the causative agent of schistosomiasis. The species of schistosome prevalent in Brazil is Schistosoma mansoni. In this case, the village upstream has inhabitants who are infected and who are passing eggs in their stools. The eggs can be washed into the stream by rainfall. Once in water, the eggs hatch and the small ciliated miracidium stage seeks out the appropriate species of snail which serves as the intermediate host. It is from this snail host that the cercariae emerge, then swim tadpole-like in the water seeking a human host. A cercaria has enzymes in its anterior end that allow it to penetrate intact human skin, enter the capillary bed, and eventually end up in the liver where maturation to an adult worm takes place. The process of penetration of skin can invoke a local inflammatory response in the human host, causing a punctate, pruritic rash that is referred to as cercarial dermatitis. A similar skin rash can occur in humans exposed to water infected with various types of bird schistosomes, including bodies of water such as the Chesapeake Bay and the Great Lakes. This dermatitis is referred to as swimmer’s itch. These non-human schistosomes cannot complete their life cycle, so persons who have swimmer’s itch in the U.S. are not in danger of developing the disease schistosomiasis.

Other answers:

The itch referred to as ‘sarcoptic’ refers to the arthropod that causes it, the scabies mite Sarcoptes scabei. This is transmitted primarily by direct contact with an infested person and their fomites (clothing, bedding, etc.).

Contact dermatitis is typically due to a delayed hypersensitivity response to an irritant (such as a detergent or chemical solvent) or allergen (such as poison ivy or adhesives or certain metals such as nickel) and requires that the inciting (noninfectious) agent be in direct contact with the skin.

Pustular dermatitis would not have the same chronology of water exposure and the lesions (pustules) are distinctly different in appearance from the punctate, nonpustular lesions seen in cercarial dermatitis.

How well did you know this?
1
Not at all
2
3
4
5
Perfectly
62
Q

A 57 y/o post-menopausal woman G4P4 has blood drawn for routine testing prior to elective repair of a uterine prolapse. Except for the prolapse, she has had no significant medical problems. Her diet is well balanced; however, the anesthesiologist elicits a history of recent weight loss and ‘digestive problems.’ Liver and renal functional tests are normal, but her hemoglobin is only 9.4 gm/dL. Surgery is postponed while iron supplements are prescribed. A month later the hemoglobin has failed to increase and she has not gained weight. Gastroscopy and colonoscopy reveal no malignancy or source of gastrointestinal bleeding, and there is no family history of cancer. The reticulocyte count remains low. The WBC is 3,800 cells/mm3. Which of the following tests would likely prove most useful in differential diagnosis of her condition?

Serum cobalamin
Serum gastrin
Serum erythropoietin
Serum TSH

A

The correct answer is Serum cobalamin.

‘Digestive problems’ raise the possibility of chronic gastritis or intestinal malabsorption associated with folate or cobalamin (B12) deficiencies. Microscopic examination of the blood smear for evidence of oval macrocytosis, enlarged polylobulated neutrophils, or giant bands forms would be useful but not sufficient to distinguish between folate and cobalamin deficiencies. The latter is most critical to exclude since treatment with folate alone can mask disease progression toward neuropathy (which occurs with vitamin B12 deficiency). The lack of evidence for blood loss, the failure of iron supplements to correct the anemia, and the low WBC count also raise the possibilities of bone marrow dysfunction due to chronic disease or a sideroblastic anemia associated with aging. Exclusion of these conditions might require bone marrow biopsy if the serum tests were not informative.

Other answers: Provided healthy kidney function, serum EPO can be expected to increase in many types of anemia. It can be useful in distinguishing between aplastic anemia (increase) and anemia of chronic renal disease (decrease). Gastrin levels are often elevated in pernicious anemia. However, high gastrin levels are not specific for pernicious anemia or B12 deficiency, and would not be obtained as a first line test for determining whether a pt is B12 deficient. Additionally, autoantibody damage to gastrin producing glands can occur in pernicious anemia, decreasing gastrin levels. Autoantibodies in pernicious anemia may cross-react with the thyroid gland epithelium and lead to a compensatory increase of TSH, but TSH levels alone would not illuminate the underlying cause.

Robbins PBOD 8ed. pp 656-658

How well did you know this?
1
Not at all
2
3
4
5
Perfectly
63
Q

A 38-year-old Caucasian non-smoker presented with a firm, discrete, painless mass in the right parotid region that had been present for approximately 8 years. Diagnosis was obtained by fine needle aspiration. The likely diagnosis was:

Mumps
Nasopharyngeal carcinoma
Pleomorphic adenoma
Warthin tumor

A

Answer: Pleomorphic adenoma.

The parotid gland is the favored site of pleomorphic adenomas but is also the exclusive location of Warthin tumor. However, the patient’s age favors pleomorphic adenoma since the highest incidence of Warthin tumor is in the 5th-7th decades of life. Warthin’s tumor also is closely linked to smoking. Mumps occurs in much younger patients, is acute in onset and is associated with painful enlargement of the parotid gland. Nasopharyngeal carcinoma presents with neck swelling and/or nasal obstruction but does not involve the parotid region. This tumor is rare in Caucasians and is especially common in Chinese nationals.

Robbins PBOD 8ed. pp 757-759

How well did you know this?
1
Not at all
2
3
4
5
Perfectly
64
Q

A 72-year-old man presents with tremor in the left hand that is lessened during voluntary movements but becomes more pronounced at rest. He complains that he has been having trouble with his balance, and that it has been taking him a longer time to complete daily activities. Walking has become more difficult, and he exhibits short “shuffling” strides. He reports that while walking he often hesitates or “freezes up,” which has caused him to fall twice in the last month. A decision is made to prescribe a combination drug therapy consisting of L‑dihydroxyphenylalanine (L‑dopa) and an inhibitor compound, carbidopa, that does not cross the blood-brain barrier but prevents the conversion of L-dopa to dopamine in peripheral tissues. Because the conversion is promoted by a specific cofactor derived vitamin, the patient was advised to avoid abnormally high intake of vitamin supplements. Large quantities of which vitamin would be most important to avoid?

Vitamine B12
Pyridoxine
Thiamine
Biotin

A

Answer: Pyridoxine

The clinical vignette is characteristic of Parkinson disease. The conversion of L-dopa to dopamine is a decarboxylation reaction that normally takes place in the brain as well as in peripheral tissues. Amino acid decarboxylases require pyridoxal phosphate (PLP) derived from the vitamin pyridoxine (vitamin B6)*. Carbidopa is an analog of L-dopa that acts as a dopa decarboxylase (DDC) inhibitor. Because it does not pass the blood-brain barrier, carbidopa acts peripherally, but does not block dopamine formation in the brain. The peripheral action of carbidopa prevents adverse systemic effects of dopamine and helps to maintain high levels of dopa sufficient for entry into brain tissue via aromatic amino acid transporters (minimizing the pharmacological first pass effect). Consumption of high levels of vitamin B6 has the potential to promote systemic decarboxylation of L-dopa by increasing activity of residual decarboxylase, counteractive to the effect of the DDC inhibitor.

Biotin is incorrect because biotin is used for carboxylation reactions, not decarboxylations. Thiamine is involved in decarboxylations, but with alpha-keto acids (like pyruvate and alpha-ketoglutarate) not amino acids. Vitamin B12 is used for only two reactions in humans; methyl transfer reactions (methionine synthase) and a radical rearrangement that converts methylmalonyl-CoA to succinyl-CoA (methylmalonyl-CoA mustase) involved in odd-chain fatty acid and branched-chain amino acid catabolism.

*Decarboxylation reactions are also involved in converting other amino acids to a number of other neurotransmitters (e.g., histamine, GABA, serotonin). PLP is also used for other types of reactions performed on amino acids such as racemization, transamination and beta/gamma eliminations. Major metabolic pathways that require PLP include: amino acid metabolism, heme biosynthesis, neurotransmitter biosynthesis and the formation of sphingolipids. Vitamin B6 is therefore essential for normal function of the nervous system and hematopoiesis.

How well did you know this?
1
Not at all
2
3
4
5
Perfectly
65
Q

A 15-year-old boy is brought by his parents to their family doctor with a complaint of weight loss and watery diarrhea. Physical examination is unremarkable. A stool sample is negative for blood, ova, and parasites. A small bowel biopsy obtained on endoscopy reveals blunting of villi with increased chronic inflammatory cells. His symptoms improve with elimination of gluten from his diet. Which of the following is associated with this disease?

T-cell lymphoma
Hemochromatosis
Invasive aspergillosis
Hepatitis B virus

A

The correct answer is T-cell lymphoma.

The child has Celiac disease, an immune mediated disease due to hypersensitivity to gluten-containing products in genetically predisposed individuals. Most individuals who develop celiac disease carry the Class II HLA allelles, HLA-DQ2 or HLA-DQ8. Clinically they present with anemia, chronic diarrhea and fatigue. Many patients are asymptomatic. The morphology of celiac disease is villous atrophy, crypt hyperplasia and intraepithelial lymphocytosis composed of CD8+ lymphocytes. T cell lymphoma and digestive tract cancers appear to occur with greater frequency in patients with Celiac disease than in the general population. Dermatitis herpetiformis (a skin condition characterized by papules and vesicles on the extremities and trunk), Type 1 diabetes, and IgA deficiency are non-malignant diseases associated with Celiac disease.

Other answers: Hemochromatosis is a multisystem disease due to mutation in the HFE gene leading to iron overload that leads to cirrhosis, heart failure and secondary diabetes. Aspergillosis is seen in immunodeficient individuals leading to blood vessel obstruction and infarction. Hepatitis B is an enveloped DNA virus that causes acute and chronic hepatitis. While chronic infection with hepatitis B virus is associated with some extrahepatic diseases, celiac disease is not one of them. Extrahepatic manifestations of hepatitis B are typically due to circulating immune complexes and include serum sickness, polyarteritis nodosa, membranous nephropathy, and membranoproliferative.

Robbins PBOD 8ed pp 795-796

How well did you know this?
1
Not at all
2
3
4
5
Perfectly
66
Q

Treatment with a single drug failed to control a patient’s complex partial seizures. You decide to treat this patient with two anti-seizure drugs simultaneously. Your patient’s current antiseizure medication is lamotrigine. What is the MOST APPROPRIATE choice of adjunct therapy if your goals are to MAXIMIZE control of your patient’s seizures and MINIMIZE the possibility of drug interactions and an overlap in mechanism of action?

valproate
phenytoin
gabapentin
carbamazepine

A

The correct answer is gabapentin.

Lamotrigine, phenytoin, valproate, and carbamazepine all share inhibition of Na+ channels as a mechanism of action. Gabapentin, in contrast, does not inhibit Na+ channels. While the mechanism of action of gabapentin is not yet fully understood, it appears that interaction with voltage sensitive calcium channels is important for its function. Gabapentin is: 1) approved for adjunct therapy in patients with complex partial seizures; 2) does not significantly overlap mechanisms of action with lamotrigine; and 3) of all the listed anti-seizure drugs, has the lowest propensity for drug interactions because it is excreted unchanged in the urine.

How well did you know this?
1
Not at all
2
3
4
5
Perfectly
67
Q

A 66 year old physician with a family history of cystic fibrosis is approaching retirement. Ten years prior, he was successfully treated for prostatic carcinoma. He drinks one or two glasses of red wine daily. Now, he is found to have an HbA1c of 7.2%. There is no response to carbohydrate restrictions and weight loss of 10 lbs so he is placed on an oral hypoglycemic agent. At a 6 month examination he reports excessive flatus and is noted to have an additional 5 lb weight loss. A stool sample appears fatty, but serum lipase is not elevated. He elects to try Pancreaze (lipase) as a dietary supplement. Ultrasound reveals no evidence of lithiasis and the serum bilirubin is not elevated; however CT detects a 1.5 cm nodule in the head of the pancreas. It is explored and totally removed. The pathology report notes that the lesion is confined within the pancreas but abuts the pancreatic duct and is attached to an adjacent vein. Which conclusion is most accurate with respect to the complex events described:

There was a drug reaction leading to chronic pancreatitis and complications
The pancreatic insufficiency and cyst removed were delayed expressions of cystic fibrosis
This patient is at increased risk for a pulmonary embolism
This was a typical progression of chronic pancreatitis induced by excessive alcohol intake
Submit

A

The correct answer is The patient is at increased risk for a pulmonary embolism.

The patient most likely had a pancreatic carcinoma. Total removal of the pancreas is the treatment for pancreatic carcinoma and a needle biopsy or frozen section during surgery would have confirmed the diagnosis. Attachment to a vein suggest vascular invasion. Hypercoagulability is a recognized risk in these cases.

Other answers: Pancreatitis due to bile reflux can be associated with excessive ethanol intake, but in this case it presumably occurred as a consequence of secretory stasis due to a slowly growing carcinoma impinging upon the intraglandular portion of the duct of Wirsung or the duct of Santorini. While chronic pancreatitis associated with alcohol can lead to pseudocyst formation there is no evidence of a relation to carcinogenesis. Islets of Langerhans are concentrated in the pancreatic tail. Except in far advanced cases function is not usually compromised either in pancreatitis. Cystic fibrosis is frequently associated with exocrine pancreas insufficiency (and thus could account for the patient’s malabsorptive syndrome) and often with impairment of islet cell function as well (causing diabetes). However, CF does not lead to mass lesions within the pancreas. In this case, the patient’s high HgbA1c is likely due to type II diabetes unrelated to the pancreatic cancer.

Robbins PBOD 8ed. pp 900-903

How well did you know this?
1
Not at all
2
3
4
5
Perfectly
68
Q

A 55-yo man presents with sudden loss of movement and sensation on part of the left side of his body. He currently smokes a pack of cigarettes per day and has a history of smoking since his early 20s. His vital signs include: Temperature, 99.0 ºF; Pulse, 80/minute; Respirations, 16/minute; and Blood pressure, 160/100 mm Hg. A cerebral angiogram reveals an occlusion in his middle cerebral artery. Laboratory results include a hemoglobin A1C of 9%. Which one of the following components of blood lipids is most important in contributing to his disease?

VLDL
Lipoprotein lipase
Oxidized LDL
Chylomicrons

A

Correct answer: oxidized LDL

A stroke is often the consequence of cerebral atherosclerosis. LDL is the major vehicle for the delivery of cholesterol to the tissues, including the arterial walls. When LDL levels are increased or when other factors including hypertension, smoking, and diabetes are present, there is a higher degree of LDL oxidation. Scavenger receptors (SR-A1 and SR-A2) expressed in macrophages nonspecifically take up oxidized LDL into the arterial wall. This initiates the formation of atheromas or plaques that can restrict blood flow, and eventually leads to the production of emboli or a rupture in the arterial wall. Of note, in contrast to LDL, HDL is thought to act as a protective factor against atherosclerosis. HDL is involved in VLDL and chylomicron maturation, and in reverse cholesterol transport. The latter process involves the transport of cholesterol from cholesterol‑rich cells back to the liver.

Incorrect answers:

Lipoprotein lipase (LPL) is an enzyme expressed on endothelial cells in heart, muscle, and adipose tissue. LPL is a triglyceride hydrolase that requires ApoCII as a cofactor. Deficiencies in LPL lead to elevated levels of triglycerides in the blood. Individuals with LPL deficiency generally have lower levels of LDL cholesterol.

Chylomicrons are formed in intestinal epithelial cells and deliver dietary fats to the liver. Triglycerides are the most abundant component of chylomicrons. Chylomicrons are the most elevated lipoprotein when LPL or ApoCII is defective.

VLDL particles are synthesized in the liver and transport triglycerides to extrahepatic tissues. Although elevated levels of VLDL increase the risk for premature atherosclerosis, the risk is far greater when LDL levels are high.

How well did you know this?
1
Not at all
2
3
4
5
Perfectly
69
Q

A 30 year old man is being evaluated by his physician for blood in his urine. As part of the work up a kidney biopsy is done and shows the following linear IgG pattern on immunofluorescence microscopy. Which of the following features is also consistent with the disease process shown?

crescents
hyaline arteriolosclerosis
mesangial deposits
subendothelial deposits

A

The correct choice is Crescents.

The immunofluorescence image shows a linear pattern consistent with anti-GBM disease. Most patients with anti-GBM disease (Goodpasture disease, if pulmonary involvement is present) develop crescents (crescentic glomerulonephritis). Mesangial, subendothelial and subepithelial deposits are not features of anti-GBM disease. Arteriolosclerosis is seen in hypertension.

Robbins PBOD 8ed pp 913, 920-921

How well did you know this?
1
Not at all
2
3
4
5
Perfectly
70
Q

Structures that control the paracellular transport of molecules between epithelial cells but do not allow direct passage of molecules from one epithelial cell to another are

desmosomes
gap junctions
occluding (tight) junctions
centrosomes

A

Answer: occluding (tight) junctions

Explanation: Paracellular transport is the movement of substances through an epithelial layer by moving between cells; in this case between the lateral sides of epithelial cells. Epithelial cells are joined together by junctional complexes. Occluding (tight) junctions are the most apical junctions and allow epithelial cells to function as a barrier. The amount of material that is transported along the paracellular pathway is dependent on the permeability of the occluding junctions.

Incorrect answers:

Desmosomes (macula adherens) are localized spotlike, anchoring junctions between cells. They do not provide a continuous structure around the cell so are not able to function as a barrier.

Gap junctions allow direct passage of molecules from the cytoplasm of one cell to the cytoplasm of the adjacent cell.

Centrosomes are located within the cell and are the microtubule-organizing center of the cell. (Note: this is different from basal bodies, which are located near the plasma membrane under the apical surface of epithelial cells and are the microtubule organizing centers of cilia.)

How well did you know this?
1
Not at all
2
3
4
5
Perfectly
71
Q

A patient with a seizure disorder that has been difficult to control is prescribed phenobarbital. For several weeks, this patient has been taking ibuprofen, a drug that is significantly metabolized (> 70%) by CYP2C9, to reduce peripheral joint pain. A week after starting the phenobarbital therapy, the patient complains of a return of the pain in his joints. A likely reason for the return of the joint pain is that chronic phenobarbital treatment

activates constitutive androstane receptors (CAR) resulting in an increase in the expression of CYP2C9 protein, increasing the elimination of ibuprofen.
inhibits the metabolism of ibuprofen by CYP3A4.
impairs the absorption of ibuprofen from the gastrointestinal tract.
inhibits the metabolism of ibuprofen by CYP2C9.

A

The correct answer is – “activates constitutive androstane receptors (CAR) resulting in an increase in the expression of CYP2C9 protein, increasing the elimination of iburofen”

Explanation: Since ibuprofen is metabolized to a significant extent by CYP2C9, a marked increased in the amount of this enzyme is likely to increase the clearance of ibuprofen from the blood by increasing its metabolism. Phenobarbital is a known inducer of the expression of this enzyme in the liver, acting by binding to the CAR receptor. The CAR-phenobarbital complex forms part of the transcriptional protein complex that increases the rate of transcription of the CYP2C9 mRNA for this protein, and thus the amount of active protein generated. The other answers are incorrect statements.

How well did you know this?
1
Not at all
2
3
4
5
Perfectly
72
Q

A 71-year old woman was seen by her physician with a complaint of left leg weakness for three months. On examination the patellar and Achilles tendon reflexes were exaggerated on the left side and the plantar extension reflex could be elicited in the left foot. Reflexes were normal on the right side of the body and the left upper limb. Vibration and joint position sense were decreased on the left side below the level of the inguinal ligament and pinprick sensation was decreased on the right leg and foot. Which of the following conditions is most likely the cause of the patient’s neurological deficits?

Compression of the left side of the spinal cord
Occlusion of the anterior spinal artery
Multiple sclerosis
Expansion of the central canal

A

Answer: compression of the left side of the spinal cord

Upper motor neuron signs in the left lower limb (including exaggerated reflexes and plantar extension reflex) suggest a lesion to the lateral corticospinal tract on the left side of the spinal cord or the primary motor cortex on the medial aspect of the frontal lobe on the right. The sensory deficits pinpoint the lesion to the spinal cord. Specifically, loss of vibration and position sense on one side and pain on the opposite side is a hallmark of spinal hemisection. In this case, the posterior columns and anterolateral spinothalamic tracts were damaged on the left side near L1. Since the posterior column system does not cross until the lower medulla, vibration and joint position sense were lost on the left. Since the anterolateral spinothalamic tract fibers are already crossed in the cord, pain sensation was lost on the side opposite the lesion (right side).

How well did you know this?
1
Not at all
2
3
4
5
Perfectly
73
Q

A 40 year old male developed proteinuria. He had a history of recurrent abdominal pain and arthritis of the knee. A kidney biopsy revealed AA amyloid. Of the following which is most likely in this patient?

Familial Mediterranean fever
Familial amyloidotic neuropathy
Medullary carcinoma of the thyroid
Multiple myeloma

A

Correct answer: Familial Mediterranean Fever.

Explanation: Familial Mediterranean Fever (FMF) is due to mutations in the MEFV gene and is associated with spontaneous attacks of fever and pain due to serosal inflammation. The most common pain syndrome is abdominal pain due to sudden peritonitis. Pleuritis and synovitis are other common clinical manifestations. Attacks typically last for a few days and then resolve. While the pathophysiology underlying FMF is not completely understood, it is clear that inflammation plays a key role. During attacks, there is an abnormally high production of IL-1 leading to the autoinflammatory syndrome. Over time, patients with FMF can develop secondary (AA) amyloidosis. AA amyloid is produced in the setting of chronic inflammation, due to increased hepatic synthesis of the acute phase protein serum amyloid A. The AA amyloid can then deposit in the kidneys, spleen, liver, and intestines. Renal amyloidosis in FMF often progresses to nephritic syndrome and then, over a period of many years, to end-stage renal disease. Intestinal amyloidosis in FMF can result in malabsorption.

Other answers: Patients with multiple myeloma and production of a monoclonal antibody (M spike) may have AL amyloid resulting from free light chains (i.e. Bence-Jones protein). Amyloid in medullary thyroid carcinoma is composed of calcitonin and is localized to the tumor. Familial amyloidotic neuropathy leads to deposition of ATTR amyloid composed of transthyretin. Of note, Alzheimer disease (not listed as an option) demonstrates Aβ amyloid, especially in the brain, derived from the APP protein.

How well did you know this?
1
Not at all
2
3
4
5
Perfectly
74
Q

Ann goes to the gym every other day, where she cross-trains on a stationary bike. She also takes the opportunity to weigh herself on the medical scale in the women’s locker room. One day, after a week of heavy exercise and strict dieting, she was mortified to see that she had gained a pound. Unknown to her (and the other scale users), the service technician had mistakenly changed the scale’s calibration, so it was reading high. Ann responded by starving herself for 2 days. Depressed by her obsession with weight, she visited her friend Jean Ann Tonic, a bad choice, because Jean convinced Ann to have a couple of shots of vodka. Ann soon collapsed, unconscious, and was rushed to the hospital. The ER doctors found that she was severely hypoglycemic, and started IV glucose. Ann regained consciousness when her blood glucose level was brought back to normal, and she recovered without incident. At a biochemical level, how did alcohol cause the hypoglycemia?

Inhibition of gluconeogenesis in her liver
Inhibition of insulin release by her pancreas
Activation of glycogen synthesis in her liver and muscle
Inhibition of glycogen breakdown in the liver

A

In a fasted state, blood glucose levels are maintained by liver glycogenolysis and gluconeogenesis. After about 24 hours of fasting, glycogen supplies are exhausted so gluconeogenesis is the only pathway that can maintain blood glucose. Ethanol is metabolized in the liver to acetate via two dehydrogenase reactions that convert NAD+ to NADH. The effect is to greatly elevate the NADH/NAD+ ratio in the liver, thereby affecting all pathways that use this pair of cofactors. One of the pathways is gluconeogenesis, which requires pyruvate and/or oxaloacetate as starting materials. When the ratio of NADH/NAD+ is high, pyruvate is reduced to lactate and oxaloacetate is reduced to malate, making the starting materials unavailable for gluconeogenesis. Therefore, one of the mechanisms by which ethanol causes hypoglycemia is inhibition of gluconeogenesis in the liver. Additionally, there is recent data that ethanol may also cause hypoglycemia by increasing insulin release from the pancreas.

Incorrect answers:
Inhibition of glycogen breakdown in the liver is incorrect because Ann’s 2-day fast depleted her of liver glycogen. Also, ethanol does not affect glycogen breakdown, which is why most people don’t faint when they drink moderately. Inhibition of insulin release would result in increased glucose levels. Also, there is recent data suggesting ethanol actually increases insulin release. One cannot rule out activation of glycogen synthesis on first principles; ethanol could have some regulatory effect on the pathways of glycogen synthesis, but this potential mechanism has not been demonstrated.

How well did you know this?
1
Not at all
2
3
4
5
Perfectly
75
Q

A gravely ill woman, with multiple medical problems, presents to the Acute Care Clinic at Walter Reed National Medical Center. She is coughing non-stop, states that she hasn’t been able to eat or drink anything for the past 48 hours, and appears acutely ill. You are concerned that she may have pneumonia and possible renal problems. You order an arterial blood gas that reveals a significant combined metabolic and respiratory acidosis. The associated decrease in blood pH will displace the hemoglobin dissociation curve to the

left, reducing the off-loading of O2 at peripheral tissues
left, increasing the off-loading of O2 at peripheral tissues
right, reducing the off-loading of O2 at peripheral tissues
right, increasing the off-loading of O2 at peripheral tissues

A

Ans B. right, increasing the off-loading of O2 at peripheral tissues

The acidosis causes the shift of the curve to the right. Oxygen delivery to peripheral tissues is increased because the decreased affinity of hemoglobin for oxygen at the lower pH permits off-loading of oxygen at higher PaO2. Interestingly, oxygen binding to hemoglobin in the lung is only minimally changed because the partial pressure of oxygen in the lung is often still high enough to saturate hemoglobin (i.e. conditions are at the upper plateau of the curve). A graph illustrating the oxygen hemogloblin dissociation curve from the Guyton and Hall physiology textbook is below. Note that in addition to low pH, high temperature and high levels of DPG (2,3-Disphosphoglycerate, an organophosphate made in red blood cells) also drive the curve to the right.

76
Q

A 57 year old American Indian arrived at the Public Health Service clinic complaining of irregular vaginal bleeding. The patient is obese and is postmenopausal. She is currently not on hormone replacement therapy. An endometrial biopsy was performed and the pathology report lists it as “Highly Suspicious for Endometrial Carcinoma.” Subsequently, a Total Abdominal Hysterectomy Bilateral Salpingo-Oophorectomy (TAH-BSO) was performed. Which of the following lesions would be considered a pre-neoplastic condition leading to endometrial cancer?

Proliferative endometrium
Simple hyperplasia
Endometrial polyp
Complex atypical hyperplasia

A

The correct answer is Complex atypical hyperplasia.

The correct order of progression from benign endometrial change to malignant disease is as follows:
Proliferative endometrium  to     
Simple hyperplasia to
Complex hyperplasia to
Complex atypical hyperplasia to  
Endometrial carcinoma 

Complex atypical hyperplasia is considered a pre-neoplastic condition.

Robbins PBOD 8ed. pp 1030-1034

77
Q

A 36 year old woman presents to you, her primary care physician, with complaints of dysuria. She has also noted a slightly bloody tinge to her urine recently. When you take a travel history, she states that she has not traveled anywhere recently but that three years ago she moved back to the US after having lived in Egypt for 5 years. A urinalysis is performed and shows 2+ heme on dipstick; microscopic exam of centrifuged urine reveals no bacteria, no WBC or RBC casts. What do you ask this patient regarding her behavior that may have led to this infection?

Did you ever wade in irrigation canals?
Did you eat drunken crab in Cairo?
Did you ever eat steak tartare?
Do you enjoy watercress in your salads?

A

This is the egg of the causative agent of urogenital schistosomiasis, Schistosoma hematobium. All schistosomes have a snail intermediate host, and infect humans via skin penetration by the larval stage known as cercariae. Wading in irrigation canals is a typical way in which this disease is transmitted in Egypt, where S. hematobium is endemic. High rates of transitional cell carcinoma of the bladder in Egypt have been epidemiologically linked to infection with this organism.

Other answers:

Watercress is a leafy plant that is associated with the transmission of the liver fluke Fasciola hepatica.

Drunken crab is a dish served in Southeast Asia and China, consisting of a species of freshwater crab pickled in rice wine. This crab serves as the secondary intermediate host for the lung fluke, Paragonimus westermani. The larvae (metacercariae) encysted in the crab are able to survive the pickling process.

Steak tartare is prepared with raw beef and can serve as a vehicle for infection with any number of pathogens (think bacteria!); among the parasitic agents acquired by consumption of raw beef is the beef tapeworm Taenia saginatta.

78
Q

A 28-year-old spouse of a deployed sailor seeks your medical help, complaining of pain over the right upper quadrant of her abdomen, plus fever, chills, and unintentional weight loss of several weeks duration. Examination reveals hepatomegaly and an elevated right hemi-diaphragm. She had been a humanitarian aid worker in Southeast Asia for three years prior to her return to the United States, but she denies any episodes of diarrhea since she returned. Although stool examination for ova and parasites is negative, serology is positive for a protozoan. Which pathogen is the MOST LIKELY cause of this patient’s symptoms?

Cryptosporidium parvum
Microsporidium bieneusi
Cyclospora cayetanensis
Entamoeba histolytica

A

The correct answer is Entamoeba histolytica.

Explanation: Although all of these agents can cause diarrhea, the problem with amebiasis is that infection my lead to liver abscess formation due to dissemination of the organism from the GI tract. The hepatomegaly, pain, fever and chills lead us to consider a liver abscess.

79
Q

A 24-year-old medical student at USUHS presents with a chief complaint of a racing heart. Physical examination shows rapid shallow respiration, regular pulse rate of 180 beats/min, and BP of 130/80 mmHg. In the past several of these attacks lasted only a few seconds, whereas other attacks disappeared when she took a deep breath and held it. Today the Valsalva maneuver that she learned from her antiarrhythmic class was not helped. Her difficulty has lasted for almost two hours, and she is worried that she has had a heart attack. Her ECG tracing shows supraventricular tachycardia. Based on this information and her history of the arrhythmia being intermittent and reversible by what is equivalent to a Valsalva maneuver, you diagnose her as having paroxysmal supraventricular tachycardia (PSVT). Which of the following treatment is the MOST appropriate?

Enhanced Valsalva maneuver
Intravenous injection of digoxin
Intravenous injection of verapamil
Intravenous injection of adenosine

A

Correct Answer: Intravenous injection of adenosine. This is the first choice for termination of PSVT.

Incorrect answers

It is unlikely that Valsalva maneuver will work since the patient has already tried many times.

Intravenous injection of verapamil is as effective as intravenous injection of adenosine in terminating PSVT. However, whether the patient has accessory pathways (eg, WPW syndrome) is not absolutely clear. Verapamil can inhibit AV conduction and enhance conduction through accessory pathways, leading to ventricular tachycardia or fibrillation.

Digoxin is also often effective in terminating PSVT. However, it has a similar problem as described for verapamil.

80
Q

LCpl Jones presents in clinic with symptoms of fatigue and irritability following his return from a humanitarian mission. What set of additional symptoms and circumstances would most convincingly lead you to make a referral for a neuropsychological evaluation?

insomnia and increased reported nicotine use since being assigned additional shifts to cover unit members who are still deployed
hypervigilance and a score above the cutoff on a screen for posttraumatic stress symptoms with a report of working with mortuary affairs
self-reported attention and memory problems (leading to compromised job performance, and missed medical appointments) since exposure to toxic chemicals during this mission
feelings of social isolation and sadness upon learning of his girlfriend’s infidelities while he was deployed

A

Correct answer: Self reported attention and memory problems since exposure to toxic chemicals during this mission.

Explanation: A change cognitive abilities following potential injury to the brain is an important reason for referral to a neuropsychologist. Most symptoms listed in the other answers are consistent with posttraumatic stress or re-adjustment issues and may be more appropriate for referral to a psychologist/behavioral health professional. However, when cognitive impairment, identified brain pathology, or deteriorating mental status is present, a referral for neuropsychological assessment is most appropriate to help with diagnosis, monitoring and treatment planning.

81
Q

A 50 year old woman presents to her physician for a lump in her breast. A mass is excised and submitted to pathology for diagnosis. The pathology report describes clusters of pleomorphic cells, some of which are forming ducts and invading the breast parenchyma. Which of the following would indicate that the lesion is more aggressive?

Presence of fibrocystic change
Her2/neu amplification
Mucinous histology subtype
ER receptor positivity

A

The correct answer is Her2/neu amplification.

The histological description indicates infiltrating ductal carcinoma. Among the 4 choices Her2/neu amplification would portend the worst prognosis. The other features would indicate a better prognosis. Generally, the following features indicate a poor prognosis: distant metastases, lymph nodes metastases, tumors greater than 2 cm in diameter, presence of inflammatory carcinoma, high grade histology, Her2/neu amplification, and ER receptor negativity.

Robbins PBOD 8ed. pp 1066-1093

82
Q

During the follicular phase of the menstrual cycle, the secretion of estrogen by the ovary increases greatly, whereas, levels of circulating gonadotropins remain constant or may even decline a little. How does the increase in estrogen output occur in the face of stable, or declining, levels of LH and FSH?

The half life of LH and FSH in the blood progressively increases throughout the follicular phase.
The anterior pituitary becomes progressively less sensitive to the negative feedback actions of progesterone and estrogen.
The developing follicles enlarge and become more sensitive to LH and FSH through up-regulation of gonadotropin receptors.
The hypothalamic response to the positive feedback actions of estrogen becomes progressively stronger throughout the follicular phase.

A

Correct answer: The developing follicles enlarge and become more sensitive to LH and FSH through up-regulation of gonadotropin receptors.

Explanation: LH and FSH receptors are upregulated to increase the sensitivity of the follicle. At the same time the developing follicles enlarge in size, enhancing their ability for steroid biosynthesis.

Other answers:

The positive feedback actions of estrogen are only manifest for a brief period at the time of ovulation.

The half life of LH and FSH are not regulated during the course of the cycle.

There is no regulation of the hypothalamic set point during the cycle, thus negative feedback by estrogen and progesterone is incorrect.

83
Q

A 34-year-old female complains of heat intolerance, palpitations and increased sweating. She has lost 8 pounds during the last 3 months, despite her increased appetite. Physical examination reveals tachycardia, cardiomegaly, tremor, exophthalmos, diffusely enlarged thyroid gland, wide-staring gaze and lid lag. Serum TSH is low, free T4 high. Which of the following statements best fits the disease process?

T4 serum levels is the single most useful screening test for this disease
Increased thyroid radioactive iodine uptake would suggest thyroiditis as a probable etiology
A multinodular increase of thyroid radioactive iodine would strongly suggest Graves’ disease
This is an autoimmune disorder caused by a type 2 hypersensitivity reaction.

A

The correct answer is d: This is an autoimmune disorder caused by a type 2 hypersensitivity reaction.

Explanation.

The clinical features suggest hyperthyroidism due to Graves’ disease. Graves’ disease is an autoimmune disorder resulting from a type 2 hypersensitivity reaction (Robbins, table 6-4). Specifically, autoantibodies in Graves’ disease bind and activate the TSH rececptor. These autoantibodies tend to be of the IgG1 subclass and oligoclonal in nature. Hypersensitivity reactions are divided into four types: type 1 (immediate, IgE mediated), type 2 (delayed, antibody mediated), type 3 (delayed, immune complex deposition with complement activation), and type 4 (delayed, T-cell mediated). As Graves’ disease is due to IgG autoantibodies, it can be thought of as a type 2 reaction.

Other answers:

A diffuse increase of thyroid radioactive iodine uptake, not a multinodular increase, is typical of Graves disease (Robbins, page 1115).

Decreased thyroid radioactive iodine uptake is a characteristic of thyroiditis (Robbins, page 1113).

TSH is the single most useful screening test for hyperthyroidism since levels are decreased even at the earliest stages of disease (Robbins, page 1109).

84
Q

The stress levels of 150 stroke patients admitted to a hospital in Madrid, along with 300 of their healthy neighbors, were evaluated on several dimensions. Researchers quantified stress levels in terms of stressful life experiences the subjects had undergone in the past year. The odds of having a stroke nearly quadrupled if the subject lived under stressful conditions in the previous year (J Neurol Neurosurg Psychiatry doi:10.1136/jnnp-2012-302420). This is an example of which of the following study designs?

Cross-sectional
Case series
Cohort
Case-control

A

In a case-control study, participants are identified based on their disease or outcome status, and information about their prior exposure is collected retrospectively. Here, the disease or outcome is stroke. Cases (stroke patients) and controls (healthy neighbors of stroke patients) were interviewed about their level of exposure in the past (stressful life experiences in the past year).

A case series only evaluates patients with disease and does not include a comparison group. Cross-sectional studies measure exposure and disease at the same time, so would be unable to distinguish between stress as a risk factor for stroke and stress as a consequence of stroke. Cohort studies identify participants on the basis of exposure and follow them forward in time; a cohort study of stress and stroke might identify participants with high and low stress and follow them forward to determine the incidence of stress in each group.

85
Q

A 24-year-old Army sergeant returns from a tour of duty in Afghanistan with a chief complaint of a skin lesion on her right wrist. The patient first noticed the lesion 3 months ago. At that time it appeared as a 0.5cm papule. Two months ago it ulcerated and became painful and filled with pus. Antibiotic therapy at that time (with amoxicillin/clavulanate) resulted in elimination of the pus and in cessation of the pain, but the ulcer has persisted and enlarged in size to 1cm in diameter. The method MOST LIKELY to establish the etiology of this wound is:

Serology for fungal antibodies
examination of stool for ova and parasites
culture of an aspirate or punch biopsy from the edge of the lesion
bacterial culture of scrapings from the middle of the lesion

A

The answer is Culture of an aspirate or punch biopsy from the edge of the lesion.

Explanation: The duration of this infection, the geographic history of the patient, and the failure of antibiotics to eradicate the ulcer suggest the infection is attributable to Leishmania species. These parasites enter the body as flagellated promastigotes through the bite of the sandfly. The promastigotes are rapidly taken up by macrophages where they change into the amastigote replicative stage. These non-flagellated amastigotes remain intracellular except when their growth results in cell lysis and spread to neighboring cells.

The diagnosis is typically made by culture for Leishmania parasites using NNN (Novy, MacNeal, Nicolle) media or liquid Schneider’s media. Sample for culture is typically obtained by injecting and then aspirating sterile saline solution from multiple areas of the lesion, or by obtaining a punch biopsy near the edge of the lesion. Histology is also typically performed, and may reveal amastigote forms in macrophages. At WRNMMC, we also perform PCR on aspirates and biopsies for speciation.

While Leishmania ulcerations often become superinfected with bacteria (as appears may have occurred in this case), the bacteria are not the underlying cause of the lesion.

86
Q

Men with early-stage prostate cancer were randomized to undergo either radical prostatectomy or observation. By the end of the 15-year study, 171 of 364 men assigned to radical prostatectomy died, compared with 183 of 367 assigned to observation. An appropriate statistical test of whether all-cause mortality differs between the two study groups would be:

T test for independent samples
Chi square test
Pearson correlation
Analysis of variance

A

Answer: chi square test

The research question involves comparing two proportions, and chi square tests are used to compare proportions among two or more groups. T tests and analysis of variance are used to compare means, not proportions – t tests compare two means and analysis of variance can be used to compare more than two means. Pearson correlation describes the extent to which subjects with high values for one variable have high values for a second variable.

87
Q

A screening test for prostate cancer is introduced which increases survival time, but only adds time between early diagnosis and the usual time when diagnosis would have been made. This is an example of:

Length bias
Overdiagnosis bias
Lead time bias
Selection bias

A

Correct answer: lead time bias

Explanation: All of these are biases inherent in evaluating screening programs, but lead time bias is what is described above. If a new screening test is used on a population and it is able to identify cases of disease at a point earlier in the course of disease, but the available treatment is ineffective in delaying death, lead time bias may be present in interpretation of the results. Lead time bias is a bias that occurs when a test identifies a disease state earlier than current practice, but does not alter actual survival time. For example: Consider a person is diagnosed at age70 with a specific cancer and dies from that cancer at age 75. Now assume a screening test could have found the cancer at age 65, but the treatment was ineffective and the person still would have died at age 75. The person’s survival from diagnosis without screening was 5 years. The person’s survival from diagnosis with screening was 10 years. Thus, if you just looked at survival from diagnosis, it appears that screening doubled survival time. However, the person derived no mortality benefit from screening as they died at the exact same age. Consequently, if you implement a screening test, you have to take into account how much earlier in the course of disease the test identified disease in considering the benefits of screening. If there is no effective treatment, you may actually cause harm in that the person may just live with the knowledge they are going to suffer and die prematurely.

Length bias occurs when a better prognosis for cases detected directly by the screening procedure than for cases diagnosed between screening exams is used as evidence that the screening program is effective (i.e. screening detects less aggressive cases with better prognosis). Overdiagnosis bias occurs when screening identifies cases that would not have clinically surfaced in a patient’s lifetime because of lack of progressive potential or death from another cause. Examples include breast and prostate cancer in older patients, who will normally die with the disease, not from it. Selection bias occurs when comparison groups differ in ways other than the main factors under study, but which influence the outcome. One example of this is the fact that healthy volunteers are more likely to engage in screening procedures than controls, making the screening test seem more effective than it truly is.

88
Q

A term gestation stillborn male infant is noted to have extensive ascites and is covered with numerous petechiae over the trunk and extremities. An examination of the face reveals that one eye is smaller than the other and that the retinas have multiple areas of discoloration. Autopsy x-rays of the brain show markedly dilated lateral ventricles surrounded by irregular densities of the cerebral cortex. On histological examination, many organs, including the liver, kidney, and lungs, displayed findings similar to those on the displayed image. Which of the following is the most likely causative organism for this congenital illness.

Cytomegalovirus
Respiratory syncytial virus
Herpes zoster virus
Epstein-Barr virus

A

ANSWER: Cytomegalovirus.

The cell in this image is infected with cytomegalovirus (CMV). The histologic features of a large cell with a large, eccentric nucleus containing a smooth, dark-purple, intranuclear inclusion with smaller, cytoplasmic inclusions is pathognomonic of CMV. One of the TORCH group of infections (Toxoplasmosis, Other, Rubella, CMV, Herpes) seen in newborns, CMV is acquired in utero through an infection of the mother that is transferred to the developing infant through the placenta. When infection occurs in the first trimester, the effects may be severe, including CNS necrosis and calcification, microophthalmia, deafness, hepatomegaly, and pneumonia, among others. With second or third trimester infections, the sequelae are much milder and may include only shedding of the virus, detectable in the urine.

Other answers: Herpes virus infection may rarely be acquired in utero and usually infects the infant as he or she is passing through the infected birth canal of the mother. However, the herpes virus inclusions are intranuclear only. RSV and EB virus are seen in older infants and children but rarely as a congenital infection.

Robbins PBOD 8ed. pp 353-355

89
Q

A 17-year-old sexually active adolescent presents to her physician with a chief complaint of abdominal pain. The pain was initially localized to the umbilicus but subsequently migrated to the right lower quadrant. The abdomen is rigid on physical examination. Bowel sounds are absent. A complete blood count shows an elevated white blood cell count with increased numbers of immature WBCs. Examination of the appendix shows transmural inflammation. Which of the following is the most likely etiology for the patient’s clinical presentation?

Fecalith
Ruptured ovarian cyst
Ectopic pregnancy
Pelvic inflammatory disease

A

The correct answer is Fecalith.

The patient has acute appendicitis diagnosed based on transmural inflammation of the appendix. Fecalith, a hard mass of fecal material, can cause appendicitis by obstructing the appendix. Obstruction of the appendix can lead to distension of the appendix by mucus, resulting in distension of the appendix and increased transmural and intraluminal pressures within the appendix. These processes can lead to thrombosis and occlusion of the small vessels of the appendix, resulting in ischemia and ultimately necrosis of the appendix. Other causes of appendicitis include lymphoid hyperplasia, tumors, calculia, and infections. However, it is important to note that most patients operated on for appendicitis do not have a fecalith or other obvious cause of appendix obstruction found at surgery.

Ectopic pregnancy, pelvic inflammatory disease, and ruptured ovarian cyst all may present with abdominal pain but do not cause acute appendicitis although they may lead to serosal inflammation of the appendix.

Robbins PBOD 8ed pp 826-827

90
Q

A 55 year old man presents to the emergency department with acute shortness of breath and chest pain which radiates to the left neck. The pain subsides with nitroglycerin. Angiography reveals 70% narrowing of the proximal LAD artery due to severe atherosclerosis. In addition to endothelial injury, accumulation of lipoproteins and foam cells, which of the following pathophysiologic events is important to the formation of atherosclerosis?

Normal endothelial cell function
Decrease in neutrophils
Release of growth factors such as PDGF
Smooth muscle atrophy within the atherosclerotic plaque

A

Correct answer: Release of growth factors such as PDGF.

Explanation: Release of growth factors, such as PDGF (platelet-derived growth factor), from macrophages, platelets and vascular wall cells is important for smooth muscle cell recruitment.

Smooth muscle cells are recruited into atherosclerosis, they do not atrophy. Neutrophils are not a major part of atherosclerosis, whereas monocytes and T-lymphocytes are. Turbulence is important for atherosclerotic plaque formation, laminar flow is protective. Endothelial cell dysfunction with increased endothelial cell permeability is important for atherosclerosis.

Robbins PBOD 8ed. pp 496-503

91
Q

A 42 year old Major presents with increased tremulousness, anxiety and palpitations. On exam, her pulse is 116 and her blood pressure is 160/70. She is thin and anxious appearing. She has warm and moist skin. She has proptosis. Her thyroid gland is firm and diffusely enlarged. Neurologic exam reveals a fine tremor of her upper extremities and a shortened relaxation phase of her deep tendon reflexes. Her serum TSH is depressed. What is the most likely cause of her symptoms and signs?

Graves’ disease
Thyroid carcinoma
TSH secreting adenoma
Hashimoto’s thyroiditis

A

Graves’ disease is a hyperthyroid disease which is often associated with goiter, proptosis, and, in rare cases, an infiltrative skin condition known as pretibial myxedema. The disease is due to autoantibodies which activate the TSH receptor. Additionally, the proptosis in Graves’ disease is likely due to the action of autoantibodies, possibly targeting fibroblasts in the eye muscles and causing inflammation within the orbit.

While Hashimoto’s thyroiditis typically leads to hypothyroidism (and thus high TSH levels), early in the course of disease it can cause hyperthyroidism with decreased TSH levels. However, neither Hashimoto’s thyroiditis nor thyroid carcinoma typically causes proptosis. TSH is increased in TSH secreting adenoma.

92
Q

A 29-year-old male is admitted to the emergency room because of lacerations in his nose and forehead from being hit with a snowball containing a large chunk of ice. He states that he is allergic to ester-type local anesthetics. The MOST APPROPRIATE choice of local anesthetic to inject prior to suturing is

tetracaine
procaine
lidocaine
benzocaine

A

The correct answer is lidocaine.

Explanation: Lidocaine is the only amide-type (non-ester) local anesthetic in the list of choices. Furthermore, benzocaine is primarily used topically in the clinic and not injected.

93
Q

A 6-year-old child becomes ill with fever, chills, and muscular pain. These symptoms persist for several days and then discontinue. Several days later, she develops an erythematous rash on her face and upper limbs. She also experiences arthritic pains in several joints. These symptoms last for several days and then discontinue. She is MOST LIKELY suffering from an infection with which ONE of the following viruses?

Varicella-Zoster Virus
Human Herpes Virus Type 6
Measles
Parvovirus B19

A

The correct answer is parvovirus B19.

The typical course of infection with parvovirus B19 is a febrile illness that is followed by a rash that is pronounced on the face (slapped cheek) AFTER the resolution of the febrile stage. The development of polyarthritis separates parvovirus B19 infection (fifth disease) from sixth disease associated with HHV6 for which the rash is more predominant on the torso and not as much so on the face. The rash of varicella-zoster virus is typically vesicular and classically occurs on the trunk and head, and then extends to the limbs. The rash of measles is classically a generalized (all over the body) macular/papular rash.

94
Q

You evaluate a 52-year-old female who complains of intense itching and flushing, typically triggered by alcohol or aspirin ingestion. On examination she presents with multiple widespread, round-to-oval, red-brown, non-scaling papules and small plaques. Bone marrow, skin and liver biopsies reveal fibrosis and round to oval infiltrates. Localized rubbing induces a dermal edema that resembles a hive. Which of the following statements is most likely?

Point mutations of the c-KIT receptor tyrosine kinase are found in many cases
About 10% of individuals, mostly children, have a systemic form of the disease
The cells responsible for this condition are lymphocytes.
Many of the signs and symptoms are caused by the release of histamine and epinephrine

A

Correct answer is Point mutations of the c-KIT receptor tyrosine kinase would be an expected genetic change.

The clinical findings are consistent with systemic mastocytosis. Urticaria pigmentosa is a limited cutaneous form of this disease that affects mainly children. About 10% of individuals with mastocytosis have the systemic form of the disease with cells infiltrating multiple organs. Systemic mastocytosis can occur with or without skin involvement. Systemic mastocytosis occurs primarily in adults. Many of the signs and symptoms of systemic mastocytosis are caused by the release of histamine and heparin. Epinephrine may be used to treat severe cases manifesting as anaphylaxis. Point mutations of the c-KIT receptor tyrosine kinase increase c-KIT signaling and drive mast cell growth and survival. The cells responsible for this condition are mast cells, not lymphocytes. Mast cells are primarily tissue-residing cells with intracytoplasmic granules and multilobed nuclei. Metachromatic stains (toluidine blue, Giemsa) may help visualize the granules .

(Robbins, page 1185-1186).

95
Q

You are monitoring a 43-year old liver transplant recipient currently receiving cyclosporine A. Over several weeks post-transplant, you notice elevated serum creatinine levels and a reduction in glomerular filtration rate. To avoid further nephrotoxicity and prevent allograft rejection, you decide to:

Reduce cyclosporine A dosage and administer tacrolimus.
Stop cyclosporine A treatment and observe the patient’s clinical course without immunosuppressive therapy.
Reduce cyclosporine A dosage and administer mycophenolate mofetil.
Maintain cyclosporine A dosage and administer prednisone.

A

The best choice is C reduce cyclosporine A dosage and administer mycophenolate mofetil (MMF).

The key to this question is avoiding nephrotoxicity by reducing the dose of the calcineurin inhibitor. MMF is an antimetabolite extensively used in solid organ transplant patients experiencing calcineurin inhibitor-induced nephrotoxicity. It effectively prevents acute graft rejection in combination with low-dose cyclosporine A. Tacrolimus is another calcineurin inhibitor associated with nephrotoxicity, so this is not an acceptable alternative to preserve renal function. Discontinuing immunosuppressive treatment of any kind will likely lead to acute allograft rejection, whereas maintaining the current dose of cyclosporine A will precipitate a further decline in renal function.

96
Q

A 4-year-old female is brought to her family medicine physician when her parents noted that she was limping. Her mother did not remember any inciting event. When she awoke this morning, she was irritable, felt warm and was noticeably limping on her right leg. She does not localize the pain well but points to her knee when questioned. Her mother noted some response to ibuprofen. On physical exam, her temperature is 103.2oF and she has tenderness to palpation over the distal femur. Laboratory evaluation is pending. Which of the following options best represents the etiology of her illness?

Osteomyelitis due to Staphylococcus epidermidis
Reiter’s syndrome
Disseminated gonorrheal infection
Osteomyelitis due to Staphylococcus aureus

A

The correct answer is Osteomyelitis due to Staphylococcus aureus.

Explanation: Two pyogenic cocci, Staphylococcus aureus and Streptococcus pneumoniae (not a choice here) predominate in bone and joint infections of children. These bacteria generally reach the site of infection by the hematogenous route from infections elsewhere in the body. In this age group disseminated gonorrheal infection is rare. Ostemyelitis attributable to S. epidermidis is uncommon but occurs in conjunction with post-operative infections in joint replacement. Reiter’s syndrome, also referred to as reactive arthritis, is a sequela to infection with certain gastrointestinal and urogenital pathogens such as Shigella, Neisseria and Chlamydia. It typically is not common in young children nor is it accompanied by a high fever and bone tenderness.

97
Q

A 78-year-old man with a history of hypertension and tobacco use presented at a local emergency room with a complaint of sudden onset left-sided weakness. Examination revealed diminished strength and hyperreflexia in the left upper and lower limbs and weakness of the musculature on the lower left side of the face. The corneal reflex could be elicited in both eyes. However, there was ptosis of the right eyelid, the right eye was deviated outward and downward, and the right pupil was dilated and unreactive to light. The patient’s neurological deficits were most likely caused by a lesion involving the

Ventral midbrain
Dorsal medulla
Ventral pons
Ventral medulla

A

Correct answer: ventral midbrain

The deficits are consistent with damage to the corticospinal and corticobulbar tracts as well as cranial nerve III. Cranial nerve III, the oculomotor nerve, controls most of the eye’s movements and is responsible for elevating the eyelid (by innervating the levator palpebrae superioris) and for constricting the pupil (by innervating sphincter papillae). Thus, a lesion to CNIII causes the eye to rotated in a “down and out” position and the pupil to be dilated.

The descending cortical fibers are located in the crus cerebri on the ventral aspect of the midbrain, and CN III fibers pass through the crus cerebri to exit the brainstem en route to the orbit. This patient most likely had a CVA involving the penetrating paramedian branches of the posterior cerebral artery that supply the right ventral midbrain.

98
Q

A 68-year-old male consults you about his increasingly severe back pain; he has also noticed that his hat does not fit anymore. X-rays disclose enlargement of cranial bones, bilateral tibia and several vertebrae. The bones show thick and coarsened cortices and cancellous bone. The skin overlying involved bones is warm. Which of the following statements is correct with this disease process?

Sarcomas develop in 0.7-0.9% of affected individuals, usually osteosarcomas or fibrosarcomas
An initial osteosclerotic phase typically ends in a quiescent osteolytic phase. The net effect is a loss of bone mass
Less than 10% of individuals with Paget disease have a family history suggestive of autosomal recessive inheritance, up to 2% may show x-linked inheritance.
Fibrosis, woven bone formation and fractures are the histologic hallmarks of this disease

A

The correct answer is Sarcomas develop in 0.7-0.9% of affected individuals, usually osteosarcomas or fibrosarcomas.

This patient has Paget disease of the bone. Sarcomas develop in 0.7-0.9% of affected individuals, usually osteosarcomas or fibrosarcomas. An initial osteolytic phase typically ends in a quiescent osteosclerotic phase. The net effect is a gain in bone mass. 15-40% of individuals with Paget disease have a family history suggestive of autosomal dominant inheritance. Some familial and sporadic cases have SQSTM1 gene mutations that through RANK signaling increased osteoclastic activity. A mosaic pattern of cancellous bone, produced by prominent cement lines, is the histologic hallmark of this disease.

Robbins pages 1216-17

99
Q

Retinoblastoma is a rare type of eye cancer that usually develops in early childhood as a result of mutations in the RB1 gene. A significant proportion of disease-causing mutations cause premature termination of RB1 protein synthesis due to mutation of a CGA (arginine) codon to a TGA (stop) codon. CG dinucleotides are sites of C5-cytosine methylation and recurrent C→T mutations, including those found in RB1, apparently result from deamination of 5-methylcytosine (5meC) within CG dinucleotides. A likely explanation for this observation is that 5meC

mispairs with guanine
deaminates to thymidine
mispairs with adenine
deaminates to uracil

A

Answer: deaminates to thymidine

Cytosine spontaneously deaminates to form uracil; uracil is not normally found in DNA and since it base-pairs with adenine, if it is not removed and replaced with cytosine, a C∙G →T∙A transition mutation will result after a round of DNA replication. Because of this, uracil is routinely removed from DNA via a uracil N-glycosylase (a base excision repair enzyme). In contrast to cytosine deamination, 5meC deaminates to thymidine. Since thymidine is a normal component of DNA, it is not recognized by the base excision repair enzymes, resulting in an uncorrected C∙G →T∙A mutation.

100
Q

A 46-year-old female complains of increasingly severe joint pains over the last 4 months. Past history discloses gradual onset of malaise and mild to moderate musculoskeletal pains 8 months prior to the joint pains. On physical examination the hand (metacarpophalangeal and proximal interphalangeal) and feet joints are swollen, red and painful. X-rays of those joints show effusions and mild juxta-articular osteopenia. Which of the following statements is correct?

This disease principally affects the joints, producing a non-suppurative, proliferative and inflammatory synovitis
Anti-TNF typically cures the disease
The presence of rheumatoid factor is fairly sensitive and specific for rheumatoid arthritis
About 80% of affected patients have autoantibodies (mostly IgG) to the Fc portion of autologous IgM

A

The correct answer is This disease principally affects the joints, producing a non-suppurative, proliferative and inflammatory synovitis.

The patient has rheumatoid arthritis, which causes a non-suppurative, proliferative and inflammatory synovitis. MCP and PIP joints, as in the question stem, are frequently involved. The pathogenesis of rheumatoid arthritis involves autoimmune activation of CD4+ helper T-cells, which in conjunction with cytokines and local inflammatory mediators, destroy joints. 80% of affected patients have autoantibodies to the Fc portion of autologous IgG. This type of antibody is called rheumatoid factor. The isotype of rheumatoid factor is usually IgM (not IgG as mentioned in one of the incorrect answers). The presence of rheumatoid factor and anti-CCP antibodies together, in conjunction with appropriate clinical findings, are fairly sensitive and specific for rheumatoid arthritis. A positive rheumatoid factor alone is not very sensitive, as it can be elevated in a number of autoimmune and other inflammatory conditions. Anti-TNF may prevent or slow joint damage, but does not cure the disease.

Robbins pages 1237-40.

101
Q

The benefit of exercise in the management of type II insulin-independent diabetes is due to its ability to:

increase the sensitivity of exercising muscle to insulin.
decrease the secretion of ACTH.
increase the use of glucose by the central nervous system.
increase the use of glucose by adipose tissue.

A

Correct answer: increase the sensitivity of exercising muscle to insulin.

Explanation: Exercise increases the number and activity of glucose transporters working on the membrane of muscle cells.

Incorrect answers

The utilization of glucose by the CNS does not vary under physiologic conditions

The hypothalamic-pituitary-adrenal axis is activated in response to the “stress” of exercise.

Exercising muscle, not adipose, is the tissue that responds to exercise with increased insulin sensitivity. Adipose tissue mobilizes its energy stores (free fatty acids and glycerol) in response to the counter regulatory hormones.

102
Q

A 30 year old Middle Eastern male who immigrated to this country three years ago presents to the clinic for a required physical examination for a new job. During the physical exam, it was discovered that he had only one testicle that was properly descended in his scrotal sac. After a proper work-up, he was immediately referred to a surgical service for an orchiectomy of the undescended testicle. Per the surgical pathology report, the patient was found to have a mixed-germ cell tumor of the testicle. Which of these germ cell tumor components would guide the overall medical therapy based on its prognosis?

Choriocarcinoma
Seminoma
Leydig tumor
Embryonal carcinoma

A

The answer is Choriocarcinoma.

A history of undescended testicle is a risk factor for germ cell tumor development. Germ cell tumors include seminoma, embryonal carcinoma, choriocarcinoma, yolk sac tumor, and teratoma. About 30% of germ cell tumors are mixed, meaning they have cells from 2 or more germ cell tumor types. Choriocarcinoma is frequently found in combination with other germ cell tumors. Choriocarcinoma is typically more aggressive than the other germ cell tumors due to its tendency to metastasize via the vascular system to liver, lung, brain, and bone. Thus, when present in a mixed germ cell tumor, presence of choriocarcinoma generally drives what medical therapy is given. Chemotherapy is used to treat metastatic choriocarcinoma and radiation therapy is generally used for localized disease. Embryonal carcinoma, Seminoma, and Yolk sac tumor are germ cell tumors that are less aggressive clinically than choriocarcinoma. Leydig tumor is designated as a sex cord-gonadal stromal tumor, not a germ cell tumor.

Robbins PBOD 8ed. pp 987-993

103
Q

Sharon Stone is a 59-year-old female who has recently suffered a myocardial infarction. Her doctor placed her on a b-blocker, nitroglycerin, captopril and an aspirin. Since Ms. Stone also had symptoms of congestive heart failure, her doctor also put her on a diuretic. After one month of this therapeutic regimen, Ms. Stone’s congestive symptoms have been resolved, but she has now developed a kidney stone composed of calcium oxalate due to excess secretion of Ca2+. What diuretic should you switch to in order to DECREASE urinary Ca2+ excretion while maintaining good diuresis?

 spironoloactone
 hydrochlorothiazide
 acetazolomide
 mannitol
 furosemide
A

The correct answer is hydrochlorothiazide.

Explanation: Thiazide diuretics, like hydrochlorothiazide, are the only diuretics that decrease the amount of Ca2+ in urine. Ms. Stone was probably originally on a loop diuretic, like furosemide, that increases Ca2+ secretion and caused the formation of Ca2+-containing kidney stones.

104
Q

A 55-year-old man develops acute onset of pleuritic pain, fever, and cough. His sputum sample reveals large numbers of polymorphonuclear leukoctes and Gram positive cocci that primarily occur in pairs. On history, his physician notes that 2 weeks earlier the man had experienced a rapid onset of fever, chills, headache, and muscle aches that lasted about 10 days. His current illness was likely a complication that resulted from a prior infection with a

single-stranded, non-segmented, non-enveloped positive-sense RNA virus
single-stranded segmented, enveloped negative-sense RNA virus
single-stranded, non segmented, enveloped, negative-sense RNA virus
double-stranded, non-enveloped DNA virus

A

The correct answer is Single-stranded, segmented, enveloped negative-sense RNA virus.

Explanation: Influenza virus infection predisposes people to secondary bacterial infections by agents such as Staphylococcus aureus and Steptococcus pneumoniae. The case does not allow us to discriminate between those bacterial infections. However, the history suggests that the patient’s problems started with influenza, a single stranded, segmented virus with negative sense RNA.

105
Q

A 68-year-old Navy captain with type II diabetes mellitus, which he has had for the past 15 years, complains of lower extremity sensory problems. Which of the following will most likely be seen on physical examination?

Diffuse enlargement of the peripheral nerves on palpation
Sensory loss in only one leg
Symmetrical distal sensory loss in a “stocking-glove” distribution
Increased deep-tendon reflexes at the ankles and knees

A

Correct answer is Symmetrical distal sensory loss in a “stocking-glove” distribution.

The patient has diabetic neuropathy, a complication seen in over 50% of diabetics. It is a symmetrical process that distally involves both motor and sensory functions. It is typically a bilateral process. Diffuse enlargement of peripheral nerves is not observed in diabetic neuropathy – it is typically encountered in hypertrophic (hereditary) forms of neuropathies such as Dejerine-Sotas disease. Deep tendon reflexes are decreased in diabetic neuropathy as it is a peripheral neuropathy. Increased DTRs are indicative of upper motor neuron lesions.

106
Q

A 61-year-old man is brought to the Emergency Room by a co-worker for chest pain that began suddenly after receiving word he was laid off. He describes the pain as a dull ache with sensation of tightness in his chest. The pain does not change significantly with respiration or position but does radiate upwards towards his jaw. He has smoked 1 pack of cigarettes every day since he was 18 years old and does not regularly exercise. He is not followed for routine care and denies any personal medical problems. He reports his parents both died in their 70s from heart problems. He appears anxious and diaphoretic. His vital signs on arrival are HR 102, BP 162/88, and RR 17. Pulse oximetry shows 96% on room air. His exam is revealing of an S4 and a 2/6 systolic ejection murmur on auscultation. An ECG is ordered and labs are pending. Based on the available information, what is the most likely etiology of this patient’s chest pain?

Unstable Angina/acute coronary syndrome
Pulmonary Embolism
Stable Angina
Pericarditis

A

Answer: unstable angina/acute coronary syndrome

This patient is presenting to the Emergency Room with classic angina pectoris. His history reveals high risk features including male, age, sedentary lifestyle, and smoking history. His presentation and examination demonstrate findings common in uncontrolled essential hypertension, a strong risk factor for atherosclerotic coronary artery disease. The acute presentation here excludes stable angina. Although a pulmonary embolism may present with chest pain, this is classic angina without pleuritic findings, and no observed DVT or historical precipitating factors (such as a long flight or car ride, recent surgery) or clinical risk factors (no known malignancy and no known inherited clotting disorders). It is possible for pericarditis to present rather quickly. However, his chest pain is not characteristic of pericarditis, given its dull quality. Also, there is no palliation of the pain with leaning forward or sitting upright.

107
Q

A 45-year-old woman complains generalized weakness which she indicates has developed over a two-year period. Upon examination you note pain and stiffness of her joints and a bilateral red-purple rash in her periorbital regions. Her weakness is predominantly of proximal muscles, with sparing of distal muscle function. Chest x-ray indicates the presence of bilateral fibrosis of the lungs in an interstitial pattern. This patient is likely suffering from a disorder that is characterized by which of the following?

an increased frequency among patients being treated with cholesterol-lowering statin medications
mutation of DMD gene
an increased risk for the presence of a visceral cancer
trinucleotide repeat expansion of a region on the 19q chromosome
Submit

A

Correct answer: an increased risk for the presence of a visceral cancer.

This is an example of a case of dermatomyositis. 20-25% of adult patients with this diagnosis also harbor a visceral cancer.

Other answers:

Mutations within the DMD gene are the underlying cause of the most common forms of childhood-onset muscular dystrophy.

Trinucleotide repeat expansion of a region on the 19q chromosome is the underlying cause of myotonic dystrophy.

Although myopathy is the most common complication of statin therapy (1.5% of users), it does not have the associated features (heliotrope rash, pulmonary fibrosis) present in dermatomyositis that were described in the question stem.

108
Q

Your patient, COL Green, is a 57-year-old Caucasian female who has hypertension. She has smoked a pack of cigarettes per day for the past 40 years. After an exercise stress test, you diagnose her with coronary artery disease. You decide to prescribe medication, but you also encourage her to make behavioral modifications. Which of the following behavioral recommendations would most likely promote improvement of long-term cardiovascular health?

quit smoking, abstain from alcohol, and adhere to a strict 1500 kCal diet.
establish a regular exercise regimen, seek a supportive social environment, and quit smoking.
seek a supportive social environment, quit smoking, and decrease intake of foods high in Omega-3 fatty acids.
eliminate stress, quit smoking, and exercise at least 6 days per week for 30-60 minutes.
Submit

A

Correct answer: Establish a regular exercise regimen, seek a supportive social environment, and quit smoking.

Explanation: Engaging in regular exercise, seeking social support, and smoking cessation are all behavioral recommendations that physicians in clinical practice are encouraged to suggest to their patients for the improvement of cardiovascular health.

Other answers:

Omega-3 fatty acids may improve cardiovascular health and, therefore, increased (not decreased) consumption is often encouraged in patients with atherosclerotic heart disease.

While weight maintenance is central to cardiovascular health, there are no empirically supported practice guidelines that patients should adhere a diet of a specific caloric amount.

While stress management contributes to cardiovascular health, elimination of stress is not a feasible recommendation. Second, regular exercise is encouraged, but 6 days per week for 30-60 minutes exceeds the minimum recommended frequency and duration necessary for improvement of cardiovascular health.

109
Q

53 year old woman abruptly develops a severe headache that she describes as “the worst headache she has ever had.” Following this, she rapidly loses consciousness. On reaching the emergency room, a CT scan reveals blood in the subarachnoid space. The most likely underlying cause of her problem is which of the following?

Multiple sclerosis
Thrombotic occlusion of the middle cerebral artery
Rupture of a berry (sacular) aneurysm
Acute bacterial meningitis

A

Correct answer: Rupture of a berry (sacular) aneurysm.

The question stem describes a classic presentation of a ruptured berry aneurysm. Robbins 8ed. pp 1297-8.

Incorrect answers:

Headache may appear in meningitis but it is not apoplectic (abrupt) in onset and is associated with the accumulation of polymorphonuclear cells (pus) in the subarachnoid space (CSF), not blood.

Multiple sclerosis is not associated with headache or blood in the subarachnoid space.

Arterial occlusion leads to ischemic stroke and not subarachnoid hemorrhage.

110
Q

A 23 year-old male is brought to your emergency room following a stab wound. Anatomically, you note a well-demarcated, horizontal, 5 cm laceration just to right of the sternum, at approximately 6 centimeters below the sternal angle (of Louis). His findings include: blood pressure 72/40; heart rate 120/min, regular; distended neck veins; clear lung fields via auscultation; and muffled heart sounds. The most effective immediate treatment for the patient is

local exploration of the laceration to ascertain its depth
sedation and endotracheal intubation
pericardiocentesis
insertion of a right-sided chest tube

A

Correct answer: pericardiocentesis

Explanation: The anatomical location of the stab wound and the presence of Beck’s triad (muffled heart sounds, distended neck veins, and low arterial blood pressure) make cardiac tampanode the most likely diagnosis. In the presence of bleeding from a stab wound to the heart, the tough, fibrous pericardium is unyielding and the pressure around the heart gradually increases as blood leaks from the affected cardiac chamber (usually the right ventricle, due to its anterior, substernal position). This increasing pressure eventually prevents the heart from filling normally and cardiac output falls (hence the drop in blood pressure and cardiac output). The pressure backs up, causing the large veins to dilate (distention of the neck veins), and the increasing layer of blood in the pericardium muffles the heart sounds. This can rapidly cause acute cardiac arrest and death.

The most effective immediate treatment is pericardiocentesis (needle drainage of the pericardium). It is done by careful insertion of a large-bore needle into the pericardial sac and removing the accumulated blood. A catheter can be left in the pericardium to allow continued drainage and to prevent re-accumulation of blood on the way to the OR, where the cardiac wound can be repaired.

A tension pneumothorax can cause decreased blood pressure and would be treated by insertion of a right-sided chest tube. However, lung sounds on the affected side are usually diminished. Other possible signs of tension pneumothorax that are not present in the case include shift of the trachea and of the point of maximal impulse to the unaffected side, and hyper-resonant percussion on the affected side.

111
Q

A patient presenting with polyuria and polydipsia fails to concentrate his urine in response to fluid deprivation or the administration of ADH. Which of the following conditions is most consistent with these observations?

A defective thirst sensing mechanism
Compulsive water drinking
Nephrogenic diabetes insipidus
Absence of renin secretion

A

Correct answer: nephrogenic diabetes insipidus.

The patient has nephrogenic diabetes insipidus. The renal-ADH mechanism is defective and cannot respond to endogenous or exogenous ADH

Incorrect answers:

The ADH system is functional in individuals with compulsive water drinking.

While affecting salt and water balance, the absence of renin does not inhibit the functions of the ADH system

While affecting salt and water balance, a defective thirst mechanism does not inhibit the functions of the ADH system

112
Q

An 86-year-old retired Rear Admiral is brought to the clinic by his daughter. She says that he keep asking her the same questions, over and over again. He has also had difficulty remembering the names of his grandchildren. She has noted this problem over the past 3 years and reports that it has been getting worse. Recently, he went out for a walk and became lost and could not find his way home. On examination, he appears confused and is unable to recall the day, the season, or the current year. Otherwise, neurologic examination is normal. MRI scan shows symmetrical enlargement of the lateral ventricles with some generalized mild atrophy of the cerebral cortex. There are no other structural abnormalities noted. Which of the following is most likely?

His brain demonstrates plaque-like accumulation of beta-amyloid protein and intraneuronal accumulation of the protein tau
He has a genetically determined disorder characterized by loss of medium spiny neurons in the basal ganglia
He demonstrates degeneration and loss of both upper and lower motor neurons
He has a mutation of the PRNP gene

A

The correct answer is His brain demonstrates plaque-like accumulation of beta-amyloid protein and intraneuronal accumulation of the protein tau.

Alzheimer’s disease is the most common cause of dementia in this age group. Histology demonstrates plaque-like accumulation of beta-amyloid protein and intraneuronal accumulation of the protein tau.

Incorrect answers:

Prion-related disorders which are caused by PRNP gene mutations are extremely rare conditions and produce much more rapid neurologic deterioration.

Loss of upper and lower neurons is indicative of amyotrophic lateral sclerosis, which is characterized by progressive weakness rather than dementia.

The hereditary disorder with loss of medium spiny neurons in the caudate nucleus is Huntington’s disease. It is characterized by prominent choreiform movements and has an onset at a much younger age.

Robbins PBOD 8ed. pp 1313-1317

113
Q

A 59-year-old female presented with a 7-month history of cough, dyspnea, cyanosis and marked dilatation of the veins of her head, neck and arms. Sometimes, especially when drinking liquids, she would choke. She also complained of feeling weak toward the end of the day. Chest x-ray revealed an anterior mediastinal mass. The patient’s clinical history are most consistent with which neoplasm:

Acute lymphoblastic leukemia
thymoma
Hodgkin disease
Lung adenocarcinoma

A

Correct answer: thymoma.

The patient’s clinical history and chest radiographic appearances are suggestive of neoplastic involvement of the mediastinum with either pressure effects or infiltration of adjacent mediastinal structures such as mainstem bronchi, esophagus and superior vena cava. The distension of the venous return in the head, neck and arms is indicative of the superior vena cava syndrome. Of the possibilities listed, thymoma and lung adenocarcinoma would be most likely to produce these clinical features, although the other two conditions may rarely do so. The histologic appearances of lymphocytes (thymocytes) admixed with epithelial cells having bland round, vesicular nuclei resembling normal thymic cortical epithelial cells are classic for thymoma. The patient’s weakness toward the end of the day is consistent with systemic myasthenia gravis, a well-known disease association with thymoma.

Other answers: Nodular sclerosing Hodgkin disease typically involves the mediastinal lymph nodes of young adults and is characterized histologically by dense bands of collagen separating a mixed pleomorphic infiltrate composed of lymphocytes, eosinophils, histiocytes and lacunar variant Reed Sternberg cells with giant multilobed nuclei.

Absence of gland formation by the tumor cells excludes a diagnosis of lung adenocarcinoma, and myasthenia gravis is not one of the paraneoplastic syndromes associated with lung adenocarcinoma.

Acute lymphoblastic leukemia is primarily a disease of children and young adults in which the leukemic infiltrates comprise a monotonous, monomorphic population of leukemic blast cells.

Robbins PBOD 8ed. pp 636-637

114
Q

A 6-month-old boy presents unable to crawl and with poor ability to sit up unsupported. The mother reports finding what looked like orange sand in his diaper. Chemical analysis showed the material consisted mainly of uric acid colored by small amounts of blood. Cultured fibroblasts showed normal rates of purine biosynthesis but a decreased ability to reutilize purine bases. What enzyme is most likely deficient in this patient?

Xanthine oxidase
PRPP synthetase
Hypoxanthine-guanine phosphoribosyltransferase
Glutamine phosphoribosyl amidotransferase
Submit

A

The presence of uric acid crystals in urine (“sand” in the diaper) indicates extremely high levels of uric acid are being excreted. The motor disability and hypotonia evident from the clinical picture taken together with the lab findings suggest Lesch-Nyhan disorder – the condition in which the purine salvage enzyme hypoxanthine-guanine phosphoribosyltransferase (HGPRT) is lacking entirely. The bizarre self-injurious behavior associated with this disorder manifests later on in childhood. Without functional salvage activity, purine bases are destined to be degraded to uric acid. The resulting decreased nucleotide pools together with increased PRPP levels lead to loss of feedback inhibition and dysregulation of the de novo synthesis pathway. This causes even higher levels of purines to be produced, which can go nowhere else but to form uric acid.

Xanthine oxidase produces uric acid (converts the bases hypoxanthine to xanthine, and xanthine to uric acid), so a deficiency of that enzyme would not lead to increased uric acid. PRPP synthetase and the phosphoribosyl amidotransferase enzyme (aka PRA synthase) are involved in de novo synthesis of purines, so a deficiency is likewise unlikely to cause uric acid overproduction.

115
Q

Gene therapy for Kartagener’s syndrome, a type of primary ciliary dsykinesia, would correct the defect in the gene encoding

myosin
dynein
collagen
actin

A

Dynein is a motor protein that is associated with microtubules and is found in cilia. The other proteins listed are not found in cilia.

Individuals with primary ciliary dyskinesia often have genetic mutations in genes encoding dynein. Patients with primary ciliary dyskinesia often have poor mucociliary clearance, and thus are susceptible to recurrent respiratory infections. Males with this disease often present with infertility because of immotile sperm. Kartagener’s syndrome is the combination of recurrent respiratory infection due to primary ciliary dsykinesia and situs inversus (left to right reversal of organs).

116
Q

A full-term newborn male presents with intermittent seizures, acidosis, muscular hypotonia, horizontal gaze paralysis, and spasticity in both legs. His condition was found to result from an X-linked recessive mutation in the alpha subunit of E1 of the pyruvate dehydrogenase complex, a type of Leigh’s disease. Which one of the following metabolic patterns can be predicted for patients with this disease?

Increased ATP/ADP ratio in muscle
Increased plasma lactate/pyruvate ratio
Increased rate of oxygen consumption in brain
Decreased rate of ketone body utilization in brain

A

Answer: increased plasma lactate/pyruvate ratio

The pyruvate dehydrogenase complex (PDC) is located in the mitochondrial matrix and functions to convert pyruvate to acetyl-CoA for subsequent entry into the TCA cycle in aerobic metabolism. Deficiency of PDC results in increased production of lactic acid in order to maintain flux through glycolysis for ATP production. Absolute levels of lactate increase, as does the lactate/pyruvate ratio. Defects in this enzyme (or for that matter, any other enzyme required for energy production) would not be predicted to cause increased levels of ATP or an increase in the ATP/ADP ratio. Cellular oxygen consumption would be expected to decrease or remain largely unaffected because part of the acetyl-CoA, that amount coming from carbohydrates via glycolysis, is no longer available as fuel for oxidation and consumption of oxygen. The brain has limited ability to utilize fatty acids for oxidation, and relies heavily on glucose and ketone bodies for energy. A defect in PDC reduces the amount of energy available from aerobic oxidation of carbohydrates and the brain is then forced to rely more heavily on ketone bodies for energy. Indeed, ketogenic diets have been found to be highly beneficial to control neurologic symptoms in patients with PDC deficiency. However, such diets appear to be less effective in cases where defects exist in other OXPHOS enzymes, such as those directly involved in the electron transport chain, because at later stages the ability to derive energy is impaired for all fuel sources.

117
Q

A 26-year-old primigravid woman delivered a stillborn male fetus. The upper portion of the cranial vault and cerebral hemispheres were absent. The brainstem and cerebellum were fully exposed. Which developmental defect most likely caused this condition?

Agenesis of the corpus callosum
Aberrant opening of the fourth ventricle
Failed closure of the neural tube
Lack of migration of neural crest cells

A

The defect described is anencephaly, which is due to failure of the rostral (anterior) portion of the neural tube to close. Neural tube closure normally occurs near the end of the fourth week of development.

Other answers:

Following formation of the neural tube, a region of the upper medulla & pons opens dorsally to form the forth ventricle. A defect in this process would affect the development of the brainstem and cerebellum, but not the higher regions of the brain.

Agenesis of the corpus callosum results in complete or partial absence of the corpus callosum. This is a very rare congenital abnormality with multiple causes that is not typically a lethal birth defect.

Neural crest cells detach from the neural tube as it closes and migrate away to form much of the peripheral nervous system and a number of non-neural structures (eg facial skeleton & pigment cells). Defects in this process can result in cleft lip, albinism, and underdevelopment of sensory and autonomic neurons in the PNS, but do not affect development of the CNS.

118
Q

Casey Studdie is a 16-year-old female. Since age 14 months she has experienced recurrent episodes of profound fatigue associated with vomiting and increased perspiration, which required hospitalization. These episodes occur only after fasting for more than 8 hours. Because her mother gave her food late at night and woke her early in the morning for breakfast, Casey’s physical and mental development have progressed normally.

On the day of admission for this episode, Casey had missed breakfast and by noon was extremely fatigued, nauseated, sweaty, and limp. She was unable to hold any food in her stomach and was rushed to the hospital where an infusion of glucose was started intravenously. Her symptoms responded dramatically to this therapy. Her initial serum glucose level was low at 38 mg/dL (reference range = 80–100). Her blood urea nitrogen (BUN) level was slightly elevated at 26 mg/dL (reference range = 8–25) as a result of vomiting, which led to a degree of volume depletion. Her blood levels of liver transaminases were slightly elevated, although her liver was not palpably enlarged.

Despite elevated levels of free fatty acids (4.3 mM) in the blood and hypoglycemia , blood ketone bodies were below normal.

After reviewing her previous hospital records, a specialist suspected that Casey’s medical problems were caused by a metabolic disorder. A battery of tests showed that Casey’s blood contained elevated levels of octanoic acid (8:0) and 4-decenoic acid (10:1,Δ4). A urine specimen showed an increase in organic acid metabolites containing from 6 to 10 carbons, including their acylcarnitine derivatives, and dicarboxylic acids.

What is the most likely site of the metabolic defect?

Mixed function oxidases of the smooth endoplasmic reticulum
Oxidation of long chain fatty acids to medium chain length fatty acids (mitochondria)
Oxidation of medium chain length fatty acids to short chain fatty acids (mitochondria)
Transport of long chain fatty acids into the mitochondrial matrix

A

This patient most likely has Medium Chain Acyl CoA Dehydrogenase Deficiency (MCADD). This is a dysfunction of fatty acid metabolism. Fatty acid oxidation by mitochondria is important for energy production, especially during times of fasting. Failure to use fatty acids efficiently as an energy source during the fasting state forces cells to rely more heavily than usual on glucose metabolism, which probably contributes to hypoglycemia. Thus, individuals with fatty acid metabolism disorders often present fatigues and with a change in mentation during times of fasting or low carbohydrate intake.

MCADD is a relatively common inborn error, and accounts for all the findings in the case. The accumulation of octanoic acid and 4-decenoic acid is consistent with a block in the beta-oxidation of medium chain fatty acids. Accumulated carboxylic acids are often excreted as their carnitine derivatives, so these derivatives are expected in MCADD. Fatty acids that cannot be oxidized by the mitochondria are treated as toxic materials and are metabolized by the omega oxidation pathway of the SER to dicarboxylic acids, so these metabolites are also expected to appear in the blood and urine. The omega pathway involves mixed function oxidation of the terminal methyl carbon of the fatty acyl chain.

119
Q

Your patient is a 13-year-old female. Her mother describes 3 separate short episodes of unresponsiveness, which were associated with finger tapping in the patient’s left hand. The patient describes the episodes as “blacking out, then ‘waking up” feeling exhausted and confused”. Which of the following CORRECTLY pairs her seizure type with a DRUG OF CHOICE for treatment?

absence - phenytoin
complex partial - valproate
complex partial - felbamate
absence - ethosuximide

A

The correct answer is complex partial - valproate.

Explanation: The impaired consciousness with postictal abnormalities is indicative of a complex partial seizure. Valproate can be used as initial treatment for complex partial seizure. Felbamate is used only for refractory seizures due to the high risk of severe adverse effects and ethosuximde is not indicated for complex partial seizures.

120
Q

An 80 year old retired Admiral with a history of dementia due to Alzheimer’s disease is admitted to your hospital service with fever, shortness of breath, and cough. On physical exam, he is lethargic, but arousable. His temperature is 103 degrees F, BP is 100/60, and pulse is 86 (without “tilt”/no orthostasis). Respirations are 21/minute. His jugular venous pressure is 6 cm of water, he has no edema, and his abdomen is benign. Cardiac exam reveals a regular rate and rhythm and his pulmonary examination is notable for crackles in his left lower lung field. On abdominal examination, he has decreased bowel sounds without tenderness or masses. Strength is 4+/5 in all extremities.

Admission lab studies are as follows:
Serum: sodium 118 mEq/L; potassium 3.5 mEq/L; chloride 84 mEq/L; bicarbonate 22 mEq/L; glucose 80 mg/dl; BUN 10 mg/dl; creatinine 0.8 mg/dl; osmolality 238 mOsm/L.
Urine: sodium 44 mEq/L; osmolality 250 mOsm/L

CXR: Left lower lobe infiltrate

Which of the following is most likely to account for his hyponatremia?

Intravascular volume depletion
Acute water intoxication due to dementia
SIADH
Pseudohyponatremia due to severe paraproteinemia from multiple myeloma

A

Answer: SIADH

The elder gentleman described in this vignette has pneumonia, but appears euvolemic, based on “tilt negative” BP and pulse and jugular venous pressure of 6 cm of water. The pulmonary crackles are most likely secondary to the pneumonic process. He has true hypo-osmolar hyponatremia, because the measured serum osmolality of 238 mOsm/L is close to the calculated serum osmolality of 236 mOsm/L. Relative to his very low serum osmolality, his urine osmolality is inappropriately high, at 250 mOsm/L (should be able to get it down to at least 100 mOsm/L). Additionally, his high urinary sodium level of 44 mEq/L suggests either euvolemia or diuretic therapy. Thus, the syndrome of euvolemic hypoosmolar hyponatremia fits here. The most likely diagnosis is the syndrome of inappropriate antidiuresis (SIAD). As a purist, you would need to check to make sure his adrenal and thyroid function are normal, as adrenal insufficiency and hypothyroidism can look like this. Be leery of this diagnosis among diuretic users, as well! The clinical setting is a clue, because SIAD is prominently associated with intracranial disease (including dementia) and pulmonary disease (including pneumonia).

121
Q

Of the following hormones, which one is most likely to act through a mechanism that involves increased binding of its receptors to DNA?

 luteinizing hormone
 melatonin
 insulin
 corticotropin releasing hormone
 progesterone
A

Correct answer: progesterone.

Progesterone, a steroid hormone, is lipid soluble and readily diffuses through the cell membrane into target cells where it acts in the nucleus, via nuclear receptors, to regulate gene expression. The rest of the choices are water soluble hormones that are excluded by the cell plasma membrane and therefore act on cell surface receptors.

122
Q

Ninety-five percent of pancreatic cancers are due to mutations in Ras which result in a loss of signal transduction termination. Which Ras function do these mutations commonly disrupt?

The ability of Ras to bind the Epidermal Growth Factor Receptor.
The ability of Ras to hydrolyze GTP to GDP.
The ability of Ras to bind to Raf (the MAPKKK).
The ability of Ras to dissociate from its beta and gamma subunits.

A

Answer: the ability of Ras to hydrolyze GTP to GDP

Ras is a monomeric G-protein that commonly stimulates MAP kinase signaling cascades often leading to unregulated cell division and tumor growth. When Ras is bound to GDP, it is inactive and cannot transduce signal. When it is bound to GTP, its protein conformation changes and it can bind to Raf to stimulate the signaling cascade. Ras is a “molecular clock” in that is has an intrinsic enzymatic activity that hydrolyzes GTP to GDP, thus terminating the signal. In many cancers, there are mutations that disrupt this enzymatic activity and thus the MAP kinase signaling cascade is unregulated and always on. These mutations do not disrupt any of the other functions listed.

123
Q

Your patient is a 13-month-old female with a history of infections and failure to thrive. She has been diagnosed with the rare inherited immunodeficiency disease called “Bare Lymphocyte Syndrome,” whose biochemical basis is failure of cells to express Class II MHC molecules. This failure is MOST LIKELY to DIRECTLY interfere with:

Killing of bacteria living in macrophage phagolysosomes
Production of cell surface IgM by B cells
Phagocytosis of antigen by macrophages
Killing of virus-infected cells by cytotoxic lymphoctyes

A

Answer: killing of bacteria living in macrophage phagolysosomes

Explanation: Failure to express class II MHC molecules will prevent the thymic development of CD4+ T cells, a primary function of which is to provide T cell help for the killing of phagocytosed bacteria by macrophages.

Incorrect answers:

Cytotoxic lymphocytes are derived from CD8+ T cells, which do not require class II MHC for their development.

While the failure to express class II MHC results in a loss of T cell help for the killing of phagocytosed bacteria, the ability of macrophages to phagocytose antigens is not affected.

While the failure to express class II MHC results in a loss of T cell help, B cells can produce IgM in the absence of T cell help.

124
Q

A 19-year old African American male suffered an acute bout of red blood cell hemolysis after taking sulfa drugs to treat a bacterial infection. Which of the following is the likely cause of the hemolytic reaction from the sulfa drugs?

Lack of NADPH production due to an inborn error of glucose 6-phosphate dehydrogenase
ATP depletion due to rapid destruction of NAD+ by the sulfa drug
Glutathione depletion due to inhibition of glutathione synthetase
Lack of transketolase activity due to drug-induced thiamine depletion

A

Correct answer: lack of NADPH production due to an inborn error of glucose 6-phosphate dehydrogenase

Glucose 6-phosphate dehydrogenase deficiency is a common inborn error in the African/African American male population. One of the products of the glucose 6-phosphate dehydrogenase reaction is NADPH, which is necessary to provide the reducing equivalents to reduce oxidized glutathione to reduced glutathione. Reduced glutathione is necessary to eliminate reactive oxygen species (ROS) that are formed in several reactions. High levels of ROS are generated by the metabolism of several drugs, notably antimalarials, but also some antibiotics. Normal individuals have a large excess of the enzyme glucose 6-phosphate dehydrogenase to cope with oxidative emergencies, but individuals with the deficiency are living on the edge, with an activity level about 10% of normal. Their deficiency manifests in situations of unusually high ROS production.

Incorrect answers:

One cannot rule out an unusual reaction in which a drug destroys thiamine. However, thiamine depletion would cause many more symptoms than red-cell hemolysis, notably neurological symptoms, as seen in some alcoholics suffering from thiamine deficiency.

“ATP depletion due to rapid destruction of NAD+ by the sulfa drug” is incorrect because the pentose pathway does not produce ATP.

The answer “Glutathione depletion due to inhibition of glutathione synthetase” cannot be ruled out from first principles, but the clinical presentation is classic for G6PDH’ase deficiency, so this is the most likely cause of the hemolysis (i.e., pick “horses” over “zebras.”)

125
Q

A 24-year-old medical student who recently began his clinical rotations comes to see you at the behest of your surgical colleague who is concerned about his wellbeing. The student recently began his surgical rotation, which he finds exhilarating. He reports staying up all hours of the night for the past week to prepare for surgical cases the next day. He feels it is extremely important for him to expand his knowledge base so he can impress his surgical mentors; much more important than sleep! During the appointment, he appears easily distractible and quite concerned that his surgical team needs his immediate assistance for a case right now. He mentions he has strong religious beliefs. He feels he’s “at the top of his game” and recently bought an expensive car to celebrate his success. This comes in stark contrast to his description of his mood during the preceding 3 months, which he describes as his “low point.” What is the most likely diagnosis in this patient?

Hyperthyroidism
Major depressive disorder
Schizophrenia, paranoid type
Bipolar disorder

A

Correct answer: bipolar disorder

This patient is demonstrating classic symptoms of bipolar disorder. The syndrome is typified by periods of mania or hypomania and depression. This patient is currently in a manic episode, evidenced by a week of no sleep and extreme focus. Oftentimes the patients feel they are performing very well and do not appreciate the diagnosis until the depressive symptoms return. Mania is not a feature of major depressive disorder. Although he states he has strong religious affiliations, he does not have hallucinations common with schizophrenia. While hyperthyroidism can be associated with decreased sleep, it is more commonly associated with tremor, palpitations and heat intolerance.

126
Q

A 72-year-old Vietnam War veteran presents to the hospital with an exacerbation of chronic obstructive pulmonary disease triggered by bronchitis. The patient is started on inhaled bronchodilators and systemic corticosteroids. After initially improving for 2 days he suddenly becomes hypotensive, requiring admission to the ICU for intravenous vasopressors to maintain his blood pressure. Blood cultures grow E. coli after 24 hours. You suspect the sepsis represents:

Exacerbation of chronic Stronglyoides stercoralis infection following steroid therapy
Exacerbation of chronic Entamoeba histolytica infection following steroid therapy
Exacerbation of chronic Schistosoma mansoni infection following steroid therapy
Exacerbation of chronic Ascaris lumbricoides infection following steroid therapy

A

The correct answer is exacerbation of chronic Stronglyoides stercoralis infection following steroid therapy.

Of the agents listed S. stercoralis helminths are known to persist for long periods of time within human hosts due to autoinfection (the intermittent penetration of L1 larvae through the intestinal wall that restarts the infection cycle). These larvae enter the circulation, become trapped in the lungs, migrate up the trachea, and are swallowed to initiate new intestinal infections. With reduced immunity attributable to steroid therapy, the autoinfection cycle proceeds unchecked. Bacterial sepsis results from the concomitant release of bowel flora during penetration of large numbers of strongyloides larvae through the intestinal walls.

127
Q

A 60-year-old woman is suffering from stiffness in her wrists and knees that is particularly painful after exercise. Her joints are swollen and a blood test reveals that she is positive for rheumatoid factor. You decide to prescribe a disease modifying anti-rheumatic drug. However, you notice from her medical history that she has a latent TB infection. Which of the following is the BEST disease modifying anti-rheumatic drug to treat her rheumatoid arthritis?

Methotrexate
Infliximab
Ibuprofen
Etanercept

A

Correct answer: methotrexate.

Explanation: The best treatment for rheumatoid arthritis is a drug that does more than relieve symptoms. Ibuprofen is often used to relieve symptoms, but is not a disease modifying anti-rheumatic drug (DMARD). Etanercept and infliximab are DMARDs, but they are “biologics” which specifically inhibit TNF-alpha action. It is known that treatment with these biologic drugs will frequently lead to reactivation of latent TB, if present. Therefore neither of these drugs should be used in a patient with latent TB. Therefore the best answer is methotrexate, a DMARD that does not have this side effect.

128
Q

A 19-year old African American male presents with a one day history of sudden onset of chest pain, cough, and shortness of breath while performing reservist duty in the Washington, D. C. area. He has a medical history of anemia and recurrent episodes of “chest symptoms” leading to recurrent hospitalizations. The current episode is typical of prior episodes leading to hospitalization. The peripheral blood smear shows a large number of immature cells and many long crescent-shaped erythrocytes typical of sickle-cell anemia. The molecular basis of the disease has been traced back to a single amino acid substitution (Glu to Val) in the primary amino acid sequence of the b-globin gene product, which leads to abnormal aggregation of the hemoglobin molecules through intermolecular interactions. This aggregation results from increased:

Hydrogen bonding
Disulfide bond formation
Ionic interactions
Hydrophobic interactions

A

Correct answer: hydrophobic interactions

The side chain of Glu is charged whereas that of Val is uncharged and nonpolar. Therefore the change from Glu to Val favors hydrophobic interactions (choice “d”). Since H-bonding requires two polar moieties, Val cannot facilitate such an interaction, ruling out hydrogen bonding. Similarly, Val, having a neutral side chain, cannot take part in ionic interaction, ruling out ionic interactions. Disulfide bond formation is not a correct answer because the disulfide bond is a specific covalent linkage between two adjacent cysteine side-chains.

129
Q

During surgical removal of retropharyngeal lymph nodes, a patient’s superior cervical ganglion was inadvertently damaged. Which complication is MOST LIKELY to be observed on the side of the damaged ganglion?

Excessive sweating on the face
Absence of pupillary light reflex
Hyposalivation
Anisocoria

A

Answer: anisocoria

Damage to the superior cervical ganglion disrupts sympathetic outflow to the head region, causing Horner’s syndrome (ptosis – drooping of the eyelid, miosis – constricted pupil, and anhidrosis – decreased sweating). The pupil on the ipsilateral side would be smaller than the other pupil due to loss of sympathetic input to the dilator puppillae. Since anisocoria means unequal pupils, anisocoria is the correct answer. The light reflex would be unaffected because it is mediated by CNIII. Salivation (CN IX & VII) and lacrimation (CN VII) also would not be affected. Sweating on the face would be reduced , not increased, by the lesion.

130
Q

In a patient with mitral stenosis, flow of blood into the left ventricle is limited, thus limiting stroke volume and cardiac output. You would expect that this patient’s

maximal heart rate is achieved at lower workloads than normal.
resting heart rate is lower than normal.
maximal arterial-venous oxygen difference is higher than normal.
exercise capacity is as high as a trained athlete.

A

The correct answer is, “maximal heart rate is achieved at lower workloads than normal.”

Explanation: Patients with mitral stenosis have a low stroke volume at rest. They compensate for this by having an increased heart rate. This adjustment at resting levels leaves less reserve to compensate for increased demand during exercise. Consequently, heart rate will reach maximum at lower workloads in subjects with mitral stenosis relative to individuals without mitral stenosis.

Incorrect answers

“Maximal arterial-venous oxygen difference is higher than normal.” Maximal a-v O2 difference is limited by how much O2 can be extracted from blood. Arterial O2 content cannot be increased. So the drop in venous O2 content limits the amount extracted. In a normal person maximum extraction is reached at about 75% oxygen saturation. The body has difficulty extracting more than that. Although mitral stenosis subjects may compensate for the low stroke volume at rest by having an increased a-v O2 difference, the maximum difference that can be achieved with an increasing workload is similar to normal subjects but achieved at a lower workload than normal.

“Exercise capacity is as high as a trained athlete.” Trained athletes can achieve a higher maximal cardiac output than normal primarily due to a lower resting heart rate. In a mitral stenosis subject, resting heart rate is even higher than normal in order to counterbalance the inability to increase stroke volume as a way of maintaining cardiac output in a normal range. Consequently, maximal heart rate will then be achieved at a lower workload in these subjects and therefore cardiac output will also be maximal at a lower workload. Since the patient’s cardiac output is compromised, exercise capacity will be limited.

“Resting heart rate is lower than normal.” Since stroke volume is lower than normal, a lower heart rate would further lower cardiac output. The patient probably has a heart rate above normal.

131
Q

Some of the most devastating inborn errors in metabolism result from single gene mutations that lead to the excessive accumulation of sphingolipids. Because these lipids are abundant in the nervous system, these diseases result in severe psycho-motor retardation and early lethality. The common biochemical defect that underlies the pathology associated with this class of lipid storage diseases (such as Tay-Sachs, Niemann-Pick, and Sandoff’s) is a decreased activity of

lysosomal hydrolases
serine palmitoyltransferase
glycosyltransferases
fatty acid desaturase

A

Answer: lysosomal hydrolases

The sphingolipidoses are a subset of the lysosomal storage diseases; in each case a hydrolytic enzyme required for degradation of one or more of the sphingolipids is defective. Consequently the substrate(s) of the defective lysosomal hydrolase accumulate and cause severe damage, especially when excessive storage occurs in the nervous system. The other enzymes are involved in lipid synthesis, not degradation, and are not located in the lysosome.

132
Q

The ability of 1,25-(OH)2vitamin D3 to protect against hypocalcemia and osteoporosis results primarily from its action on the

parathyroid glands to increase the secretion of parathyroid hormone.
thyroid to increase the secretion of calcitonin.
intestine to increase the absorption of calcium.
kidney to increase the reabsorption of calcium.

A

Correct answer: intestine to increase the absorption of calcium.

Explanation: The intestine is an important site for 1,25-(OH)2vitamineD3, bioactive vitamine D.

Incorrect answers.

PTH, but not 1,25-(OH)2vitamin D3, bioactive vitamin D, causes the kidney to increase the reabsorption of calcium.

The parathyroid senses calcium and responds to hypocalcemia, not 1,25-(OH)2vitamin D3

The calcitonin secreting cells within the thyroid senses calcium and respond to hypercalcemia, not 1,25-(OH)2vitamin D3, bioactive vitamin D

133
Q

A patient with anorexia nervosa presented to her physician’s office with a chief complaint of becoming easily fatigued. She is 5 feet 7 inches tall and has a body weight of 85 lbs, less than 65% of her ideal weight. The patient was then hospitalized with a diagnosis of severe malnutrition (marasmus) secondary to anorexia nervosa. Clinical findings included decreased body core temperature, blood pressure, and pulse.

Admission laboratory studies revealed a blood glucose level of 65 mg/dL (normal fasting blood glucose = 80–100). Serum ketone body concentration was 4,200 μM (normal = about 70).

Although the patient’s blood glucose is below the normal fasting range , she is experiencing only a moderate degree of hypoglycemic symptoms despite her severe, near starvation diet.

Which set of liver enzymes is induced or activated in her condition, allowing her to maintain blood glucose levels?

Phosphoenolpyruvate carboxykinase; Fructose -1,6- bis phosphatase; Glucose 6-phosphatase
Phosphodiesterase; Protein phosphatase-2; Glycogen synthase
Acetyl CoA carboxylase; Pyruvate dehydrogenase; Citrate lyase
Glucokinase; Phosphofructokinase-1; Pyruvate kinase

A

Correct answer: The patient is experiencing fatigue for a number of reasons. These include iron deficiency anemia and deficiencies in vitamins such as thiamine, riboflavin, and niacin. It is less likely, but possible, that she also has subclinical deficiencies of pantothenate or biotin.

Phosphoenolpyruvate carboxykinase; Fructose -1,6- bis phosphatase; and Glucose 6-phosphatase are enzymes of the gluconeogenic pathway in liver, an essential pathway to maintain blood glucose in a fasted state. They are activated and/or induced by counter-regulatory hormones, especially glucagon.

Incorrect answers:

Glucokinase; Phosphofructokinase-1; Pyruvate kinase: These enzymes are active in the well-fed state, and activated by insulin. They take glucose out of the blood. Glucokinase traps glucose in the cell as glucose 6-phosphate. The other two enzymes are part of the glycolytic pathway, which opposes gluconeogenesis. Glycolysis is switched off when the gluconeogenic pathway is active.

Phosphodiesterase; Protein phosphatase-2; Glycogen synthase: These enzymes iare activated by insulin, and reverse the effects of glucagon on the cAMP signaling cascade. They are active in the well fed state, not the fasted state.

Acetyl CoA carboxylase; Pyruvate dehydrogenase; Citrate lyase: These enzymes are also activated by insulin. Pyruvate dehydrogenase is activated when glucose is being used for energy or for fatty acid biosynthesis. Acetyl CoA carboxylase and citrate lyase are part of the pathways for synthesis of fatty acids from glucose, relevant only in the well-fed state.

134
Q

Multiple sclerosis (MS) is a disease that attacks myelin in the central nervous system (CNS). Clinical symptoms of MS include blurred or double vision, muscle weakness in the extremities, and difficulty with coordination and balance.

The cells that synthesize myelin in the CNS and are destroyed in MS are

Astrocytes
Schwann cells
Microglia
Oligodendroglia

A

Explanation: Oligodendroglia are the only cells in the CNS that synthesize myelin.

Incorrect answers:

Astrocytes provide physical and metabolic support for neurons of the central nervous system.

Microglia are phagocytic cells found in the central nervous system.

Schwann cells synthesize myelin in the peripheral nervous system.

135
Q

Your routine physical examination of a 53 year old man reveals an irregular, firm, 3.5 cm diameter, non-fixed ulcer proximal to the anal verge. Your anoscopy shows that it is just distal to the dentate line and your biopsy confirms a squamous cell carcinoma. Colonoscopy and other screenings are negative. Anatomically, an area with a high likelihood of metastases from this lesion is the

superficial inguinal lymph nodes
the right hepatic lobe
internal iliac lymph nodes
perineal nodes

A

Correct answer: superficial inguinal lymph nodes

Explanation: The dentate line (also called the pectinate line or mucocutaneous line) demarcates the limit of the anal valves and can be visualized within the anal canal. It is an important landmark and indicates the junction of the superior (visceral) part of the anal canal (from the embryonic hindgut) and the inferior (somatic) part (from the embryonic proctodeum). Above and below this line, the arterial supply, innervation, and venous and lymphatic drainage differ dramatically. Inferior to the dentate line (towards the perineum), the lymphatics (like most of the rest of the perineum) characteristically drain into the superficial inguinal lymph nodes and one must adopt palpation of the inguinal lymph nodes as a routine examination for any lesion in this area. If the nodes are found to be positive histologically for metastases, it will have an important bearing on the accurate staging of the lesion. This may then impact on treatment decisions.

136
Q

You are on duty when a foreign soldier is brought to you after sustaining multiple extremity wounds due to an IED blast. His heart rate is 110 beats per minute and his blood pressure 95/65 mmHg. His hematocrit is 35 (normal 40-45). The ambulance tech estimates he has lost nearly 1 liter of blood. The normal mean arterial blood pressure, in this patient, was 92 mmHg and is now 75 mmHg. Blood flow to the brain is

decreased due to reflex vasoconstriction
maintained constant due to baroreceptor mediated cerebrovascular dilation
maintained constant due to local mechanisms
maintained constant due to reflex cerebrovascular constriction

A

Correct answer: maintained constant due to local mechanisms.

Explanation: Autoregulation of blood flow is local mechanism, the ability of a tissue to maintain constant blood flow in the face of a change in perfusion pressure. Blood pressure and thereby perfusion pressure is down to about 75 mmHg. The normally healthy brain can maintain blood flow to a perfusion pressure of about 50 mmHg.

Incorrect answers:

“decreased due to reflex vasoconstriction” Blood flow is maintained. The brain’s vasculature is relatively insensitive to the influence of nerves.

“maintained constant due to reflex cerebrovascular constriction” Vasoconstriction would cause an increase in vascular resistance and decrease flow.

“maintained constant due to baroreceptor mediated cerebrovascular dilation” The brain’s vasculatureis relatively insensitive to the influence of nerves. Autoregulation is a function of the tissue itself.

Nerves do not participate in local autoregulation.

137
Q

A likely way in which a localized adenocarcinoma of the rectosigmoid colon (junctional region between the sigmoid colon and the rectum) can metastasize to the liver is via

transport through lymphatic vessels draining to the superficial inguinal nodes
direct extension into the left peritoneal gutter
transport through through the vena caval venous system
transport through the portal venous system
Submit

A

Correct answer: transport through the portal venous system

Explanation: Of the choices, the most likely route is a blood-borne metastasis through the portal system: i.e., from the superior rectal vein into the inferior mesenteric vein, and thence directly to the liver.

Incorrect answers:

The tributaries of the vena caval system correspond to the paired visceral and parietal branches of the abdominal aorta and do not classically drain the distal colon. Rather, the unpaired viscera (like the rectosigmoid colon) are drained by tributaries of the portal venous system, which ends in the liver.

The superficial inguinal nodes characteristically drain the skin and its adnexae from the part of the perineum that is inferior to the pectinate line of the anal canal. Thus, superficial inguinal nodal metastases are sometimes seen with transitional cell carcinomas of the anus – but not usually with adenocarcinomas of the rectosigmoid.

Regarding the left peritoneal gutter: Although the descending colon is retroperitoneal and is covered anteriorly by the peritoneum of the left peritoneal gutter, the sigmoid colon is intraperitoneal and distal to this region. Although a direct extension of a rectosigmoid carcinoma could conceivably reach the left peritoneal gutter, this is usually a late finding and a blood-borne metastasis via the portal system is much more likely.

138
Q

A 26-year old woman was brought to the emergency room in a coma after her husband could not wake her from an afternoon nap. He reported that his wife had been feeling nauseated and drowsy and had been vomiting for 24 hours. She is clinically dehydrated and her blood pressure is low. Her respirationsare deep and rapid, and her pulse rate is rapid. Her breath has the “fruity” odor of acetone. Arterial blood samples show a pH = 7.02 (7.36 – 7.44), PaCO2 = 28 mm Hg (37 – 43 mm Hg) , [HCO3- ] = 8 mM (24 -28). Her blood glucose is 648 mg/dL (80 – 110), and ketone bodies are present in both blood and urine. She is diagnosed with diabetic ketoacidosis, and successfully treated with administration of fluids, electrolytes, and insulin. Further testing shows that she has Type I diabetes. Dysregulation of which step is thought to be most important in the over production of ketone bodies in the Type I diabetic?

Formation of ketone bodies from acetyl CoA in the liver mitochondrial matrix
Beta oxidation of fatty acids to acetyl CoA in the liver mitochondrial matrix
Hydrolysis of triglyceride to diglycerides and fatty acid in adipose tissue
Transport of fatty acids across the liver mitochondrial inner membrane

A

Answer: Hydrolysis of triglyceride to diglycerides and fatty acid in adipose tissue

Uncontrolled Type I diabetics have high levels of fatty acids in their blood owing to lack of control of the first step in fat mobilization, hydrolysis of triglycerides to diglycerides and fatty acids in adipose tissue. The enzyme that catalyzes this step is aptly named hormone-sensitive-lipase. It is activated by counter-regulatory hormones such as glucagon and epinephrine, via the cAMP signalling cascade. Insulin reverses the effects of these hormones., slowing down or blocking fat mobilization. Ordinarily, even in a fasted state, the rate of the lipase reaction is controlled by a balance of opposing effects of insulin and counter-regulatory hormones. In the absence of insulin, the large flux of fatty acids into the blood and into the liver causes over production of ketones by simple mass action, i.e., by providing a large amount of substrate for the relevant pathways. Insulin inhibits one other step in the list shown above – transport of fatty acids across the liver mitochondrial matrix (via the carnitine transport system). However, even in a normal individual, with basal insulin, the transport system is operating at maximal velocity in the fasted state.

139
Q

You are on duty when a soldier is brought to you after sustaining traumatic injury to his abdomen and lower extremities from an IED (improvised explosive device) blast approximately 3 hours ago. His heart rate is 110 beats per minute and his blood pressure 100/70 mmHg. His hematocrit is 35 (normal 40-45). The ambulance tech estimates he has lost nearly 1 liter of blood. The hematocrit is most likely reduced to 35 because

he was given whole blood after he was wounded
venoconstriction of the splanchnic venous circulation to deliver blood to other areas of the body
transcapillary flux of interstitial fluid due to low capillary pressure
preferential red blood cell loss due to hemodynamic streaming and turbulent flow at the wound site

A

Correct answer: Transcapillary flux of interstitial fluid due to low capillary pressure.

Explanation: Low capillary pressure will enhance the movement of cell free fluid from the interstitium into the blood (Starling’s law of capillary exchange), reducing the concentration of red blood cells. Capillary pressure is reduced through two major mechanisms. First, when blood pressure is lowered capillary pressure is also. Secondly, reflex sympathetic vasoconstriction causes an increase in the pressure drop from artery to capillary and, decreasing capillary pressure even more.

Incorrect answers:

Whole blood administration would tend to maintain hematocrit because whole blood contains blood cells in a normal concentration.

A person bleeds whole (unconcentrated and undiluted) blood from a wound. A significant bleed would not be associated with hemoconcentration. Turbulence has no effect on red blood cell loss but would favor bleeding of whole blood.

Venoconstriction would occur as a reflex response to hemorrhage. However, as this would mobilize whole blood from the splanchnic circulation into other areas of the body, it would serve to maintain hematocrit and cardiac output.

140
Q

An 18 year old man is involved in a motor vehicle accident. In the ER he is alert and oriented and complains of pain in his left hip. On examination, the left lower extremity is internally (medially) rotated and is obviously shortened in comparison to the uninjured side, but there is no apparent fracture. The sensation and circulation to the foot appear to be intact, but the patient maintains the left knee in slight flexion and guards it from being extended. Your working diagnosis is

posterior dislocation of the hip
sciatic nerve palsy
traumatic diastasis of the symphysis pubis
anterior dislocation of the hip

A

Correct answer: posterior dislocation of the hip

Explanation: Anatomically, this scenario is classic for a posterior dislocation of the hip joint. It can be associated with axial loading of the femur when the passenger is thrown forward, slamming the bent knee into the dashboard. Clinically, the head of the femur is forced posteriorly and superiorly, and sometimes it also fractures the posterior rim of the acetabulum as it is driven back. The lower extremity is characteristically anatomically shortened and the thigh is internally (medially) rotated since the hip adductors and medial hamstrings are now quite tight. This tightening also brings the knee into flexion and it cannot be straightened by the patient.

On the other hand, with an anterior hip dislocation, there is no anatomic shortening of the extremity, the thigh is externally (laterally) rotated, and the anteriorly displaced femoral head can usually be palpated under the skin just inferior to the middle of the inguinal ligament - sometimes with the femoral artery “draped” over it and easily palpable.

Sciatic nerve palsy is not associated with this posture, but is rather characterized by flaccid weakness of the knee flexors (hamstrings) and calf musculature and hypesthesia (poor sensation) of the sensory territories innervated by the tibial and common fibular (peroneal) nerves (characteristically including parts of the sole of the foot).

141
Q

Which of the following conditions is the most typically results in increased pulmonary compliance (the pressure-volume curve for the lungs shifted to the left)?

Restrictive lung disease
Surgical removal of one lung
Chronic Obstructive Pulmonary Disease (emphysema)
Fibrosis of the lungs

A

Answer: Chronic obstructive lung disease

Compliance is change in volume / change of pressure. COPD is a condition with floppy lungs. the volume expands with little effort, but the lack of elasticity makes it difficult to expel the air. In contrast, fibrosis is “stiff lung” and compliance is decreased. Removal of a lung should not change the characteristics of the remaining lung if it is otherwise healthy.

142
Q

A formula-fed, 1-month-old boy is exposed to his sister who has chickenpox, which is caused by the varicella zoster virus (VZV). The boy does not develop any clinical signs of VZV infection. His mother had the infection 15 years ago. The boy has MOST LIKELY been protected from developing chickenpox by

CD8+ T cells
IgM
Neutrophils
IgG

A

Explanation: IgG crosses the placenta from mother to fetus and is stable in the circulation for several months after birth. If the mother previously had chickenpox (or the vaccine), some of her IgG would be specific for VZV, neutralizing the virus in the infant.

Incorrect answers

CD8+ T cells cannot cross the placenta from mother to fetus; the infant would not have VZV-specific memory CD8+ T cells at the time of infection.

IgM cannot cross the placenta from mother to fetus, and the infant would have VZV-specific IgM-secreting B cells at the time of infection.

Neutrophils do not play a major role in anti-viral immunity

143
Q

Nerve damage resulting from fracture of the humerus in its middle-third is most likely to produce loss of which of the following?

flexion of the thumb
extension of the wrist and digits
flexion of the digits
pronation in the forearm

A

Correct answer: extension of the wrist and digits

Explanation: The above type of fracture is most closely associated with radial nerve injury. In its course through the arm, the radial nerve is closely applied to the middle third of the humerus within the radial (spiral) groove of the humerus. Certain fractures in this area, especially if the fragments are displaced and depending on the force of the injury, can injure the radial nerve, causing a weakness or paralysis of the muscles that are innervated by the radial nerve. These include the wrist and digital extensor units, and such injuries can result in inability to raise (extend) the wrist and digits of the hand. The other responses are associated with injuries to the median and ulnar nerves.

144
Q

A recent study followed 9,637 Finnish men and 11,430 women who were 25 to 74 years of age and free of hypertension during the baseline measurements (1982-2002). Healthy lifestyle factors were defined as: (1) not smoking, (2) alcohol consumption less than 50g per week, (3) leisure time physical activity at least 3 times per week, (4) daily consumption of vegetables, and (5) normal weight (BMI). This is an example of which type of study?

Ecologic study
Clinical trial
Cross sectional study
Cohort study

A

Correct Answer: Cohort study.

Explanation: A cohort study starts with exposed and unexposed groups and assesses outcomes at a later time. The rate or proportion of the outcome in the exposed group is compared to the rate or proportion of outcome in the unexposed group to assess risk associated with the exposure. The exposures in a cohort study are not assigned by the investigator, but occur based on environmental, lifestyle, genetic, or other factors.

Other answers:

Clinical Trial: the investigator assigns the exposure in a clinical trial and then follows to assess the rate of outcomes in the different exposure groups.

Ecologic Study: an ecologic study uses population exposure data and population outcome data. Thus, you don’t necessarily know if the persons with the outcome were the same persons who were exposed. These studies are generally less expensive since they use existing data and you may use the results to generate or support a hypothesis, which in turn may justify more rigorous studies.

Cross-sectional study: also known as a prevalence study, this design looks at the current “burden of disease” in a population and may be useful for health services planning or hypothesis generation. The outcome and exposure status of each individual in a population of interest is assessed at the same time, thus you often are not certain whether the exposure occurred before the outcome.

145
Q

Among overweight/obese children and adolescents in the US, the odds ratio comparing odds of pre-high blood pressure or high blood pressure (pre-HBP/HBP) among subjects with high sodium intake compared to those with low sodium intake was 3.5 (95% confidence interval 1.3 – 9.2) (Pediatrics 130:4, 611-619). Based on this finding, what conclusion can be drawn about the association between sodium intake and pre-HBP/HBP among overweight or obese children and adolescents in the US?

The result could be due to type II error
There is a 95% chance that the sample odds ratios is between 1.3 and 9.2
High sodium intake is associated with a more than 3-fold increase in odds of pre-HBP/HBP
The association is not statistically significant (P > .05)

A

Answer: high sodium intake is associated with a more than 3-fold increase in odds of pre-HBP/HBP

The odds ratio of 3.5 indicates that the odds of pre-HBP/HBP among children with high sodium intake is 3.5 times higher than the odds of pre-HBP/HBP among children with low sodium intake. The 95% confidence interval does not include 1.0, indicating that the odds ratio is significantly different from 1.0 at the 5% significance level (P

146
Q

Which of these cancer chemotherapeutic agents would be MOST EFFECTIVE in treating a patient with CD20-positive non-Hodgkin’s B cell lymphoma, while having the LEAST bone marrow toxicity?

cyclophosphamide
irinotecan
cetuximab
rituximab

A

Correct answer: Rituximab.

Explanation. Rituximab is far less toxic than cyclophosphamide (an alkylating agent) and irinotecan (a “natural product” inhibitor of DNA topoisomerase I.) Rituximab is a chimeric (mouse/human) anti-CD20 monoclonal antibody that promotes complement mediated lysis of malignant B-cells in CD20-positive non-Hodgkin’s B cell lymphoma, and CLL. Cetuximab is directed against the EGF receptor and not CD-20.

147
Q

A 10-year-old boy presents with atypical pneumonia caused by the fungus Aspergillus fumigatus. A complete blood count reveals that all types of white blood cells are present in normal quantities. Serum immunoglobulin concentrations are in the high normal range and the patient makes a normal delayed-type hypersensitivity (DTH) response to tetanus toxoid. However, all the patient’s neutrophils are unable to reduce nitro blue tetrazolium (NBT) dye. Which enzyme is MOST LIKELY defective in this patient?

Eosinophil peroxidase
NADPH oxidase
Cytochrome oxidase
Granzyme B

A

Answer: NADPH oxidase

Explanation: Failure to reduce NBT indicates a defect in the neutrophil oxidative burst, especially in phagocyte NADPH oxidase enzyme which catalyzes the reaction between NADPH and oxygen to form superoxide. Individuals with defects in oxidative burst have a decreased ability to kill phagocytosed organisms. This defect underlies the disease chronic granulomatous disease (CGD). CGD occurs in about 1 per 200,000 births in the U.S. Patients with CGD develop recurrent pneumonia, superficial skin infections, abscesses of the skin and organs such as the liver, bacteremia, and osteomyelitis. Common infectious agents include Staphylocci, Burkholderia, Serratia, Nocardia, and Asperigllus. All of these are usually controlled in part by oxidative burst inside macrophages after the organisms have been phagocytosed.

Incorrect answers

Cytochrome oxidase is an oxidizing enzyme containing iron and a porphyrin. It is found in mitochondria and is important in cell respiration as an agent of electron transfer from certain cytochrome molecules to oxygen molecules

In the presence of H2O2 produced by eosinophils, and either chloride or bromide ions, eosinophil peroxidase provides a potent mechanism by which eosinophils kill multicellular parasites.

Granzyme B is one of the serine proteases located in the granules of cytotoxic lymphocytes and NK cells

148
Q

You prescribe duloxetine to a 25-year-old man who is suffering from major depressive disorder. He is very irritable and reports an inability to sleep well or focus on his work. Three weeks after taking duloxetine, his symptoms start to improve. Duloxetine is MOST likely inhibiting

the dopamine transporter but not the norepinephrine or serotonin transporters.
the serotonin and dopamine transporters but not the norepinephrine transporter.
the norepinephrine and dopamine transporters but not the serotonin transporter.
the norepinephrine and serotonin transporters but not the dopamine transporter.
the serotonin transporter but not the norepinephrine or dopamine transporters.

A

Correct answer: The norepinephrine and serotonin transporters but not the dopamine transporter.

Explanation: Duloxetine is an SNRI – a specific serotonin and norepinephrine reuptake inhibitor. In general the SSRIs and the SNRIs do not affect the dopamine transporter. None of the other answers provide the correct specificity of inhibition of the transporters.

149
Q

A 35 year old Army Captain presents with a 4-month history of a nonproductive cough. The cough occasionally awakens him at night. He denies wheezing, dyspnea, heartburn, or chest pain. On physical examination, he is in no acute distress. Vitals signs are T= 98.4 F, HR = 80/min, RR= 16/min, and BP= 122/78 mm Hg. He has mild nasal mucosal congestion and cobblestoning of the oropharynx. His cardiac and pulmonary examinations are normal; results of chest radiography are normal. What is the MOST likely diagnosis?

Mitral stenosis
Post-nasal drip syndrome (upper airway cough syndrome)
Congestive Heart Failure (CHF)
GERD

A

Post nasal drip syndrome is the most common cause of chronic cough followed by asthma and GERD. In the absence of symptoms or signs suggesting the latter two diagnoses and with the nasal mucosal findings and cobblestoning the most likely diagnosis is post-nasal drip syndrome. There are no cardiac findings to support a diagnosis of CHF. Additionally, if mitral stenosis were the cause of this patient’s chronic cough one would expect to hear a murmur on cardiac exam and potentially observe an abnormal cardiac silhouette on chest x ray.

150
Q

As compared to levels normally present in the early follicular phase, low levels of estrogen and progesterone, and very high levels of LH and FSH are found in a female who is:

entering puberty.
undergoing ovulation.
in the postmenopausal state.
a newborn.

A

Correct answer: in the postmenopausal state.

The postmenopausal state is the condition that exists after ovarian follicles have been depleted and the production of estrogen and progesterone is essentially reduced to zero. Accordingly, the pituitary is relieved of steroid hormone negative feedback and the secretion of LH and FSH is therefore unrestrained.

Incorrect answers

At the time of puberty levels of LH, FSH, estrogen and progesterone are all rising to normal adult levels

At ovulation levels of estrogen, LH and FSH are all elevated, marking the follicular growth that has occurred during the follicular phase and the ovulatory surge of the gonadotropins.

In a newborn all activities of the hypothalamic-pituitary-gonadal axis are profoundly suppressed.

151
Q

A 21-year-old truck driver has developed allergic rhinitis. Which of the following is the BEST treatment to relieve her symptoms while driving?

diphenhydramine
fexofenadine
verapamil
meclizine

A

Correct answer: fexofenadine.

The anti-histamines, diphenydramine and meclizine, are first generation anti-histamines and therefore cross the blood brain barrier and cross react with a number of different receptors, causing drowsiness and loss of alertness and reaction time. Meclizine is used to treat motion sickness and not allergic rhinitis. For a truck driver you need to be particularly aware of the detrimental side effects of first generation anti-histamines like diphenhydramine, and prescribe a second generation anti-histamine, which do not cause drowsiness. The only one provided here is fexofenadine. Verapamil is a calcium channel blocker. It is not used to treat allergic rhinitis.

152
Q

Baby Jones is born at 26 weeks gestation (11weeks premature) and weighs 900 grams (2 lbs). At 7 days of age the infant develops respiratory distress, bounding pulses, and widened pulse pressures. A chest x-ray demonstrates excessive pulmonary blood flow and an enlarged heart. The use of a prostaglandin inhibitor may help close which ONE of the following structures to help minimize excessive pulmonary blood flow?

Crista dividens
Ductus venosus
Ductus arteriosus
Foramen ovale

A

Answer: Ductus arteriosus

The ductus arteriosus is a vessel that connects the pulmonary artery to the aorta in utero. Since pulmonary vascular resistance is very high in the non-respiring fetal lung, pulmonary artery pressure is high pushing blood into the aorta. This shunts poorly oxygenated blood toward the placenta. During pregnancy, prostaglandins help keep the ductus arteriosus open.

At birth, the ductus typically closes within 15-20 hours. The process of closure is complex, and is likely due to a number of mechanisms including decreased prostaglandin levels (due to removal of the placenta, a main source of prostaglandins during pregnancy) and vasoconstriction initiated by increase in blood oxygen levels.

A patent ductus arteriosus is detrimental after birth. After birth, the intake of oxygen into the lungs by breathing results in vasodilation of the pulmonary vascular bed, substantially decreasing pulmonary vascular resistance. This decreased pulmonary vascular resistance relative to systemic resistance results in a reversal of flow through a patent ductus (from right to left shunting, to left to right shunting). Thus, after birth blood is shunted through a patent ductus arteriosus from the systemic circulation to the pulmonary circulation. The increased flow of blood through the pulmonary artery increases pulmonary artery pressure and puts additional stress on the right heart. This causes enlargement of the heart. These stresses can be reduced by closure of the ductus. A prostaglandin inhibitor can help close a patent ductus arteriousus.

Incorrect answers

Blood is also shunted from right to left via the foremen ovale. However, closure of the foremen is not believed to be chemically controlled. The FO closes due to the reversal of pressures in the atrium closing it like a flap or valve.

The Crista Dividens is part of the foramen ovale and is not sensitive to chemicals.

The ductus venosus carries blood from the placenta to the right heart, bypassing the liver. It atrophies when the placenta is removed and no longer carries blood.

153
Q

A 20 y/o nursing student recently recovered from a week long bout of ‘flu’ but complains of persistent fatigue. A hemogram revealed a hemoglobin of 10.1 gm/dL ,platelet count of 27,000, and WBC count of 2,470 with differential count of 24% mature neutrophils, 8% monocytes and 68% mature lymphocytes. Which of the following results would most likely be associated with her illness?

Elevated platelet count
Elevated serum erythropoietin
Elevated unconjugated bilirubin in serum
Elevated reticulocyte count

A

Answer: elevated serum EPO

The hemogram points toward a post-viral pancytopenia associated with the onset of either a transient or life-threatening aplastic anemia. Provided retained normal renal function, a homeostatic increase in serum EPO would be anticipated. Changes in the serum bilirubin would suggest accelerated RBC destruction but targeted RBC destruction is not typically accompanied by a decreased WBC or an elevation in the platelet count. An elevated platelet count with a decreased neutrophil count would be paradoxical since they share common bone marrow precursors. Elevated reticulocyte count would be observed in cases of RBC loss (bleeding) or destruction.

Robbins PBOD 8ed. pp 662-665

154
Q

Dr. Holdonasek is looking for your advice on how to treat an Asian patient experiencing seizures. He would like to prescribe the anticonvulsant drug carbamazepine, but he vaguely remembers he should run a test first to avoid an unpleasant hypersensitivity reaction. Which of the following tests do you tell Dr. Holdonasek to order?

HLA genotype
Serum test for IgE levels
Serum test for immune complexes
Urine protein test

A

The correct answer is HLA genotype.

Explanation: Patients of East Asian descent that harbor the HLA-B*1502 allele are at risk for carbamazepine-induced Steven-Johnsons Syndrome, a life-threatening skin condition associated with a Type IV hypersensitivity reaction. Although carbamazepine use is associated with a small risk of causing proteinuria, proteinuria is not a contraindication for using carbamazepine and is not related to a drug-induced hypersensitivity reaction.

155
Q

A 56 year old woman from Hagerstown, MD returns from a two week safari vacation in Kenya. She did not seek any pre-travel medical advice before departure. While there, she spent a few days in urban areas (Nairobi) but was also on safari in Masai Mara and Tsavo (game parks). During her trip, she ate at a few restaurants as well as the catered food served at the game park camps where she stayed. On occasion, she drank local water when she did not have bottled water available. Shortly after her return to the US she developed diarrhea with fever and myalgias. Following a call to her travel companion, she is given some ciprofloxacin (which her friend had obtained from her travel clinic) and feels transiently better. Her fever persists, though, and she develops chills and night sweats. Because of these symptoms, she is brought to your clinic by her friend. You take a careful history and decide to order thick and thin smears of her blood. What is the next step in management of this patient?

Give her a prescription for oral doxycycline with instructions to her friend to have her return to your clinic tomorrow for follow up.

Give her a prescription for doxycycline and quinine, orally, with instructions to return to your clinic tomorrow for follow up.

Call the CDC to begin process of obtaining IV artesunate while patient is admitted to the ICU and started on IV quinidine.

Start CoArtem (artemether + lumefantrine), orally, and observe her in the ER for several hours to ensure that she is responding.

A

Answer: call the CDC to begin process of obtaining IV artesunate while patient is admitted to the ICU and started on IV quinidine.

This is a medical emergency. A non-immune patient with hyperparasitemic infection with Plasmodium falciparum has a high risk of death due to complicated malaria. This blood smear shows ~ 50 RBCs, of which 7 are parasitized cells, so she has a parasitemia of approximately 14%. Of course, additional laboratory studies should be obtained to determine whether she has evidence of organ dysfunction (hepatic, renal, pulmonary), metabolic acidosis, or coagulopathy. She should also have serial neurological assessments of her level of consciousness. Nonetheless, with only this blood smear in hand, you know this patient warrants immediate admission to the ICU and prompt initiation of intravenous antimalarial chemotherapy. Intravenous quinidine remains the only FDA-approved therapy for severe malaria in the US but intravenous artesunate is available under an emergency IND (investigational new drug) program that is run by the CDC in collaboration with the Walter Reed Army Institute of Research. Frequently, hospital formularies no longer carry intravenous quinidine since it is no longer used by cardiologists as an antiarrhythmic drug. Quinidine can be a complicated medication to manage because of consequential QT interval prolongation. If the patient had received a quinolone-based oral treatment enroute to care, that would have to be taken into account while choosing the initial quinidine dosing. Quinidine only acts on the 3rd ring trophozoite stage of the parasite. Artesunate and related medications, however, are active against all blood stages of the parasite. While awaiting the arrival of IV artesunate, if no quinidine is available, oral artemether-based combination therapy should be initiated. (The number to call at CDC is always provided in the Sanford Guide to Antimicrobial Therapy; it is also found at http://www.cdc.gov/malaria/diagnosis_treatment/artesunate.html.)

Other answers: Despite the lack of a given history of vomiting in this patient’s presentation, high parasitemia meeting critera for severe malaria should be treated with parenteral therapy. Doxycycline alone can be used for chemoprophylaxis, but it is a relatively slow acting drug that should never be used alone for treatment, regardless of plasmodium species or level of parasitemia. Doxycycline + oral quinine is an acceptable regimen for uncomplicated malaria with low parasitemia, but in a non-immune patient with falciparum malaria it should be used cautiously and NOT as an outpatient. Now that artemether + lumefantrine (Co-Artem®, Sanofi) is an approved product in the USA, this would be a better choice of oral regimen if the patient had no evidence of complicated malaria or high parasitemia

156
Q

Which of the following is an alkylating agent that must be activated by hepatic cytochrome P450s?

cyclophosphamide
lomustine
cetuximab
irinotecan

A

Correct answer: cyclophosphamide

Explanation: Cyclophosphamide and lomustine are the only alkylating agents listed. The requirement for activation by cytochrome p450 only applies to cyclophosphamide. This is a frequent Step 1 question.

Incorrect answers: Cetuximab is an antibody directed against the EGF receptor. Irinotecan is a “natural product” inhibitor of DNA topoisomerase I.

157
Q

In the operating room, your patient is positioned with her left side down during a general anesthetic. Following your uncomplicated two-hour procedure, the patient awakens in the recovery room and states that she cannot raise (extend) her left ankle or left toes. Several weeks later this has not improved, and the patient cannot walk without a cumbersome foot-toes extension splint. The most likely reason for this is a/an:

Compression neuropathy of the left fibular (peroneal) nerve
Idiosyncratic reaction to the general anesthetic
Neuropathic effect of the paralyzing agent used during the anesthetic
Conscious effort by the patient to pursue malpractice litigation

A

Correct answer: compression neuropathy of the left fibular (peroneal) nerve

Explanation: This scenario is characteristic of a fibular (aka peroneal) nerve injury secondary to compression neuropathy. In this case, the cause is arguably inadequate padding during the time that the patient was positioned with her left side down, while undergoing a contralateral surgical procedure. The weight of her opposite lower extremity, the surgical drapes, and possibly instruments are then transmitted to the left side and its pressure points: the left greater trochanter of the femur; the left knee and fibular nerve; and the left lateral malleolus.

The fibular nerve, which is motor to the anterior and lateral compartments of the leg (the “leg” is anatomically defined as that part of the lower extremity from the knee to the ankle), innervates the main ankle and toe extensors (tibialis anterior, extensor digitorum longus, and extensor hallucis longus). Without this function, it becomes impossible for the patient to effectively extend the ankle during ambulation and the patient continually stumbles because the foot and toes cannot clear the walking surface. Fast walking or running become impossible.
When a patient is unconscious, the common fibular nerve is vulnerable to compression because it is situated relatively superficially between the skin and the neck of the fibula. As such, it can be easily compressed between the bone and any hard surface. This injury characteristically occurs when the patient is unconscious (as during a surgical procedure) or inebriated. Unfortunately, some of these neuropathies can be permanent.

It is incumbent on the surgeon himself/herself to routinely check all pressure points when the patient is asleep and positioned on the operating table, and before commencing any procedure. This is a responsibility that should not be delegated, and must be done on every case in order to personally confirm that vulnerable areas are well protected with adequate padding. As such, it should also be logged in the operation report that this is done postoperatively.

158
Q

A 45 y.o. woman with a recent Breast Cancer diagnosis undergoes chemotherapy that targets fast replicating cells. She reports nausea and gastrointestinal distress following the chemotherapy treatment. Which component of the GI system is most responsible for her symptoms?

Mesothelium of the ileum
Longitudinal smooth muscle
Endothelium of blood vessels of the lamina propria
Epithelium of the intestinal villi

A

Answer: epithelium of the intestinal villi.

Of the choices provided, the cells of the epithelium of the intestinal villi are the only ones that reproduce with a high turnover rate. Consequently, these cells are most affected by the chemotherapy. The other cell types divide slower and are not significantly affected.

159
Q

A 4-year-old child who has had repeated infections with staphylococci and streptococci (both bacteria that colonize extracellular spaces) has normal phagocytic function and delayed-type hypersensitivity (DTH) responses. A lymph node biopsy would MOST LIKELY reveal an absence of:

macrophages
germinal centers
paracortical areas
CD8+ T cells

A

Answer: germinal centers

Explanation: Germinal centers are the location of B cells and antibody production within lymph nodes. Staphylococci and streptococci are extracellular pathogens, which are typically cleared in part by antibody responses.

Incorrect answers

Since the DTH responses are normal, there is no evidence of a T- cell defect

Since the phagocytic function in the child is normal, and macrophages are a major phagocytic cell, it is unlikely macrophages would be absent from the lymph nodes.

The paracortex is the area within the secondary lymphoid organs where T cells reside. Since the DTH responses are normal, there is no evidence of a T- cell defect.

160
Q

Where are the uterine arteries located?

within the fold of peritoneum containing the ovarian vessels
within the cardinal (transverse cervical) ligaments
in the fimbriae
inferior to the ureters bilaterally

A

Correct answer: within the cardinal (transverse cervical) ligaments

Explanation: The cardinal (transverse cervical) ligaments are a portion of the uterosacral support system and help to support the uterus and uterine cervix. The uterine arteries run within the peritoneal broad ligament and the ureters are close to these vessels. The ureters pass on each side of the cervix area and are caudal to the uterine arteries (a relationship referred to as “water runs under the bridge”). Because of this proximity to the uterine arteries, the ureters are vulnerable to injury during a hysterectomy since these arteries must be clamped and ligated during this procedure. An injury to the ureter, especially if not immediately repaired, can be devastating to ureteric and renal function.

161
Q

Patients with vitamin B12 deficiency often show clinical signs of folate deficiency as well. (B12 deficiency causes neurological problems whereas folate deficiency causes hematological problems.) Analysis of the blood of these patients often shows that the total concentration of folate is normal (the assay cannot distinguish between the various forms of folate). The explanation for the folate deficiency symptoms is that B12 is necessary for a key step in folate metabolism. Which one?

FH4 + serine –> N5, N10 –methylene FH4
N5, N10 –methylene FH4 –> N5- methyl FH4
N5- methyl FH4 –> FH4
Folate –> tetrahydrofolate (abbreviated FH4)

A

Correct Answer: N5- methyl FH4 –> FH4

Explanation: See Fig 40.9 of Marks (3rd Edition) for the “methyl cycle.” Vitamin B12 is necessary to recycle FH4. When B12 is deficient, all the folate in the cell winds up as methyl FH4, the so called “methyl trap.” In this situation, other forms of folate are depleted, one of which is N5, N10 –methylene FH4 , needed to produce dTMP from dUMP (Fig 40.5). Without dTMP, DNA cannot be formed. The clinical consequence is megaloblastic anemia, caused by the inability of hematopoietic cells to divide.

162
Q

In the adult central nervous system, glia cells that are phagocytotic and play a role in defense against invading microorganisms are

Microglia
Astrocytes
Ependymal cells
Neurons

A

Explanation: Microglia are phagocytic cells found in the central nervous system.

Incorrect answers:

Astrocytes provide physical and metabolic support for neurons of the central nervous system.

Ependymal cells line the fluid-filled cavities of the central nervous system and place a role in fluid transport within the brain.

Neurons are the functional units of the nervous system.

163
Q

A 35 y.o. Navy surgeon complains of heartburn and stomach pain. After endoscopy, she is diagnosed with a duodenal ulcer. Which one of the following mechanisms best describes how the duodenum protects itself against ulcer formation due to excess acid?

Peptidases are released in the inactive form.
S cells release secretin.
Enterochromaffin-like cells release histamine.
The duodenum has a stratified epithelial mucous membrane.
The duodenum has a thick layer of aqueous mucus.

A

Answer: S cells release secretin.

S cells release secretin which stimulates the secretion of bicarbonate by the pancreas. This neutralizes acidic chyme entering the duodenum.

Incorrect answers:

Histamine would increase acid secretion in the stomach by the parietal cells.

Peptidases are released in the inactive form, but this is irrelevant to the acid problem.

“The duodenum has a stratified epithelial mucous membrane” is an incorrect statement. The duodenum has a simple columnar epithelium.

164
Q

A 40-year-old Major presents to your office complaining of a several week history of severe unilateral headaches with rhinorrhea and tearing of the eye on the side of the headache. Episodes are often precipitated by alcohol. He has had these headaches in the past, and they can be present for weeks to months, after which a headache-free period occurs. What is the MOST likely diagnosis?

Cluster headache
Brain tumor
Tension headache
Temporal arteritis

A

Answer: cluster headache

This headache is most consistent with a type of neurovascular headache called cluster headache. Cluster headaches occur most often in young men, have a characteristic periodicity (they cluster together in time, then go away for a period of time, then return), and cause lacrimation, nasal stuffiness, and sometimes conjunctival inflammation. Migraines tend not to come and go in this manner, are more throbbing, and are more likely to be associated with nausea and vomiting. Sinusitis is usually bilateral and may have associated fever and purulent discharge. Tension headaches are usually described as bandlike, without lacrimation or nasal congestion.

165
Q

Upon reaching the summit of Annapurna (altitude 26,500 ft) while using supplemental oxygen breathing, a climber is found to have heart rate of 120 beats per minute, systolic blood pressure 120 mm Hg, and systemic arterial oxygen saturation 98%. Compared with her status at camp (altitude 24,800 ft) a day earlier, when her heart rate was 60 beats per minute, systolic blood pressure was 100 mm Hg, and arterial oxygen saturation was 97%, this climber’s coronary blood flow when evaluated at summit is likely:

increased in response to acute myocardial hypoxia
increased in response to raised myocardial metabolic activity
unchanged due to coronary homeostatic mechanisms
decreased due to altitude-induced coronary vasoconstrictor reflexes

A

The correct answer is: Increased in response to raised myocardial metabolic activity.

An increase in heart rate and/or an increase in systolic blood pressure will augment the work of the heart and myocardial metabolic demand. Since the coronary circulation has limited ability to extract more O2 from each milliliter of blood, coronary blood flow must increase to satisfy the greater demand.

Incorrect answers:

“Unchanged due to coronoary homeostatic mechanisms” is incorrect. Myocardial O2 demand is increased by the greater metabolic activity associated with the increase in heart rate and blood pressure. Since the O2 content is already low in venous blood returning from the myocardium, this increased O2 demand must be met by increased supply or ischemia will occur. Coronary blood flow must increase.

“Decreased due to altitude-induced coronary vasoconstrictor reflexes” is incorrect. This is the opposite of what occurs. Increased O2 demand requires increased supply, which occurs through dilation of coronary arteries.

“acute myocardial hypoxia” is not occurring. This is documented by her nearly-complete hemoglobin O2 saturation (98%) at summit. Systemic arterial saturations of 95-100% are normal at rest and during strenuous exertion. Even in the absence of systemic arterial hypoxia, one might argue that transient local micro-hypoxia stimulates the increase in blood flow. However, the extent of hypoxia must be very small: myocardium supplied by normal coronary arteries and perfused with normal levels of fully saturated hemoglobin never exhibits hypoxic dysfunction, even at maximal exertion.

166
Q

Patient X is a 7-year-old female with a history of recurrent viral infections. In the course of your
extensive evaluation of Patient X, you perform a fluorescence-activated cell sorting (FACS) analysis on peripheral blood from this patient. You find Patient X completely lacks CD8+ T cells. This deficiency would likely explain the patient’s illness because CD8+ T cells are cytotoxic T lymphocytes that kill target cells via secretion of:

complement and acute-phase proteins
antibodies and complement
Fas and Fas ligand
pore-forming proteins and proteases

A

Answer: pore-forming proteins and proteases

Explanation: Pore-forming proteins such as perforins are secreted by the cytolytic T cell and create pores in the membranes of target cells. Proteases such as granzymes are secreted by cytolytic T cells and enter target cells via the perforin-induced pores, causing target cell death.

Incorrect answers

Cytolytic T cells do not secrete antibodies or complement.

Cytolytic T cells do not secrete complement or acute phase proteins.

Fas ligand, which induces apoptotic cell death when it binds to its receptor, is expressed on the surface of cytolytic T cells but is not secreted.

167
Q

A 24 year old Ensign “passes out” while standing in formation. She had been standing for approximately 45 minutes when she had the onset of flushing, nausea and lightheadedness. She next recalls a brief “dimming or graying” of her vision, and then recalls nothing until “coming to” looking up into the concerned faces of 160 fellow students standing over her. She is brought to the WRNMMC Emergency Department by ambulance within 10 minutes, and has a completely normal physical exam and ECG. She now complains of fatigue. Her pulse oximeter reads 99% oxygen saturation on room air. The MOST likely explanation for her loss of consciousness is which of the following?

Syncope due to aortic stenosis
Vasovagal syncope (neurocardiogenic syncope)
Grand mal seizure
Syncope due to high-degree A-V block

A

Answer: vasovagal syncope

Everything in this case favors vasovagal syncope, aka “the common faint.” There is a clear-cut strong emotional provocation, followed by premonitory (warning) symptoms of an impending faint: the transient feelings of warmth, nausea, and lightheadedness with associated dimming of vision proceeding to “blackout.”

Other answers: Prompt recovery after hitting the deck makes seizure unlikely. The fact that the event was not associated with significant exertion mitigates against aortic or pulmonic outflow tract obstruction, as does the normal physical exam. Patients with syncope due to an arrhythmia often experience no prodrome prior to losing consciousness. The normal ECG also decreases the likelihood that arrhythmia was the underlying cause.

168
Q

A 63 year-old woman presents to the emergency room with shortness of breath. She recently returned home from a 12-hour flight. You consider two tests for diagnosing a pulmonary embolism: V/Q scan and CT Angiography. Assume the V/Q scan has a sensitivity of 70% and a specificity of 90%, and the CT Angiography has a sensitivity of 90% and a specificity of 70%. Which test, if negative, would be better at ruling out a pulmonary embolism in this case?

V/Q Scan
CT Angiography
There is not enough information to answer this question
They are equally good at ruling out disease

A

Correct answer: CT Angiography.

In this hypothetical scenario, the CT angiography is more sensitive than the V/Q scan (90% vs 70%). In other words, it has a lower false negative rate (10% vs 30%), which means a negative result is better at ruling out a pulmonary embolism. Specificity is more important for ruling in disease, so the V/Q scan in this case would be better. Because the sensitivities are different, they are not equal at ruling out disease (CT angiography is better). Remember the mnemonic “SPin SNout”: specific tests are better for ruling in, while sensitive tests are better for ruling out.

169
Q

A 62-year-old hospital heating and air-conditioning technician seeks medical attention for pulmonary symptoms including cough, nausea, diarrhea, and a fever of 102.5oF. His symptoms have steadily worsened over the last three days. The patient is a moderate smoker, and he does not use alcohol. A Gram stain and culture of sputum showed only organisms consistent with normal flora in the respiratory tract. A urine antigen screen reveals the causative agent. It is a bacterium that

is lactose positive on MacConkey agar and produces a mucoid capsule
survives in the environment within free-living amebas
has a zoonotic reservoir in parrots and domesticated fowl
is acid fast and grows very slowly on Lowenstein-Jensen agar

A

The correct answer is survives in the environment within free-living amebas.

Explanation: This case describes legionnaire’s disease. Legionella pneumophila is a facultative intracellular organism that colonizes the alveolar macrophages in a manner similar to its lifestyle in the environment where it parasitizes and survives within amebas. Clues to this being Legionella pneumophila are the age, occupation and lifestyle of the patient. Unlike other types of pneumonias, this disease is often accompanied by gastrointestinal symptoms. Diagnostic tests are difficult and time consuming, but fortunately there is a urine antigen detection kit that recognizes the Legionella LPS. The other pneumonia-causing organisms alluded to in the other answers are Chlamydia psittaci (which has a zoonotic reservoir in parrots and domesticated fowl), Mycobacterium tuberculosis (which is acid-fast and grows very slowly on Lowenstein-Jensen agar) and Klebsiella pneumonia (which is lactose positive and produces a mucoid capsule).

170
Q

A 25 year old woman presents with a chief complaint of passing thin, white, flat tissue fragments in her stool several times over the past few weeks. She has no diarrhea and no pain in her abdomen, and reports that generally she feels quite well. Travel history is significant for working in rural areas of Madagascar for 3 years as a Peace Corps volunteer until six months ago. She says that a few times while there she ate pork that may have been a bit undercooked. You are concerned because at the top of your differential diagnosis is a parasitic infection which can increase the risk of:

liver abscess
epilepsy
iron deficiency anemia
bladder cancer

A

Answer: epilepsy

Thin, white, flat tissue fragments are most consistent with proglottid segments of a tapeworm. Infections with intestinal tapeworms rarely cause systemic symptoms. Rather, individuals sometimes just report seeing proglottid segments in their stool or may report the sensation of proglottid segments moving through the anus. Infection with the pork tapeworm, Taenia solium, increases the risk of epilepsy because ingestion of eggs leads to neurocysticercosis (infection with larval cysts in the brain). Remember – the adult tapeworm stage is acquired by ingestion of infective larvae in undercooked pork. The larvae cannot mature in humans and so die in tissue, sometimes being enveloped in cysts, becoming calcified, or both. Patients with high exposures to this parasite sometimes have abnormalities on plain radiography of their muscle girdles, shoulders, back and thighs. Even strict vegetarians can acquire neurocysticercosis by ingesting eggs in food that has been contaminated with human fecal matter. Patients with active adult tapeworms, such as the patient in this vignette, are at increased risk for neurocysticercosis because they are at increased risk of swallowing eggs passed in their own stool. T. saginata is the beef tapeworm. Distinguishing between these two tapeworms requires a close inspection of the worm. The beef tapeworm is not associated with cysticercosis.

Other answers:

Of parasitic infections, iron deficiency anemia is most commonly associated with hookworm infections. Roundworm infections such as hookworms do not typically present as pieces of tissue passed in the stool. While most tapeworms do not cause anemia, Diphylobothrum latum can but typically does so by inducing vitamin B12 deficiency, not iron deficiency. Also, D. latum is much less prevalent than T. solium worldwide in humans and is not endemic in Africa.

Liver abscesses are typically caused by bacteria or amoebae. The larval form of the dog tapeworm Echinococcus multilocularis, which is acquired by ingesting eggs from the feces of dogs infected with the adult parasite, typically causes cystic lesions in the liver but can be present in other organs or body compartments. This tapeworm is very small, and the adult worms do not infect humans.

Bladder cancer has been associated with Schistosoma haemotobium, a fluke infection which resides primarily in the veins draining the bladder.

171
Q

LT Jordan is a 28-year old male surgery resident who has been taking night call shifts for the past six weeks, and has begun to develop a sleep disorder. What are the most likely negative impacts of sleep deprivation on LT Jordan’s work performance?

Noticeable weight change, hypervigilance, and increased conflict with other physicians and nurses.
Increased alcohol use, hyperarousal, decreased appetite.
Increased caffeine use, social isolation, and lower work productivity.
B. Increased conflict with other physicians and nurses, lower work productivity, impairments in memory, and increased risk for medical errors.

A

Correct answer: Increased conflict with other physicians and nurses, lower work productivity, impairments in memory, and increased risk for medical errors.

Explanation: Increased conflict with colleagues, lower work productivity, and impaired memory have all been observed in sleep-deprived individuals. Further, as compared to the other response options, all three of these behaviors would be most likely to contribute to impaired occupational performance.

Other answers:

Hypervigilance is not associated with sleep deprivation; in contrast, sleep deprivation impairs vigilance.

Hyperarousal and decreased appetite are not associated with sleep deprivation. While weight changes and eating behavior changes are observed in individuals who are not getting sufficient sleep, decreased appetite is not as strongly associated with sleep deprivation.

While increased caffeine use may be observed to compensate for drowsiness, this behavior would not cause as strong a reason for concern relating to occupational performance as the behaviors in the correct response. Additionally, while social isolation may indicate a mood disturbance precipitated by sleep deprivation, it is not directly associated with sleep deprivation and would not be as strong a contributor to occupational impairment.

172
Q

Your 32-year-old female patient presents with infertility and a long history of insomnia, which has been well controlled for the last 6 months with a prescription hypnotic. Lab results reveal increased serum prolactin and decreased serum testosterone levels. Which of the following is MOST likely to cause these adverse effects?

modafinil
phenobarbital
dantrolene
remelteon

A

The correct answer is ramelteon.

Explanation: Ramelteon is a melatonin agonist with short acting hypnotic properties that can be used for sleep onset insomnia. Of the mentioned medications, use of ramelteon has the highest risk of increased serum prolactin and decreased serum testosterone levels. Dantrolene and modafinil are a muscle relaxant and stimulant, respectively. Phenobarbitol is not generally prescribed to treat insomnia.

173
Q

During a routine annual visit your 50 year old patient with no previous significant cardiovascular history presents with a blood pressure of 135/86 mmHg and a heart rate of 75 beats per minute. Auscultation reveals normal heart sounds with no murmurs. Ultrasound reveals an ejection fraction of 65%. Lead I of the ECG shows a large net positive voltage in the QRS complex and AVF shows an isoelectric QRS complex voltage. This information supports the following interpretation:

Significant right axis deviation and probable right ventricular hypertrophy: repeat ultrasound specifically checking for pulmonary stenosis.

Mild left axis deviation: recommend that patient take a β-blocker.

Severe left axis deviation and probable left ventricular hypertrophy: admit this patient for cardiac catheterization

Normal axis: follow patient for developing hypertension.

A

Answer: normal axis: follow patient for developing hypertension.

Mean QRS axis is between 0º and 30º and is normal. Normal is about -30º to +100º. The characteristics of this ECG may occur for several normal reasons. Normal blood pressure is 120/80. This patient is prehypertensive by definition (systolic BP between120-139 or diastolic between 80-89 mmHg). BP can possibly be reduced with exercise and diet.

Incorrect answers:

“Severe left axis deviation and probable left ventricular hypertrophy: admit this patient for cardiac catheterization.” Left shift of the QRS would be accompanied by a mean axis of the QRS 120º. Lead I would be negative.

“Mild left axis deviation: recommend that patient take a β-blocker.” The patient does not have left axis deviation and even if he did, that would not be an indication for drug therapy since left axis deviation can occur for many reasons unrelated to hypertension. While drug therapy for hypertension might include a beta blocker, this patient has not met the criteria for having hypertension. BP can possibly be reduced with exercise and a diet with higher content of fruits and vegetables, and it is always appropriate to recommend these measures to anyone with borderline high blood pressure.

174
Q

A 23 year-old male is seen in your clinic with an inguinal hernia. With your examination you confirm that a hernia is present and you remember that if the hernia has a neck that is lateral to the inferior epigastric artery, it is classified as a/an

direct inguinal hernia
indirect inguinal hernia
parasagittal inguinal hernia
Cooper’s ligament hernia

A

Correct answer: indirect inguinal hernia

Explanation: An inguinal hernia is a protrusion of parietal peritoneum and viscera (such as omentum or small intestine) through a normal or abnormal opening from the cavity in which they belong. Inguinal hernias account for 75% of abdominal hernias and about 2/3 to 3/4 of these are of the indirect type.

Inguinal hernias can be classified anatomically by their relationship to the inferior epigastric artery. Those that proceed lateral to the inferior epigastric artery and course within the processus vaginalis on into the inguinal canal are classified as indirect inguinal hernias. Those that are medial to the inferior epigastric artery and course through or around the inguinal canal are classified as direct inguinal hernias. The classification of the hernia may directly impact the method that is selected for the anatomical repair of the hernia.

175
Q

Which of these cancer chemotherapeutic drugs is CORRECTLY paired with its mechanism of action?

5-fluorouricil - inhibition of hypoxanthine-guanine-phosphoribosyl transferase

etoposide - inhibition of DNA topoisomerase I

methotrexate - inhibition of dihydrofolate reductase

doxorubicin - inhibition of thymidylate synthetase

A

Correct answer: methotrexate – inhibition of dihydrofolate reductase

Explanation: To answer this question correctly you must know the mechanism of action of each of these commonly used cancer chemotherapeutic drugs. Knowing these mechanisms allows you to use them correctly and in appropriate combinations while avoiding predictable drug-drug interactions. The inhibition of dihydrofolate reductase by folate analogues is a recurring theme in many areas of pharmacology and is something that appears over and over on Step 1 exams.

Incorrect answers: The mechanism of action of 5-fluorouracil is irreversible inhibition of thymidylate synthesis (TS) after conversion of 5-FU to 5-FdUMP. Remember that TS is the enzyme that transfer the methyl group to 5-dUMP to create 5-dTMP, and essential precursor for DNA synthesis.

Doxorubicin is an intercalating agent that results in DNA damage, and an inhibitor of topoisomerase. It does not inhibit thymidylate synthetase.

Etoposide is an inhibitor of DNA topoisomerase II (the ATP-dependent topoisomerase), not topoisomerase I. This may seem like minutiae, but the distinction between the type I and type II topoisomerases is what makes it possible to use irinotecan AND etoposide in the same therapeutic protocol.

Note: Other commonly used cancer chemotherapy drugs are paclitaxel and vinca alkaloids. Paclitaxel freezes microtubules in the polymerized state. Vinca alkaloids like vincristine inhibit polymerization. This is why the two should never be used together.

176
Q

You are on duty when a soldier is brought to you who was found by the side of the road. He has a wrapping stained with dried blood on his abdomen and he is pale. His heart rate is 110 beats per minute and his blood pressure 100/65 mmHg. His hands are clammy. His hematocrit is 35 (normal 40-45). The ambulance tech estimates he has lost nearly 1 liter of blood. His heart rate and blood pressure are what they are because of:

abdominal compression
sympathetic vasoconstriction and cardiac excitation
sympathetic vasodilation and cardiac excitation
parasympathetic vasoconstriction and cardiac excitation

A

Correct answer: sympathetic vasoconstriction and cardiac excitation.

Explanation: Baroreceptors sense the acute drop in blood pressure created by hemorrhage. The reflex output is designed to raise blood pressure towards normal (120/80 mmHg). The reflex response will be to excite the sympathetic nervous system and inhibit the parasympathetic. The net result will be 1) an increase in heart rate via sympathetic stimulation and reduction in parasympathetic input to the SA and AV nodes, 2) an increase in contractility due primarily to increased sympathetic activity to the myocardium and a modest effect of reducing parasympathetic activity, and 3) vasoconstriction due almost entirely to sympathetic activation of vascular nerves.

Incorrect answers

“parasympathetic vasoconstriction and cardiac excitation” There is minimal innervation of the blood vessels by the parasympathetic nerves. Parasympathetics also have no influence on basal vascular tone. So, removal of parasympathetic activity will not cause vasoconstriction. Reduction in parasympathetic activity will increase heart rate and mildly increase contractility but not to the degree that sympathetic stimulation does.

“sympathetic vasodilation and cardiac excitation” Vasodilation would tend to lower blood pressure. Withdrawal of sympathetic activity would cause vasodilation and cardiac depression, both of which would reduce blood pressure and make the situation worse.

“abdominal compression” Abdominal compression may cause pain which could cause a transient increase in heart rate. Abdominal compression could transiently increase venous return and thus cardiac output. Neither of these could sustain an increase in blood pressure or heart rate. Blood is dried so any transients would have subsided. The activation of the sympathetic and deactivation of the parasympathetic nervous system mediated by the arterial baroreceptors would be strong and sustained and the most likely and efficient means to maintain blood pressure.

177
Q

A 58 year-old woman presents to the emergency room with shortness of breath. She recently returned home from a 10-hour flight. Based on her signs and symptoms, you determine there is a 50% likelihood she has a pulmonary embolism. CT angiography is performed and is positive. CT Angiography has a sensitivity of 90% and a specificity of 70%. Using the information provided, what is the likelihood she has a pulmonary embolism after the test?

Hint: Use a 2X2 table or the formulas below (Sn=sensitivity, Sp=specificity. P=probability):

LR+=Sn/(1-Sp) Odds=(1-p)/p

LR-=(1-Sn)/Sp P=Odds/(1+Odds)

75%
65%
85%
55%

A

Correct answer is C. 75%

Explanation:

There are at least two different ways to solve this problem. The first is with likelihood ratios, starting with the pre-test probability of 50%, as follows:

a) Pretest Odds = p/(1-p) = 0.5/0.5 = 1
b) Likelihood Ratio positive= Sn/(1-sp) = 0.9/0.3 = 3
c) Posttest Odds = pretest odds x LR+ = 1 x 3 = 3
d) Posttest Probability = odds/(1 + odds) = 3/(1+3) = ¾ = 75%

Secondly, complete a 2x2 table using the provided sensitivity and specificity. Then calculate the PPV = 75%.

178
Q

Your patient Mrs. Vine suffers from non-valvular atrial fibrillation, and you initiate dabigatran therapy. After five days, she returns to the clinic with large bruises on her legs and prolonged bleeding from her gums after flossing. To prevent future excessive bleeding, you decide to:

Inject recombinant factor VIIa
Administer vitamin K
Reduce her dosage and monitor closely
Infuse fresh frozen plasma

A

The correct answer is: reduce her dosage and monitor closely.

Dabigatran is a direct thrombin inhibitor. It does not require tests for monitoring its anticoagulant effect. Unlike heparin and warfarin, there is no specific therapy available to reverse the anticoagulant effect of dabigatran. Without a reliable test to gauge the efficiency of new oral anticoagulants like dabigatran, excessive bleeding can indicate an overdose. Because there is no approved antidote and severe bleeding is not apparent, reducing her dosage is the best course of action in this scenario.

179
Q

Cardiac muscle cells act as a syncytium because the cells are connected to each other. The structures in the membrane that allow ions to pass between cells are

desmosomes
occluding (tight) junctions
gap junctions
hemidesmosomes

A

Correct answer: gap junctions

Explanation: Gap junctions allow direct passage of molecules and ions to travel between cells and thus enable ionic continuity between adjacent cells.

Incorrect answers:

Desmosomes are spot welds that anchor cells together.

Hemidesmosomes are spot welds that anchor cells to the basement membrane.

Occluding (tight) junctions allow epithelial cells to function as a barrier.

180
Q

CPT Brady is a 29-year old female, fourth-year surgical resident who has presented to you, her primary care physician, for her annual physical exam. After taking a thorough medical and psychosocial history, you suspect that she might be at risk for professional burnout. According to Freudenberger & Richelson (1980), which of the following signs or symptoms reported by CPT Brady most strongly suggests that she might require immediate medical attention in the near future unless she seeks professional help and/or makes significant lifestyle changes?

Appearing physically exhausted and stating that she feels hopeless about her future as a surgeon.

Holding herself to a higher standard than her peers, even though she is already a top contender for chief resident.

Reporting the recent onset of somatic symptoms including stomach upset and tension headaches several days per week.

Sacrificing sleep and regular meals in order to perform more surgeries and demonstrating excessive ambition.

A

Correct answer: Appearing physically exhausted and stating that she feels hopeless about her future as a surgeon.

Explanation: All of the answers contain signs and behaviors of professional burnout that are included in Freudenberger & Richelson’s (1980) Burnout Cycle. However, overt physical exhaustion and endorsing hopelessness both occur toward the end of the cycle where individuals may experience physical and mental collapse indicative of full blown “Burnout Syndrome,” which requires medical intervention. All three of the alternate responses tend to present early in the Burnout Cycle. While they are maladaptive over time and may progress to the more severe syndrome without early intervention, they are not severe enough to warrant medical attention.

181
Q

LCDR Mason is a 37 year-old, African American female who has come to you for her annual physical examination. Upon reviewing her weight and height, you calculate that her BMI is 29. You determine that LCDR Mason appears to be motivated to lose weight and is enthusiastic about engaging in a structured weight loss regimen, so you decide to provide psychoeducation about the process. Which statement best describes a healthy approach to weight loss?

A healthy diet, exercise regimen, and behavior modification should be continued indefinitely even after an individual reaches his/her goal weight.

Weight loss requires both a decrease in energy intake and a decrease in energy expenditure.

It would be reasonable to aim for a 20% reduction in body weight over the course of 6 months.

Weight loss tends to get easier after approximately six months because adherence usually improves over time.

A

Correct answer: A healthy diet, exercise regimen, and behavior modification should be continued indefinitely even after an individual reaches his/her goal weight.

Explanation: The correct answer describes a healthy approach to weight loss because it reflects the long-term, lifestyle modifications that are required to maintain a healthy weight after an initial period of weight loss.

Other answers

Weight loss requires an increase (not decrease) in energy expenditure.

20% reduction in body weight is too ambitious of a weight loss goal for a 6-month period. A goal of 10% every 6 months is more reasonable and supported by the scientific literature.

Adherence to weight loss regimens tends to decrease over time, leading to increased difficulty of continued weight loss.

182
Q

A 26-year-old man is undergoing chemotherapy for Hodgkin’s lymphoma. Vaccination against which ONE of the following viruses is CONTRAINDICATED for use in his immediate household contacts (his wife or children)?

Human papilloma virus
Smallpox virus
Rabies virus
Hepatitis B virus

A

The correct answer is Smallpox virus.

Of the vaccines listed above, all are subunit or inactive virus-based vaccines except for the smallpox vaccine, which uses live vaccinia (cowpox) virus. In his immunocompromised state of impaired cell-mediated immunity, this patient is at risk of developing an infection with live virus from the vaccine if exposed. In addition to smallpox vaccination, vaccines that use live attenuated viruses should also generally be avoided in patients with substantially compromised cell-mediated immunity. These include MMR (measles/mumps/rubella), Varicella vaccine, Herpes zoster vaccine, and Yellow fever vaccine.

183
Q

You evaluate a 48-year-old obese male who complains of polyuria and polydipsia for the last 10 months. His fasting blood sugar is elevated and urinalysis discloses glucosuria. Which of the following statements is correct?

The clinical findings and epidemiology suggest type 1 diabetes mellitus

Diabetic ketoacidosis is a serious and common complication of this disease

Microscopic sections from the pancreas would probably disclose lymphocytic or eosinophilic insulitis

Insulin resistance by peripheral tissues and inadequate insulin secretion are probably the main metabolic defects of this patient

A

The correct answer is Insulin resistance by peripheral tissues and inadequate insulin secretion are probably the 2 main metabolic defects of this patient.

The clinical findings are consistent with type 2 diabetes mellitus. The main metabolic defects that characterize type 2 diabetes are insulin resistance by peripheral tissues and inadequate insulin secretion (Robbins, page 1132).

Incorrect answers:

Amyloid deposition within islets is characteristic of type 2 diabetes mellitus, insulitis as described in the question stem of type 1 (Robbins, page 1139).

Unlike type 1 diabetes mellitus, type 2 diabetes patients are often older (>40 years old) and obese (Robbins, page 1143).

Diabetic ketoacidosis is a serious complication of type 1 diabetes mellitus; it may also occur in type 2 diabetes, but is not as common, or as severe (Robbins, page 1143).

184
Q

A 75-year-old woman presents to your clinic with fever, focal seizures, and muscular weakness restricted to one side of the body (hemiparesis), MRI shows hemorrhagic lesions in inferior medial portion of her temporal lobe. What anti-viral treatment is recommended?

Acyclovir
Azidothymidine
Ribavirin
Oseltamivir

A

The correct answer is Acyclovir.

The development of encephalitis or central nervous system symptoms in an elderly person, particularly with lesions in the temporal lobe, is suggestive of herpes simplex type 1 infection that has reactivated and spread from the trigeminal nerve. HSV replication is inhibited by acyclovir, which becomes phosphorylated by the viral thymidine kinase and is then converted to a triphosphate nucleotide analogue by cellular kinases. The high affinity of this acyclo-GTP nucleotide analogue for the viral DNA polymerase allows it to be readily incorporated during DNA synthesis, but with no 3’ moiety, viral DNA replication terminates in infected cells.

Azidothymidine is a nucleoside analog reverse-transcriptase inhibitor predominantly used in combination with other antiretroviral agents for treatment of HIV. Oseltamivir is a neuraminidase inhibitor used in the treatment of influenza virus infection. Ribavirin is a nucleoside antimetabolite with modest activity against a number of viruses, including hepatitis C, influenza, respiratory syncytial virus, and some hemorrhagic fever viruses.

185
Q

A 4-year-old female injures her index finger and has soft tissue swelling about the distal interphalangeal joint on physical exam. Radiographs demonstrate widening of the physis compared to other adjacent asymptomatic physes. However, the metaphysis and epiphysis of the involved distal phalanx appear intact. Using the Salter-Harris classification system for pediatric fractures, this best describes what type of fracture?

Salter Harris type 2 fracture
Salter Harris type 4 fracture
Salter Harris type 1 fracture
Salter Harris type 3 fracture

A

CORRECT ANSWER: Salter Harris (SH) type 1 fracture is the correct answer for this question. SH type 1 fractures affect only the physis without extension to the metaphysis or epiphysis. These may demonstrate widening of the physis or a normal physis with soft tissue swelling that may become more apparent over time as the fracture heals or in comparison to the opposite asymptomatic hand.

OTHER ANSWERS:

A SH type 2 fracture would involve the physis as well as the metaphysis.

A SH type 3 fracture would involve the physis and epiphysis but spare the metaphysis.

A SH type 4 fracture would involve the physis as well as the metaphysis and epiphysis.

Of note, a SH type 5 fracture (not listed) would also affect only the physis but is typically a crush injury leading to narrowing or decrease of the height of the physis on radiography.

186
Q

You evaluate a 38-year-old man who presents with vesicles and bullae on the scalp, face, axilla, groin and points of pressure. Oral ulcers preceded these lesions by 6 months. A skin biopsy shows
intraepidermal blisters and dermal infiltrates of lymphocytes, histiocytes, and eosinophils. Which of the following statements best fits the disease process?

This disease is caused by IgG antibodies against desmogleins.

The patient likely has bullous pemphigoid.

The common histologic determinant of all forms of this disease is spongiosis.

Subepidermal, non-acantholytic blisters are characteristic of pemphigus vulgaris.

A

Correct answer: this disease is caused by IgG antibodies against desmogleins.

The clinical and morphologic findings are consistent with pemphigus vulgaris. Pemphigus is a rare autoimmune disease that typically presents with blisters of the skin and/or mucous membranes. The three main types of pemphigus are pemphigus vulgaris, pemphigus foliaceus, and paraneoplastic pemphigus. The most prevalent one is pemphigus vulgaris, accounting for 80% of cases. The common histologic determinant of all forms of pemphigus is acantholysis, which is disruption of intercellular connections between epithelial cells. Pemphigus is caused by IgG antibodies to demosmogleins, adhesions molecules that hold epithelial cells together. By disrupting the connections between eplithelial cells, the autoantibodies in pemphigus vulgaris cause superficial (or suprabasal…aka above the basement membrane) intraepidermal bullae with acantholysis. Immunofluorescence of skin biopsy would show antibodies binding to the outlines of individual epithelial cells. Because they are superficial, the bullae in pemphigus are fragile and frequently rupture. This makes patients with pemphigus at high risk for skin infections and scarring. Bullous pemphigoid differs from pemphigus in several respects. Patients with bullous pemphigoid have autoantibodies against BPAG ( a component of hemidesmosomes in the basement membrane of the epidermis). Consequently, bullae are subepidermal and tend to be intact. Immunofluorescence would reveal a linear pattern of antibodies at the base of the epidermis. Because individuals with bullous pemphigoid have less rupture of their bullae than those with pemphigus vulgaris, bullous pemphigoid tends to be less clinically severe than pemphigus vulgaris. Finally, patients with bullous pemphigoid tend not to have oral lesions.